CAT Questions | CAT Reading Comprehension

CAT Verbal Ability | RC for CAT

The following questions are from Reading Comprehension for Verbal Ability for CAT. Practice RC Passages for CAT. Please scroll the page to see them all. Reading Comprehension questions are an integral part of the CAT Exam. RC Passages and the accompanying questions account for around 24 questions out of 34 questions in CAT Verbal Section. If you would like to take these questions as a Quiz, head on here to take these questions in a test format, absolutely free.

Passage 1 : CAT Reading Comprehension: Power in language

The first systems of writing developed and used by the Germanic peoples were runic alphabets. The runes functioned as letters, but they were much more than just letters in the sense in which we today understand the term. Each rune was an ideographic or pictographic symbol of some cosmological principle or power, and to write a rune was to invoke and direct the force for which it stood. Indeed, in every Germanic language, the word “rune” (from Proto-Germanic *runo) means both “letter” and “secret” or “mystery,” and its original meaning, which likely predated the adoption of the runic alphabet, may have been simply “(hushed) message.”

Each rune had a name that hinted at the philosophical and magical significance of its visual form and the sound for which it stands, which was almost always the first sound of the rune’s name. For example, the T-rune, called *Tiwaz in the Proto-Germanic language, is named after the god Tiwaz (known as Tyr in the Viking Age). Tiwaz was perceived to dwell within the daytime sky, and, accordingly, the visual form of the T-rune is an arrow pointed upward (which surely also hints at the god’s martial role). The T-rune was often carved as a standalone ideograph, apart from the writing of any particular word, as part of spells cast to ensure victory in battle.

The runic alphabets are called “futharks” after the first six runes (Fehu, Uruz, Thurisaz, Ansuz, Raidho, Kaunan), in much the same way that the word “alphabet” comes from the names of the first two Hebrew letters (Aleph, Beth). There are three principal futharks: the 24-character Elder Futhark, the first fully-formed runic alphabet, whose development had begun by the first century CE and had been completed before the year 400; the 16-character Younger Futhark, which began to diverge from the Elder Futhark around the beginning of the Viking Age (c. 750 CE) and eventually replaced that older alphabet in Scandinavia; and the 33-character Anglo-Saxon Futhorc, which gradually altered and added to the Elder Futhark in England. On some inscriptions, the twenty-four runes of the Elder Futhark were divided into three ættir (Old Norse, “families”) of eight runes each, but the significance of this division is unfortunately unknown.

Runes were traditionally carved onto stone, wood, bone, metal, or some similarly hard surface rather than drawn with ink and pen on parchment. This explains their sharp, angular form, which was well-suited to the medium.

Much of our current knowledge of the meanings the ancient Germanic peoples attributed to the runes comes from the three “Rune Poems,” documents from Iceland, Norway, and England that provide a short stanza about each rune in their respective futharks (the Younger Futhark is treated in the Icelandic and Norwegian Rune Poems, while the Anglo-Saxon Futhorc is discussed in the Old English Rune Poem).

While runologists argue over many of the details of the historical origins of runic writing, there is widespread agreement on a general outline. The runes are presumed to have been derived from one of the many Old Italic alphabets in use among the Mediterranean peoples of the first century CE, who lived to the south of the Germanic tribes. Earlier Germanic sacred symbols, such as those preserved in northern European petroglyphs, were also likely influential in the development of the script.

The earliest possibly runic inscription is found on the Meldorf brooch, which was manufactured in the north of modern-day Germany around 50 CE. The inscription is highly ambiguous, however, and scholars are divided over whether its letters are runic or Roman. The earliest unambiguous runic inscriptions are found on the Vimose comb from Vimose, Denmark and the Øvre Stabu spearhead from southern Norway, both of which date to approximately 160 CE. The earliest known carving of the entire futhark, in order, is that on the Kylver stone from Gotland, Sweden, which dates to roughly 400 CE.

The transmission of writing from southern Europe to northern Europe likely took place via Germanic warbands, the dominant northern European military institution of the period, who would have encountered Italic writing firsthand during campaigns amongst their southerly neighbors. This hypothesis is supported by the association that runes have always had with the god Odin, who, in the Proto-Germanic period, under his original name *Woðanaz, was the divine model of the human warband leader and the invisible patron of the warband’s activities. The Roman historian Tacitus tells us that Odin (“Mercury” in the interpretatio romana) was already established as the dominant god in the pantheons of many of the Germanic tribes by the first century.

From the perspective of the ancient Germanic peoples themselves, however, the runes came from no source as mundane as an Old Italic alphabet. The runes were never “invented,” but are instead eternal, pre-existent forces that Odin himself discovered by undergoing a tremendous ordeal.

  1. The word “pantheon” in the passage refers to

    1. A temple of all the gods
    2. All the gods collectively of a religion
    3. A monument or building commemorating a nation's dead heroes
    4. A domed circular temple at Rome, erected a.d. 120–124 by Hadrian
    Choice B

  2. Which of the following statements is incorrect?

    1. Unlike the Latin alphabet, which is an essentially utilitarian script, the runes are symbols of some of the most powerful forces in the cosmos
    2. Runic writing was probably first used in southern Europe and was carried north by Germanic tribes.
    3. The word “rune” and its meaning was derived from the runic alphabet.
    4. The first runic alphabets date back to the 1st century CE.
    Choice C

  3. Which of the following can be inferred from the passage?
    a. Runic script was most likely derived from Old Italic script.
    b. Runes were not used so much as a simple writing system, but rather as magical signs to be used for charms.
    c. In the Proto-Germanic period, the god Tiwaz was associated with war, victory, marriage and the diurnal sky.
    d. The knowledge of the meanings attributed to the runes of the Younger Futhark is derived from the three Rune poems.

    1. All the above
    2. ii and iv
    3. i, ii and iv
    4. i and iii
    Choice D

  4. Which of the following cannot be reasonably inferred with regard to the beliefs of the Proto-Germanic people?

    1. Odin came upon the runes after going through a lot of torment.
    2. The name of a rune was almost always the first sound of a God’s name
    3. The cosmological power represented by a rune was invoked by writing it.
    4. Proto-German Gods were modeled on humans.
    Choice B

Passage 2 : CAT Reading Comprehension: Upholding the Law

Hard cases, it is said, make bad law. The adage is widely considered true for the Supreme Court of India which held in the height of the Emergency, in ADM Jabalpur v. Shivkant Shukla that detenus under the Maintenance of Internal Security Act (MISA) could not approach the judiciary if their fundamental rights were violated. Not only was the law laid down unconscionable, but it also smacked of a Court more “executive-minded than the executive”, complicit in its own independence being shattered by an all-powerful government. So deep has been the impact of this judgment that the Supreme Court’s current activist avatar is widely viewed as having its genesis in a continuing need to atone. Expressions of such atonement have created another Court made to measure — this time not to the measure of the government but rather the aggrandised self-image of some of its judges.

Let us look back to the ADM Jabalpur case. As a court of law, the Supreme Court was called upon in the case to balance the interest of public order in an Emergency with the right to life and personal liberty guaranteed to every person. Nine High Courts called upon to perform the same function had found a nuanced answer by which they had held that the right to life cannot be absolutely subservient to public order merely because the government declared so — the legality of detentions could be judicially reviewed, though the intention of the government would not be second-guessed by the Court. This was a delicate balance. The Supreme Court however reversed this view and made the right to life and personal liberty literally a bounty of the government. Given that the consequences of their error were entirely to the government’s advantage, it was widely viewed as the death of an independent judiciary. The excessively deferential, almost apologetic language used by the judges confirmed this impression.

Today, however, while public interest litigation has restored the independent image of the Supreme Court, it has achieved this at the cost of quality, discipline and the constitutional role judges are expected to perform. The Court monitors criminal trials, protects the environment, regulates political advertising, lays down norms for sexual harassment in the workplace, sets guidelines for adoption, supervises police reform among a range of other tasks of government. That all these tasks are crucial but tardily undertaken by government can scarcely be questioned. But for an unelected and largely unaccountable institution such as the Supreme Court to be at the forefront of matters relating to governance is equally dangerous — the choice of issues it takes up is arbitrary, their remit is not legal, their results often counterproductive, requiring a degree of technical competence and institutional capacity in ensuring compliance that the Court simply does not possess. This sets an unhealthy precedent for other courts and tribunals in the country, particularly the latter whose chairpersons are usually retired Supreme Court Justices. To take a particularly egregious example, the National Green Tribunal has banned diesel vehicles more than 10 years old in Delhi and if reports are to be believed, is considering imposing a congestion charge for cars as well. That neither of these are judicial functions and are being unjustly being usurped by a tribunal that has far exceeded its mandate, is evidence of the chain reaction that the Supreme Court’s activist avatar has set off across the judicial spectrum.

Finally, the Court’s activism adds to a massive backlog of regular cases that makes the Indian justice delivery mechanism, slow, unreliable and inefficient for the ordinary litigant. As on March 1, 2015, there were over 61,000 cases pending in the Supreme Court alone. It might be worthwhile for the Court to set its own house in order, concomitantly with telling other wings of government how to do so.

As we mark 40 years of the Emergency and the darkest period in the Supreme Court’s history, it might be time to not single-mindedly harp on the significance of an independent judiciary. Judicial independence, is and must remain a cherished virtue. However, it would be blinkered to not confront newer challenges that damage the credibility of our independent judiciary today — unpardonable delays and overweening judges taking on the mantle of national government by proxy. The Supreme Court 40 years on is a different institution — it must be cognizant of its history but not at the cost of being blind to its present.

  1. Which of the following is a suitable title for the passage?

    1. An Atonement Gone Too Far
    2. Sanctimony from a Ruined Pedestal
    3. The ADM Jabalpur's Case: The Supreme Court's Darkest Hour
    4. Overcompensating for Past Mistakes
    Choice A

  2. The author says that the Supreme Court was “more executive-minded than the executive” during the Emergency. Which of the following options captures the essence of what the writer means by the phrase: 'more “executive-minded than the executive”'?

    1. The Supreme Court abdicated its independence to an authoritarian government by embracing its perspective.
    2. The Supreme Court was more emphatic than the Government about exercising executive power under the MISA.
    3. The Supreme Court reflected the unconscionable actions taken by the government by upholding its laws.
    4. The Supreme Court wanted to curry favor with the government through its deferential decisions during Emergency.
    Choice B

  3. Which of the following cannot be reasonably inferred from the passage?

    1. The Supreme Court was complicit in curbing judicial independence during the Emergency.
    2. Public interest litigations have, post-Emergency, led to the judiciary overreaching into the realm of legislature.
    3. The Indian Judiciary ought not indulge in general supervisory jurisdiction to correct actions and policies of government.
    4. The Indian judiciary must be equipped with technical competence and institutional capacity to ensure compliance to orders passed in relation to public interest litigations.
    Choice D

  4. The word “egregious” in the passage is farthest in meaning to :

    1. outrageous
    2. flagitious
    3. distinguished
    4. arrant
    Choice C

  5. Which of the following is the author least likely to agree with?

    1. The rise in judicial activism is in danger making the Supreme Court diffuse and ineffective, encroaching into the functions of government.
    2. Where the Supreme Court is only moved for better governance and administration, which does not involve the exercise of any proper judicial function, it should refrain from acting.
    3. Adoption, police reform and environment issues are the remit of the judiciary.
    4. The Indian judicial system needs to focus on clearing the massive backlog of cases to re-establish its credibility.
    Choice C

Passage 3 : CAT Reading Comprehension: Upholding the Law

Sound the alarm! The kingdom of letters has admitted Trojan horses: James Frey, JT Leroy, Misha Defonseca, Margaret B. Jones, Herman Rosenblat, and now Matt McCarthy, portions of whose baseball memoir, the New York Times reports, are “incorrect, embellished or impossible.” The watchmen have let down their guards.

I write: Hold your horses. In the rush to diagnose these fake memoirs as symptoms of a diseased culture, we have failed to consider an equally plausible alternative. What if the exposure of fake memoirists is not due to an increased frequency of lying, but rather to our increased ability to root out liars and hold them accountable for their verisimilitudes? Perhaps the outings of these hoaxes mark not a blurring of the line between fact and fiction, but a further demarcation.

Indeed, it may be helpful to remember that the novel was born from exactly such confusion. One of the standards by which the earliest novels were judged was their ability to convince readers that their narratives were, in fact, real. Authors deployed several tricks to scaffold the illusion. 'Robinson Crusoe' was “written by himself,” according to the novel’s title page, which omitted Daniel Defoe’s name. Samuel Richardson’s novel 'Pamela', an attempt to instruct in good conduct through entertainment, was written as a series of letters penned by the heroine. In his preface to the novel, which excluded his name altogether, Richardson included several real letters from friends to whom he had shown the manuscript, but he changed the salutation from “Dear Author” to “Dear Editor” and even, writing under the guise of “editor,” praised “Pamela’s” letters. However, this was a lie, but not a hoax. Richardson wanted his novels to be read with "Historical Faith", since they contained, he believed, "the truth of the possible- the truth of human nature". Richardson’s authorship was revealed shortly after Pamela’s publication, but rather than serving time on Oprah’s couch, he was hailed as an innovator of the novelistic form.

Whereas novels were unashamedly fake memoirs at their conception, our recent hoaxes suggest that the line between the genres, once drawn, cannot easily be erased. This is in no small part due to the Internet’s surveillance. All along, historians had raised questions about Misha Defonseca, who claimed to have survived the Holocaust by living with a pack of wolves, but the engine of her downfall was her former publisher Jane Daniel’s blog. James Frey’s sine qua non of the fudged-memoir genre, A Million Little Pieces, was debunked by the website The Smoking Gun, which posted his actual arrest records and compared them to Frey’s embellished retellings. Deborah Lipstadt used her blog to gather evidence against Herman Rosenblat’s memoir.

If anything, you could argue that the fact-checkers are doing too good a job. There seems to be some risk that, in attempting to hold memoirs to journalistic standards of factuality, the watchdogs miss the forest for the trees, fixating on minor details in books whose general pictures are correct. The New York Times includes in its dossier against Matt McCarthy disputations by teammates who McCarthy alleges threatened children and made fun of Hispanics, as though their denials of having said such self-incriminating things were more trustworthy than McCarthy’s accusations. When Jose Canseco published his baseball memoirs Juiced and Vindicated, reviewers caviled over minor details and unsubstantiated claims, including that Alex Rodriguez had used steroids. Recent events have proven the gist of Canseco’s memoirs largely correct.

Indeed, it seems unlikely that, say, every claim in Casanova’s The Story of My Life would hold up to such scrutiny. And yet, if we knew this were the case, would we excise it from the canon? Writers’ enormous talents can sometimes render moot questions of their works’ factuality; our fraudsters, meanwhile, attempted to compensate for their meager talents by actually inhabiting their bloated fictions. They suffer not an excess of imagination, which can illuminate even the most mundane experiences, but a retreat from it. And yet simply because they lost their handles on the truth does not mean that the culture also has. Maybe the symptom of our age is not the fake memoirists themselves, but the catching of fake memoirists. In which case: Sound the church bells! The traitors are routed! The watchmen won!

  1. Which of the following is a suitable title for the passage?

    1. How to write a memoir
    2. The age of literary fraud
    3. Who is afraid of fake memoirists
    4. Writing in the age of the internet
    Choice C

  2. Which of the following is the author unlikely to agree with?

    1. There isn’t more literary fraud in our age. More fraud is coming to light due to the Internet’s surveillance.
    2. The line dividing novels and fake memoirs was never clear.
    3. As long as the main or essential part of a memoir is correct, it does not matter if lesser details do not stand up to verification.
    4. There exists now a widespread, diseased culture of literary fraud.
    Choice D

  3. With regard to the novel ‘Pamela’, the author states that Richardson’s artifice “ was a lie, but not a hoax”. What does he mean?

    1. It was an unintentional deception that contained the truth of human nature and was hence acceptable to readers.
    2. It was just a ploy to capture the imagination of the readers with the truth of the possible.
    3. It was a deception perpetrated simply to make money.
    4. It was a mere prank, and did not generate public interest.
    Choice B

  4. The word ‘verisimilitude’ in the passage is farthest in meaning to

    1. absurdity
    2. plausibility
    3. authenticity
    4. credibleness
    Choice A

Passage 4 : CAT Reading Comprehension: Power in language

Considered amongst the greatest works of Western literature, the Iliad, paired with its sequel, the Odyssey, is attributed to Homer.

However, that the author of the Iliad was not the same as the compiler of the fantastic tales in the Odyssey is arguable on several scores. The two epics belong to different literary types; the Iliad is essentially dramatic in its confrontation of opposing warriors who converse like the actors in Attic tragedy, while the Odyssey is cast as a novel narrated in more everyday human speech. In their physical structure, also, the two epics display an equally pronounced difference. The Odyssey is composed in six distinct cantos of four chapters ("books") each, whereas the Iliad moves unbrokenly forward with only one irrelevant episode in its tightly woven plot. Readers who examine psychological nuances see in the two works some distinctly different human responses and behavioral attitudes. For example, the Iliad voices admiration for the beauty and speed of horses, while the Odyssey shows no interest in these animals. The Iliad dismisses dogs as mere scavengers, while the poet of the Odyssey reveals a modern sentimental sympathy for Odysseus's faithful old hound, Argos.

But the most cogent argument for separating the two poems by assigning them to different authors is the archeological criterion of implied chronology. In the Iliad the Phoenicians are praised as skilled craftsmen working in metal and weavers of elaborate, much-prized garments. The shield which the metalworking god Hephaistos forges for Achilles in the Iliad seems inspired by the metal bowls with inlaid figures in action made by the Phoenicians and introduced by them into Greek and Etruscan commerce in the 8th century B.C. In contrast, in the Odyssey Greek sentiment toward the Phoenicians has undergone a drastic change. Although they are still regarded as clever craftsmen, in place of the Iliad's laudatory polydaidaloi ("of manifold skills") the epithet is parodied into polypaipaloi ("of manifold scurvy tricksters"), reflecting the competitive penetration into Greek commerce by traders from Phoenician Carthage in the 7th century B.C.

One thing, however, is certain: both epics were created without recourse to writing. Between the decline of Mycenaean and the emergence of classical Greek civilization—which is to say, from the late 12th to the mid-8th century B.C.—the inhabitants of the Greek lands had lost all knowledge of the syllabic script of their Mycenaean fore-bears and had not yet acquired from the easternmost shore of the Mediterranean that familiarity with Phoenician alphabetic writing from which classical Greek literacy (and in turn, Etruscan, Roman, and modern European literacy) derived. The same conclusion of illiterate composition may be reached from a critical inspection of the poems themselves. Among many races and in many different periods there has existed (and still exists sporadically) a form of purely oral and unwritten poetic speech, distinguishable from normal and printed literature by special traits that are readily recognizable and specifically distinctive. To this class the Homeric epics conform. Hence it would seem an inevitable inference that they must have been created either before the end of the 8th century B.C. or so shortly after that date that the use of alphabetic writing had not yet been developed sufficiently to record lengthy compositions. It is this illiterate environment that explains the absence of all contemporary historical record of the authors of the two great epics.

It is probable that Homer's name was applied to two distinct individuals differing in temperament and artistic accomplishment, born perhaps as much as a century apart, but practicing the same traditional craft of oral composition and recitation. Although each became known as "Homer, " it may be (as one ancient source asserts) that “homros “was a dialectal word for a blind man and so came to be used generically of the old and often sightless wandering reciters of heroic legends in the traditional meter of unrhymed dactylic hexameters. Thus there could have been many Homers. The two epics ascribed to Homer, however, have been as highly prized in modern as in ancient times for their marvelous vividness of expression, their keenness of personal characterization, their unflagging interest, whether in narration of action or in animated dramatic dialogue.

  1. Which of the following cannot be reasonably inferred from the passage?

    1. Before the 12th century BC , the use of syllabic writing existed in Ancient Greece.
    2. Phoenician traders flourished in Greece at the time the Homeric epics were composed.
    3. Greek, Roman and modern European literacy can be traced back to the Phoenicians.
    4. Iliad and Odyssey are purely oral poetic speech, set to rhyme.
    Choice D

  2. Which of the following can be characterized as the main idea of the passage?

    1. There could have been many Homers, old and often sightless wandering reciters of heroic legends.
    2. Attributing the composition of the Iliad and Odyssey to one Homer is erroneous.
    3. Both Iliad and Odyssey were created without recourse to writing.
    4. The Iliad and the Odyssey are of distinct literary types, physical structure and style.
    Choice B

  3. The term epithet as used in the passage is farthest in meaning to

    1. Sobriquet
    2. Moniker
    3. Nickname
    4. Jargon
    Choice D

The following questions are from IPMAT Rohtak and Indore sample papers. If you want to take these questions as a mock please click below.

IPMAT Rohtak Sample Paper Mock
IPMAT Indore Sample Paper Mock

Please note that the explanation button will take you to the IPMAT solution page.


Passage 5 : IPMAT Reading Comprehension

Read the passage below and answer the questions that follow :

Harold a professional man who had worked in an office for many years had a fearful dream. In it, he found himself in a land where small slug-like animals with slimy tentacles lived on people's bodies. The people tolerated the loathsome creatures because after many years they grew into elephants which then became the nation's system of transport, carrying everyone wherever he wanted to go. Harold suddenly realised that he himself was covered with these things, and he woke upscreaming. In a vivid sequence of pictures this dream dramatised for Harold what he had never been able to put in to words; he saw himself as letting society feed on his body in his early years so that it would carry him when he retired. He later threw off the "security bug" and took upfreelance work.

  1. IPMAT 2020 Sample Paper - IPM Rohtak Verbal Ability

    In his dream Harold found the loathsome creatures

    1. in his village
    2. in his own house
    3. in a different land
    4. in his office
    Choice C
    in a different land

  2. IPMAT 2020 Sample Paper - IPM Rohtak Verbal Ability

    Which one of the following phrases best helps to bring out the precise meaning of 'loathsome creatures'?

    1. Security bug and slimy tentacles
    2. Fearful dream and slug-like animals
    3. Slimy tentacles and slug-like animals
    4. slug-like animals and security bug
    Choice C
    Slimy tentacles and slug-like animals

  3. IPMAT 2020 Sample Paper - IPM Rohtak Verbal Ability

    The statement that 'he later threw off the security bug' means that

    1. Harold succeeded in overcoming the need for security
    2. Harold stopped giving much importance to dreams
    3. Harold started tolerating social victimisation
    4. Harold killed all the bugs troubled him
    Choice A
    Harold succeeded in overcoming the need for security

  4. IPMAT 2020 Sample Paper - IPM Rohtak Verbal Ability

    Harold's dream was fearful because

    1. it brought him face to face with reality
    2. it was full of vivid pictures of snakes
    3. he saw huge elephant in it
    4. in it he saw slimy creatures feeding on people's bodies
    Choice D
    in it he saw slimy creatures feeding on people's bodies

Passage 6 : IPMAT Reading Comprehension

Read the passage below and answer the questions that follow :

Concussions are brain injuries that occur when a person receives a blow to the head, face, or neck. Although most people who suffer a concussion experience initial bouts of dizziness, nausea, and drowsiness, these symptoms often disappear after a few days. The long-term effects of  concussions,however, are less understood and far more severe. 

Recent studies suggest that people who suffer multiple concussions are at significant risk for developing chronic traumatic encephalopathy (CTE), a degenerative brain disorder that causes a variety of dangerous mental and emotional problems to arise weeks, months, or even years after the initial injury. These psychological problems can include depression, anxiety, memory loss, inability to concentrate, and aggression. In extreme cases, people suffering from CTE have even committed suicide or homicide. The majority of people who developthese issues are athletes who participate in popular high-impact sports, especially football. Although new sports regulations and improvements in helmet technology can helpprotect players, amateur leagues, the sports media, and fans all bear some of the responsibility for reducing the incidence of these devastating injuries.Improvements in diagnostic technology have provided substantial evidence to link severe and often fatal psychological disorders to the head injuries that players receive while on the field. 

Recent autopsies performed on the brains of football players who have committed suicide have shown advanced cases of CTE in every single victim.In response to the growing understanding of this danger, the National Football League (NFL) has revised its safety regulations. Players who have suffered a head injury on the field must undergo a concussion sideline assessmenta series of mental and physical fitness tests before being allowed back in the game.

In an effort to diminish the amount of head and neck injuries on the field, NFL officials began enforcing stricter penalty calls for helmet-to-helmet contact, leading with the head, and hitting a defenseless player. Furthermore, as of 2010, if a player's helmet is accidentally wrenched from his head during play, the ball is immediately whistled dead. It is hoped that these new regulations, coupled with advances in helmet design, will reduce the number of concussions, and thus curb further cases of CTE. Efforts by the NFL and other professional sports leagues are certainly laudable; we should commend every attempt to protect the mental and physical health of players. However, new regulations at the professional level cannot protect amateur players, especially young people.

Fatal cases of CTE have been reported in victims as young as 21. Proper tackling form using the arms and shoulders to aim for a player's midsection should be taught at an early age. Youth, high school, and college leagues should also adopt safety rules even more stringent than those of the NFL. Furthermore, young athletes should be educated about the serious dangers of head injuries at an early age. Perhaps the most important factor in reducing the number of traumatic brain injuries, however, lies not with the players, the coaches, or the administrators, but with the media and fans. 

Sports media producers have become accustomed to showcasing the most aggressive tackles and the most intense plays. NFL broadcasts often replay especially violent collisions while the commentators marvel at the players physical prowess. Some sports highlights television programs even feature weekly countdowns of the hardest hits. When the media exalts such dangerous behavior, professionals are rewarded for injuring each other on the field and amateurs become more likely to try to imitate their favorite NFL athletes. Announcers, commentators, television producers, and sportswriters should engage in a collective effort to cease glorifying brutal plays. In turn, fans should stopexpecting their favorite players to put their lives on  the line for the purposes of entertainment. Players must not be encouraged to trade their careers, their health, their happiness, and even their lives for the sake of a game.

  1. IPMAT 2020 Sample Paper - IPM Rohtak Verbal Ability

    Based on information in the passage, it can be inferred that all of the following statements are true except

    1. tackling is not always dangerous; however, players who use improper tackling form may injure others
    2. scientists have established a definitive link between players who die untimely deaths and the onset of CTE
    3. NFL officials have done little to address the problem of CTE
    4. athletes who are praised for exceptionally brutal hits are likely to continue engaging in such dangerous behavior
    Choice C
    NFL officials have done little to address the problem of CTE

  2. IPMAT 2020 Sample Paper - IPM Rohtak Verbal Ability

    According to the passage, which of the following factors contribute(s) to the incidence of CTE in
    amateur players?
    I. inconsistent application of safety regulations for all levels
    II. lack of education about the dangers of head injuries
    III. amateur players' desire to emulate professionals

    1. l only
    2. II only
    3. I and II only
    4. II and III only
    Choice C
    I and II only

  3. IPMAT 2020 Sample Paper - IPM Rohtak Verbal Ability

    As used in paragraph 3, which is the best synonym for laudable?

    1. praiseworthy
    2. ineffectual
    3. memorable
    4. audacious
    Choice A
    praiseworthy

  4. IPMAT 2020 Sample Paper - IPM Rohtak Verbal Ability

    The author's tone in the final paragraph can best be described as

    1. remorseful
    2. hopeless
    3. perplexed
    4. insistent
    Choice D
    insistent

  5. IPMAT 2020 Sample Paper - IPM Rohtak Verbal Ability

    As used in the final paragraph, which is the best antonym for exalts?

    1. mitigates
    2. venerates
    3. mollifies
    4. expedites
    Choice A
    mitigates

  6. IPMAT 2020 Sample Paper - IPM Rohtak Verbal Ability

    In describing the sports media, the author emphasizes its

    1. responsibility
    2. entertainment value
    3. senselessness
    4. danger
    Choice A
    responsibility

  7. IPMAT 2020 Sample Paper - IPM Rohtak Verbal Ability

    In the final paragraph, the author mentions sports highlights television programs as an example of how
    I. the media glorifies violence
    II. amateurs learn to mimic professional athletes
    III. professional athletes gain approval

    1. I only
    2. II only
    3. I and II only
    4. II and III only
    Choice A
    I only

Passage 7 : IPMAT Reading Comprehension

Read the passage below and answer the questions that follow :

The ravages [of the storm] were terrible in America, Europe, and Asia. Towns were overthrown, forests uprooted, coasts devastated by the mountains of water which were precipitated on them, vessels cast on the shore, whole districts leveled by waterspouts, several thousand people crushed on land or drowned at sea; such were the traces of its (468) ________, left by this devastating tempest.
                       -Ralph Waldo Emerson

  1. IPMAT 2020 Sample Paper - IPM Rohtak Verbal Ability

    What is the meaning of the word over throwning the context of this passage?

    1. surrendered
    2. devastated
    3. capitulated
    4. Yielded
    Choice B
    devastated

  2. IPMAT 2020 Sample Paper - IPM Rohtak Verbal Ability

    What is the meaning of the word leveled as it is used in the passage?

    1. razed
    2. marked
    3. spread
    4. raised
    Choice A
    razed

  3. IPMAT 2020 Sample Paper - IPM Rohtak Verbal Ability

    Which word, if inserted in the blank, makes the most sense in the context of the passage?

    1. Velocity
    2. Fury
    3. reward
    4. benevolence
    Choice B
    Fury

Passage 8 : IPMAT Reading Comprehension

Read the following passage and choose the answer that is closest to each of the questions that are based on the passage.

Supposing half a dozen or a dozen men were cast ashore from a wreck on an uninhabited island and left to their own resources, one of course, according to his capacity, would be set to one business and one to another; the strongest to dig and to cut wood, and to build huts for the rest: the most dexterous to make shoes out of bark and coats out of skins; the best educated to look for iron or lead in the rocks, and to plan the channels for the irrigation of the fields. But though their labours were thus naturally severed, that small group of shipwrecked men would understand well enough that the speediest progress was to be made by helping each other-not by opposing each other; and they would know that this help could only be properly given so long as they were frank and open in their relations, and the difficulties which each lay under properly explained to the rest. So that any appearance of secrecy or separateness in the actions of any of them would instantly, and justly, be looked upon with suspicion by the rest, as the sign of some selfish or foolish proceeding on the part of the individual. If, for instance, the scientific man were found to have gone out at night, unknown to the rest, to alter the sluices, the others would think, and in all probability rightly think, that he wanted to get the best supply of water to his own field; and if the shoemaker refused to show them where the bark grew which he made the sandals of, they would naturally think, and in all probability rightly think, that he didn't want them to see how much there was of it, and that he meant to ask from them more corn and potatoes in exchange for his sandals than the trouble of making them deserved. And thus, although each man would have a portion of time to himself in which he was allowed to do what he chose without let or inquiry - so long as he was working in that particular business which he had undertaken for the common benefit, any secrecy on his part would be immediately supposed to mean mischief; and would require to be accounted for, or put an end to: and this all the more because, whatever the work might be, certainly there would be difficulties about it which, when once they were well explained, might be more or less done away with by the help of the rest; so that assuredly every one of them would advance with his labour not only more happily, but more profitably and quickly, by having no secrets, and by frankly bestowing, and frankly receiving, such help as lay in his way to get or to give. 

  1. IPMAT 2020 Question Paper - IPM Indore Verbal

    When a dozen men are cast away on an imaginary island, the best educated would look for metals in rocks because

    1. metals can be used to make weapons.
    2. such an island probably has unexploited resources.
    3. he may find it beneath him to dig or cut or make shoes.
    4. he is suited for such work.
    Choice D
    he is suited for such work.

  2. IPMAT 2020 Question Paper - IPM Indore Verbal

    The author states that any appearance of secrecy or separateness would instantly and justly be looked upon with suspicion. From this statement we may infer that

    1. what is secret is not what is separate
    2. secrecy is not exactly the same as separateness
    3. it is natural to be suspicious of secrecy
    4. it only takes an instant for a relationship to deteriorate
    Choice D
    it only takes an instant for a relationship to deteriorate

  3. IPMAT 2020 Question Paper - IPM Indore Verbal

    The instance of the shoemaker who refuses to show his source and asks for more corn and potatoes, is an example of

    1. a strong bargain.
    2. unfair practice.
    3. the system of barter.
    4. the intent to make trouble.
    Choice B
    unfair practice.

  4. IPMAT 2020 Question Paper - IPM Indore Verbal

    According to the author, whatever one's work might be

    1. hardships are going to be part of it.
    2. one cannot keep complaining.
    3. one should expect others to assure of help and advance our labours.
    4. one must offer help to others in order to receive help.
    Choice D
    one must offer help to others in order to receive help.

  5. IPMAT 2020 Question Paper - IPM Indore Verbal

    The author's belief is that for progress to happen

    1. a team should consist of people with multiple talents.
    2. co-operation among team members is essential.
    3. one must deal with those who are secretive.
    4. transparency among all concerned is mandatory.
    Choice D
    transparency among all concerned is mandatory.

  6. IPMAT 2020 Question Paper - IPM Indore Verbal

    The writer makes a hypothesis, which can be related to

    1. communities in general.
    2. an imaginary island, rich with resources.
    3. an ideal world of talented people.
    4. a primitive and unsophisticated world.
    Choice A
    communities in general.

Passage 9 : IPMAT Reading Comprehension

Read the following passage and choose the answer that is closest to each of the questions that are based on the passage.

The perennial debate over gender differences threatens to remain inconclusive. Stereotypes pertaining to male superiority and female submissiveness could be traced to earlier ages where assigned roles were needed as survival measures. But, can we today see a swing away from these stereotypes, or have they established a stranglehold on our perceptions? In this gendered world, we continue to live with notions that one's gender determines one's skills and preferences, from toys and colours to career choices. So the girl child will be presented with a Barbie doll, while the boy child will receive a Lego set.

Does that mean that our brains are different? This myth has been exploded by a British professor of cognitive neuroimaging. Her research attempts to establish how these stereotypes mould our ideas of ourselves. She examines how science has been misinterpreted or misused to ask the wrong questions, instead of challenging the status quo. She urges us to move beyond a binary view of people's brains and instead to see these as highly individualised, profoundly adaptable, and full of unbounded potential. Her conclusive findings establish that no brain differences can be found that are solely gender related. In other words, modern neuroscientists have identified no decisive category-defining differences between the brains of men and women.

As a result of these findings we owe it to ourselves to dump the myths and look at ourselves afresh. We need to recognise that the male and female brain debate is a distraction, besides being based on inaccuracies. It is possibly harmful too, because it can be used as a hook to justify saying there is no point in girls doing science because they do not have a science brain; or compelling boys to opt for science because their brains are shaped for that subject. It can also condemn boys for being emotional, as this is seen as a feminine trait. And, most dangerous of all, to proclaim that boys, not girls, are meant to lead.

  1. IPMAT 2020 Question Paper - IPM Indore Verbal

    The research of a British professor of cognitive neuroimaging has succeeded in establishing that

    1. the brains of men and women are alike.
    2. science needs to challenge the status quo.
    3. society must break away from attempts at stereotyping gender issues.
    4. the potential of a human brain is not directly linked to gender.
    Choice D
    the potential of a human brain is not directly linked to gender.

  2. IPMAT 2020 Question Paper - IPM Indore Verbal

    By referring to the world as "gendered' the writer wants to convey that

    1. gender differences can be detected right from childhood.
    2. society continues to be fixated on gender stereotypes.
    3. one's gender is bound to determine one's abilities.
    4. the debate on gender differences will never be resolved.
    Choice B
    society continues to be fixated on gender stereotypes.

  3. IPMAT 2020 Question Paper - IPM Indore Verbal

    One of the dangers in adopting a binary view of the human brain is that it can

    1. promote the notion of feminine and masculine traits.
    2. determine as well as limit academic choices.
    3. lead to the distortion and misinterpretation of scientific data.
    4. be used to encourage male dominance and superiority.
    Choice A
    promote the notion of feminine and masculine traits.

  4. IPMAT 2020 Question Paper - IPM Indore Verbal

    The writer of this passage wants to emphasise the need to

    1. use new insights provided by scientific research for a better understanding of human abilities.
    2. continuously debate issues of gender differences to achieve human progress.
    3. question the findings of scientific inquiry into the functioning of the human brain.
    4. accept gender differences as essential to the survival of the human species.
    Choice A
    use new insights provided by scientific research for a better understanding of human abilities.

  5. IPMAT 2020 Question Paper - IPM Indore Verbal

    The synonym for 'stranglehold' (Para 1) is

    1. asphyxiation
    2. containment
    3. prohibiting entry
    4. overwhelming control
    Choice D
    overwhelming control

  6. IPMAT 2020 Question Paper - IPM Indore Verbal

    The antonym for "unbounded' (Para 2) is

    1. imprisoned
    2. aggressive
    3. restricted
    4. fearful
    Choice C
    restricted

The Questions that follow, are from actual CAT papers. If you wish to take them separately or plan to solve actual CAT papers at a later point in time, It would be a good idea to stop here.


CAT VARC : CAT 2023 Question Paper Slot 1

The passage below is accompanied by four questions. Based on the passage, choose the best answer for each question.

Many human phenomena and characteristics – such as behaviors, beliefs, economies, genes, incomes, life expectancies, and other things – are influenced both by geographic factors and by non-geographic factors. Geographic factors mean physical and biological factors tied to geographic location, including climate, the distributions of wild plant and animal species, soils, and topography. Non-geographic factors include those factors subsumed under the term culture, other factors subsumed under the term history, and decisions by individual people. . . .

[T]he differences between the current economies of North and South Korea . . . cannot be attributed to the modest environmental differences between [them] . . . They are instead due entirely to the different [government] policies . . . At the opposite extreme, the Inuit and other traditional peoples living north of the Arctic Circle developed warm fur clothes but no agriculture, while equatorial lowland peoples around the world never developed warm fur clothes but often did develop agriculture. The explanation is straightforwardly geographic, rather than a cultural or historical quirk unrelated to geography. . . . Aboriginal Australia remained the sole continent occupied only by hunter/gatherers and with no indigenous farming or herding . . . [Here the] explanation is biogeographic: the Australian continent has no domesticable native animal species and few domesticable native plant species. Instead, the crops and domestic animals that now make Australia a food and wool exporter are all non-native (mainly Eurasian) species such as sheep, wheat, and grapes, brought to Australia by overseas colonists.

Today, no scholar would be silly enough to deny that culture, history, and individual choices play a big role in many human phenomena. Scholars don't react to cultural, historical, and individual-agent explanations by denouncing "cultural determinism," "historical determinism," or "individual determinism," and then thinking no further. But many scholars do react to any explanation invoking some geographic role, by denouncing "geographic determinism" . . .

Several reasons may underlie this widespread but nonsensical view. One reason is that some geographic explanations advanced a century ago were racist, thereby causing all geographic explanations to become tainted by racist associations in the minds of many scholars other than geographers. But many genetic, historical, psychological, and anthropological explanations advanced a century ago were also racist, yet the validity of newer non-racist genetic etc. explanations is widely accepted today.

Another reason for reflex rejection of geographic explanations is that historians have a tradition, in their discipline, of stressing the role of contingency (a favorite word among historians) based on individual decisions and chance. Often that view is warranted . . . But often, too, that view is unwarranted. The development of warm fur clothes among the Inuit living north of the Arctic Circle was not because one influential Inuit leader persuaded other Inuit in 1783 to adopt warm fur clothes, for no good environmental reason.

A third reason is that geographic explanations usually depend on detailed technical facts of geography and other fields of scholarship . . . Most historians and economists don't acquire that detailed knowledge as part of the professional training.

  1. CAT 2023 Slot 1 - VA RC

    All of the following can be inferred from the passage EXCEPT:

    1. agricultural practices changed drastically in the Australian continent after it was colonised.
    2. individual dictat and contingency were not the causal factors for the use of fur clothing in some very cold climates.
    3. while most human phenomena result from culture and individual choice, some have bio-geographic origins.
    4. several academic studies of human phenomena in the past involved racist interpretations.
    Choice C
    while most human phenomena result from culture and individual choice, some have bio-geographic origins.

  2. CAT 2023 Slot 1 - VA RC

    All of the following are advanced by the author as reasons why non-geographers disregard geographic influences on human phenomena EXCEPT their:

    1. dismissal of explanations that involve geographical causes for human behaviour.
    2. belief in the central role of humans, unrelated to physical surroundings, in influencing phenomena.
    3. lingering impressions of past geographic analyses that were politically offensive.
    4. disciplinary training which typically does not include technical knowledge of geography.
    Choice A
    dismissal of explanations that involve geographical causes for human behaviour.

  3. CAT 2023 Slot 1 - VA RC

    The examples of the Inuit and Aboriginal Australians are offered in the passage to show:

    1. how environmental factors lead to comparatively divergent paths in livelihoods and development.
    2. human resourcefulness across cultures in adapting to their surroundings.
    3. how physical circumstances can dictate human behaviour and cultures.
    4. that despite geographical isolation, traditional societies were self-sufficient and adaptive.
    Choice C
    how physical circumstances can dictate human behaviour and cultures.

  4. CAT 2023 Slot 1 - VA RC

    The author criticises scholars who are not geographers for all of the following reasons EXCEPT:

    1. their rejection of the role of biogeographic factors in social and cultural phenomena.
    2. their outdated interpretations of past cultural and historical phenomena.
    3. their labelling of geographic explanations as deterministic.
    4. the importance they place on the role of individual decisions when studying human phenomena.
    Choice B
    their outdated interpretations of past cultural and historical phenomena.

The passage below is accompanied by four questions. Based on the passage, choose the best answer for each question.

For early postcolonial literature, the world of the novel was often the nation. Postcolonial novels were usually [concerned with] national questions. Sometimes the whole story of the novel was taken as an allegory of the nation, whether India or Tanzania. This was important for supporting anti-colonial nationalism, but could also be limiting – land-focused and inward-looking.

My new book "Writing Ocean Worlds" explores another kind of world of the novel: not the village or nation, but the Indian Ocean world. The book describes a set of novels in which the Indian Ocean is at the centre of the story. It focuses on the novelists Amitav Ghosh, Abdulrazak Gurnah, Lindsey Collen and Joseph Conrad [who have] centred the Indian Ocean world in the majority of their novels. . . . Their work reveals a world that is outward-looking – full of movement, border-crossing and south-south interconnection. They are all very different – from colonially inclined (Conrad) to radically anti-capitalist (Collen), but together draw on and shape a wider sense of Indian Ocean space through themes, images, metaphors and language. This has the effect of remapping the world in the reader's mind, as centred in the interconnected global south. . . .

The Indian Ocean world is a term used to describe the very long-lasting connections among the coasts of East Africa, the Arab coasts, and South and East Asia. These connections were made possible by the geography of the Indian Ocean. For much of history, travel by sea was much easier than by land, which meant that port cities very far apart were often more easily connected to each other than to much closer inland cities. Historical and archaeological evidence suggests that what we now call globalisation first appeared in the Indian Ocean. This is the interconnected oceanic world referenced and produced by the novels in my book. . . .

For their part Ghosh, Gurnah, Collen and even Conrad reference a different set of histories and geographies than the ones most commonly found in fiction in English. Those [commonly found ones] are mostly centred in Europe or the US, assume a background of Christianity and whiteness, and mention places like Paris and New York. The novels in [my] book highlight instead a largely Islamic space, feature characters of colour and centralise the ports of Malindi, Mombasa, Aden, Java and Bombay. . . . It is a densely imagined, richly sensory image of a southern cosmopolitan culture which provides for an enlarged sense of place in the world.

This remapping is particularly powerful for the representation of Africa. In the fiction, sailors and travellers are not all European. . . . African, as well as Indian and Arab characters, are traders, nakhodas (dhow ship captains), runaways, villains, missionaries and activists. This does not mean that Indian Ocean Africa is romanticised. Migration is often a matter of force; travel is portrayed as abandonment rather than adventure, freedoms are kept from women and slavery is rife. What it does mean is that the African part of the Indian Ocean world plays an active role in its long, rich history and therefore in that of the wider world.

  1. CAT 2023 Slot 1 - VA RC

    All of the following claims contribute to the "remapping" discussed by the passage, EXCEPT:

    1. the global south, as opposed to the global north, was the first centre of globalisation.
    2. the world of early international trade and commerce was not the sole domain of white Europeans.
    3. cosmopolitanism originated in the West and travelled to the East through globalisation.
    4. Indian Ocean novels have gone beyond the specifics of national concerns to explore rich regional pasts.
    Choice C
    cosmopolitanism originated in the West and travelled to the East through globalisation.

  2. CAT 2023 Slot 1 - VA RC

    Which one of the following statements is not true about migration in the Indian Ocean world?

    1. Geographical location rather than geographical proximity determined the choice of destination for migrants.
    2. The Indian Ocean world's migration networks were shaped by religious and commercial histories of the region.
    3. The Indian Ocean world's migration networks connected the global north with the global south.
    4. Migration in the Indian Ocean world was an ambivalent experience.
    Choice C
    The Indian Ocean world's migration networks connected the global north with the global south.

  3. CAT 2023 Slot 1 - VA RC

    On the basis of the nature of the relationship between the items in each pair below, choose the odd pair out:

    1. Postcolonial novels : Border-crossing
    2. Postcolonial novels : Anti-colonial nationalism
    3. Indian Ocean world : Slavery
    4. Indian Ocean novels : Outward-looking
    Choice A
    Postcolonial novels : Border-crossing

  4. CAT 2023 Slot 1 - VA RC

    All of the following statements, if true, would weaken the passage's claim about the relationship between mainstream English-language fiction and Indian Ocean novels EXCEPT:

    1. most mainstream English-language novels have historically privileged the Christian, white, male experience of travel and adventure.
    2. the depiction of Africa in most Indian Ocean novels is driven by an Orientalist imagination of its cultural crudeness.
    3. very few mainstream English-language novels have historically been set in American and European metropolitan centres.
    4. the depiction of Africa in most Indian Ocean novels is driven by a postcolonial nostalgia for an idyllic past.
    Choice A
    most mainstream English-language novels have historically privileged the Christian, white, male experience of travel and adventure.

The passage below is accompanied by four questions. Based on the passage, choose the best answer for each question.

[Fifty] years after its publication in English [in 1972], and just a year since [Marshall] Sahlins himself died—we may ask: why did [his essay] "Original Affluent Society" have such an impact, and how has it fared since? . . . Sahlins's principal argument was simple but counterintuitive: before being driven into marginal environments by colonial powers, hunter-gatherers, or foragers, were not engaged in a desperate struggle for meager survival. Quite the contrary, they satisfied their needs with far less work than people in agricultural and industrial societies, leaving them more time to use as they wished. Hunters, he quipped, keep bankers' hours. Refusing to maximize, many were "more concerned with games of chance than with chances of game." . . . The so-called Neolithic Revolution, rather than improving life, imposed a harsher work regime and set in motion the long history of growing inequality . . .

Moreover, foragers had other options. The contemporary Hadza of Tanzania, who had long been surrounded by farmers, knew they had alternatives and rejected them. To Sahlins, this showed that foragers are not simply examples of human diversity or victimhood but something more profound: they demonstrated that societies make real choices. Culture, a way of living oriented around a distinctive set of values, manifests a fundamental principle of collective self-determination. . . .

But the point [of the essay] is not so much the empirical validity of the data—the real interest for most readers, after all, is not in foragers either today or in the Paleolithic—but rather its conceptual challenge to contemporary economic life and bourgeois individualism. The empirical served a philosophical and political project, a thought experiment and stimulus to the imagination of possibilities.

With its title's nod toward The Affluent Society (1958), economist John Kenneth Galbraith's famously skeptical portrait of America's postwar prosperity and inequality, and dripping with New Left contempt for consumerism, "The Original Affluent Society" brought this critical perspective to bear on the contemporary world. It did so through the classic anthropological move of showing that radical alternatives to the readers' lives really exist. If the capitalist world seeks wealth through ever greater material production to meet infinitely expansive desires, foraging societies follow "the Zen road to affluence": not by getting more, but by wanting less. If it seems that foragers have been left behind by "progress," this is due only to the ethnocentric self-congratulation of the West. Rather than accumulate material goods, these societies are guided by other values: leisure, mobility, and above all, freedom. . . .

Viewed in today's context, of course, not every aspect of the essay has aged well. While acknowledging the violence of colonialism, racism, and dispossession, it does not thematize them as heavily as we might today. Rebuking evolutionary anthropologists for treating present-day foragers as "left behind" by progress, it too can succumb to the temptation to use them as proxies for the Paleolithic. Yet these characteristics should not distract us from appreciating Sahlins's effort to show that if we want to conjure new possibilities, we need to learn about actually inhabitable worlds.

  1. CAT 2023 Slot 1 - VA RC

    We can infer that Sahlins's main goal in writing his essay was to:

    1. highlight the fact that while we started off as a fairly contented egalitarian people, we have progressively degenerated into materialism.
    2. put forth the view that, despite egalitarian origins, economic progress brings greater inequality and social hierarchies.
    3. hold a mirror to an acquisitive society, with examples of other communities that have chosen successfully to be non-materialistic.
    4. counter Galbraith's pessimistic view of the inevitability of a capitalist trajectory for economic growth.
    Choice C
    hold a mirror to an acquisitive society, with examples of other communities that have chosen successfully to be non-materialistic.

  2. CAT 2023 Slot 1 - VA RC

    The author mentions Tanzania's Hadza community to illustrate:

    1. that forager communities' lifestyles derived not from ignorance about alternatives, but from their own choice.
    2. how two vastly different ways of living and working were able to coexist in proximity for centuries.
    3. how pre-agrarian societies did not hamper the emergence of more advanced agrarian practices in contiguous communities.
    4. that hunter-gatherer communities' subsistence-level techniques equipped them to survive well into contemporary times.
    Choice A
    that forager communities' lifestyles derived not from ignorance about alternatives, but from their own choice.

  3. CAT 2023 Slot 1 - VA RC

    The author of the passage criticises Sahlins's essay for its:

    1. outdated values regarding present-day foragers versus ancient foraging communities.
    2. failure to supplement its thesis with robust empirical data.
    3. critique of anthropologists who disparage the choices of foragers in today's society.
    4. cursory treatment of the effects of racism and colonialism on societies.
    Choice D
    cursory treatment of the effects of racism and colonialism on societies.

  4. CAT 2023 Slot 1 - VA RC

    The author of the passage mentions Galbraith's "The Affluent Society" to:

    1. contrast the materialist nature of contemporary growth paths with the pacifist content ways of living among the foragers.
    2. document the influence of Galbraith's cynical views on modern consumerism on Sahlins's analysis of pre-historic societies.
    3. show how Galbraith's theories refute Sahlins's thesis on the contentment of pre-hunter-gatherer communities.
    4. show how Sahlins's views complemented Galbraith's criticism of the consumerism and inequality of contemporary society.
    Choice D
    show how Sahlins's views complemented Galbraith's criticism of the consumerism and inequality of contemporary society.

The passage below is accompanied by four questions. Based on the passage, choose the best answer for each question.

RESIDENTS of Lozère, a hilly department in southern France, recite complaints familiar to many rural corners of Europe. In remote hamlets and villages, with names such as Le Bacon and Le Bacon Vieux, mayors grumble about a lack of local schools, jobs, or phone and internet connections. Farmers of grazing animals add another concern: the return of wolves. Eradicated from France last century, the predators are gradually creeping back to more forests and hillsides. "The wolf must be taken in hand," said an aspiring parliamentarian, Francis Palombi, when pressed by voters in an election campaign early this summer. Tourists enjoy visiting a wolf park in Lozère, but farmers fret over their livestock and their livelihoods. . . .

As early as the ninth century, the royal office of the Luparii—wolf-catchers—was created in France to tackle the predators. Those official hunters (and others) completed their job in the 1930s, when the last wolf disappeared from the mainland. Active hunting and improved technology such as rifles in the 19th century, plus the use of poison such as strychnine later on, caused the population collapse. But in the early 1990s the animals reappeared. They crossed the Alps from Italy, upsetting sheep farmers on the French side of the border. Wolves have since spread to areas such as Lozère, delighting environmentalists, who see the predators' presence as a sign of wider ecological health. Farmers, who say the wolves cause the deaths of thousands of sheep and other grazing animals, are less cheerful. They grumble that green activists and politically correct urban types have allowed the return of an old enemy.

Various factors explain the changes of the past few decades. Rural depopulation is part of the story. In Lozère, for example, farming and a once-flourishing mining industry supported a population of over 140,000 residents in the mid-19th century. Today the department has fewer than 80,000 people, many in its towns. As humans withdraw, forests are expanding. In France, between 1990 and 2015, forest cover increased by an average of 102,000 hectares each year, as more fields were given over to trees. Now, nearly one-third of mainland France is covered by woodland of some sort. The decline of hunting as a sport also means more forests fall quiet. In the mid-to-late 20th century over 2m hunters regularly spent winter weekends tramping in woodland, seeking boars, birds and other prey. Today the Fédération Nationale des Chasseurs, the national body, claims 1.1m people hold hunting licences, though the number of active hunters is probably lower. The mostly protected status of the wolf in Europe—hunting them is now forbidden, other than when occasional culls are sanctioned by the state—plus the efforts of NGOs to track and count the animals, also contribute to the recovery of wolf populations.

As the lupine population of Europe spreads westwards, with occasional reports of wolves seen closer to urban areas, expect to hear of more clashes between farmers and those who celebrate the predators' return. Farmers' losses are real, but are not the only economic story. Tourist venues, such as parks where wolves are kept and the animals' spread is discussed, also generate income and jobs in rural areas.

  1. CAT 2023 Slot 1 - VA RC

    The inhabitants of Lozère have to grapple with all of the following problems, EXCEPT:

    1. poor rural communication infrastructure.
    2. lack of educational facilities.
    3. decline in the number of hunting licences.
    4. livestock losses.
    Choice C
    decline in the number of hunting licences.

  2. CAT 2023 Slot 1 - VA RC

    Which one of the following has NOT contributed to the growing wolf population in Lozère?

    1. The granting of a protected status to wolves in Europe.
    2. The shutting down of the royal office of the Luparii.
    3. An increase in woodlands and forest cover in Lozère.
    4. A decline in the rural population of Lozère.
    Choice B
    The shutting down of the royal office of the Luparii.

  3. CAT 2023 Slot 1 - VA RC

    The author presents a possible economic solution to an existing issue facing Lozère that takes into account the divergent and competing interests of:

    1. tourists and environmentalists.
    2. environmentalists and politicians.
    3. politicians and farmers.
    4. farmers and environmentalists.
    Choice D
    farmers and environmentalists.

  4. CAT 2023 Slot 1 - VA RC

    Which one of the following statements, if true, would weaken the author's claims?

    1. Wolf attacks on tourists in Lozère are on the rise.
    2. Having migrated out in the last century, wolves are now returning to Lozère.
    3. The old mining sites of Lozère are now being used as grazing pastures for sheep.
    4. Unemployment concerns the residents of Lozère.
    Choice A
    Wolf attacks on tourists in Lozère are on the rise.

CAT VARC : CAT 2023 Question Paper Slot 2

The passage below is accompanied by four questions. Based on the passage, choose the best answer for each question.
The Second Hand September campaign, led by Oxfam . . . seeks to encourage shopping at local organisations and charities as alternatives to fast fashion brands such as Primark and Boohoo in the name of saving our planet. As innocent as mindless scrolling through online shops may seem, such consumers are unintentionally—or perhaps even knowingly—contributing to an industry that uses more energy than aviation. . . .

Brits buy more garments than any other country in Europe, so it comes as no shock that many of those clothes end up in UK landfills each year: 300,000 tonnes of them, to be exact. This waste of clothing is destructive to our planet, releasing greenhouse gasses as clothes are burnt as well as bleeding toxins and dyes into the surrounding soil and water. As ecologist Chelsea Rochman bluntly put it, "The mismanagement of our waste has even come back to haunt us on our dinner plate."

It's not surprising, then, that people are scrambling for a solution, the most common of which is second-hand shopping. Retailers selling consigned clothing are currently expanding at a rapid rate . . . If everyone bought just one used item in a year, it would save 449 million lbs of waste, equivalent to the weight of 1 million Polar bears. "Thrifting" has increasingly become a trendy practice. London is home to many second-hand, or more commonly coined 'vintage', shops across the city from Bayswater to Brixton.

So you're cool and you care about the planet; you've killed two birds with one stone. But do people simply purchase a second-hand item, flash it on Instagram with #vintage and call it a day without considering whether what they are doing is actually effective?

According to a study commissioned by Patagonia, for instance, older clothes shed more microfibres. These can end up in our rivers and seas after just one wash due to the worn material, thus contributing to microfibre pollution. To break it down, the amount of microfibres released by laundering 100,000 fleece jackets is equivalent to as many as 11,900 plastic grocery bags, and up to 40 per cent of that ends up in our oceans. . . . So where does this leave second-hand consumers? [They would be well advised to buy] high-quality items that shed less and last longer [as this] combats both microfibre pollution and excess garments ending up in landfills. . . .

Luxury brands would rather not circulate their latest season stock around the globe to be sold at a cheaper price, which is why companies like ThredUP, a US fashion resale marketplace, have not yet caught on in the UK. There will always be a market for consignment but there is also a whole generation of people who have been taught that only buying new products is the norm; second-hand luxury goods are not in their psyche. Ben Whitaker, director at Liquidation Firm B-Stock, told Prospect that unless recycling becomes cost-effective and filters into mass production, with the right technology to partner it, "high-end retailers would rather put brand before sustainability."

  1. CAT 2023 Slot 2 - VA RC

    The central idea of the passage would be undermined if:

    1. Primark and Boohoo recycled their clothes for vintage stores.
    2. customers bought all their clothes online.
    3. second-hand stores sold only high-quality clothes.
    4. clothes were not thrown and burnt in landfills.
    Choice C
    second-hand stores sold only high-quality clothes.

  2. CAT 2023 Slot 2 - VA RC

    The act of "thrifting", as described in the passage, can be considered ironic because it:

    1. has created environmental problems.
    2. is an anti-consumerist attitude.
    3. is not cost-effective for retailers.
    4. offers luxury clothing at cut-rate prices.
    Choice A
    has created environmental problems.

  3. CAT 2023 Slot 2 - VA RC

    Based on the passage, we can infer that the opposite of fast fashion, 'slow fashion', would most likely refer to clothes that:

    1. do not shed microfibres.
    2. are of high quality and long lasting.
    3. are sold by genuine vintage stores.
    4. do not bleed toxins and dyes.
    Choice B
    are of high quality and long lasting.

  4. CAT 2023 Slot 2 - VA RC

    According to the author, companies like ThredUP have not caught on in the UK for all of the following reasons EXCEPT that:

    1. recycling is currently not financially attractive for luxury brands.
    2. luxury brands do not like their product to be devalued.
    3. the British don't buy second-hand clothing.
    4. luxury brands want to maintain their brand image.
    Choice C
    the British don't buy second-hand clothing.

The passage below is accompanied by four questions. Based on the passage, choose the best answer for each question.

Over the past four centuries liberalism has been so successful that it has driven all its opponents off the battlefield. Now it is disintegrating, destroyed by a mix of hubris and internal contradictions, according to Patrick Deneen, a professor of politics at the University of Notre Dame. . . . Equality of opportunity has produced a new meritocratic aristocracy that has all the aloofness of the old aristocracy with none of its sense of noblesse oblige. Democracy has degenerated into a theatre of the absurd. And technological advances are reducing ever more areas of work into meaningless drudgery. "The gap between liberalism's claims about itself and the lived reality of the citizenry" is now so wide that "the lie can no longer be accepted," Mr Deneen writes. What better proof of this than the vision of 1,000 private planes whisking their occupants to Davos to discuss the question of "creating a shared future in a fragmented world"? . . .

Deneen does an impressive job of capturing the current mood of disillusionment, echoing left-wing complaints about rampant commercialism, right-wing complaints about narcissistic and bullying students, and general worries about atomisation and selfishness. But when he concludes that all this adds up to a failure of liberalism, is his argument convincing? . . . He argues that the essence of liberalism lies in freeing individuals from constraints. In fact, liberalism contains a wide range of intellectual traditions which provide different answers to the question of how to trade off the relative claims of rights and responsibilities, individual expression and social ties. . . . liberals experimented with a range of ideas from devolving power from the centre to creating national education systems.

Mr Deneen's fixation on the essence of liberalism leads to the second big problem of his book: his failure to recognise liberalism's ability to reform itself and address its internal problems. The late 19th century saw America suffering from many of the problems that are reappearing today, including the creation of a business aristocracy, the rise of vast companies, the corruption of politics and the sense that society was dividing into winners and losers. But a wide variety of reformers, working within the liberal tradition, tackled these problems head on. Theodore Roosevelt took on the trusts. Progressives cleaned up government corruption. University reformers modernised academic syllabuses and built ladders of opportunity. Rather than dying, liberalism reformed itself.

Mr Deneen is right to point out that the record of liberalism in recent years has been dismal. He is also right to assert that the world has much to learn from the premodern notions of liberty as self-mastery and self-denial. The biggest enemy of liberalism is not so much atomisation but old-fashioned greed, as members of the Davos elite pile their plates ever higher with perks and share options. But he is wrong to argue that the only way for people to liberate themselves from the contradictions of liberalism is "liberation from liberalism itself". The best way to read "Why Liberalism Failed" is not as a funeral oration but as a call to action: up your game, or else.

  1. CAT 2023 Slot 2 - VA RC

    The author of the passage faults Deneen's conclusions for all of the following reasons, EXCEPT:

    1. its repeated harking back to premodern notions of liberty.
    2. its failure to note historical instances in which the process of declining liberalism has managed to reverse itself.
    3. its very narrow definition of liberalism limited to individual freedoms.
    4. its extreme pessimism about the future of liberalism today and predictions of an ultimate decline.
    Choice A
    its repeated harking back to premodern notions of liberty.

  2. CAT 2023 Slot 2 - VA RC

    The author of the passage is likely to disagree with all of the following statements, EXCEPT:

    1. the essence of liberalism lies in greater individual self-expression and freedoms.
    2. liberalism was the dominant ideal in the past century, but it had to reform itself to remain so.
    3. claims about liberalism's disintegration are exaggerated and misunderstand its core features.
    4. if we accept that liberalism is a dying ideal, we must work to find a viable substitute.
    Choice B
    liberalism was the dominant ideal in the past century, but it had to reform itself to remain so.

  3. CAT 2023 Slot 2 - VA RC

    All of the following statements are evidence of the decline of liberalism today, EXCEPT:

    1. ". . . the creation of a business aristocracy, the rise of vast companies . . ."
    2. "And technological advances are reducing ever more areas of work into meaningless drudgery."
    3. "Democracy has degenerated into a theatre of the absurd."
    4. "'The gap between liberalism's claims about itself and the lived reality of the citizenry' is now so wide that 'the lie can no longer be accepted,'. . ."
    Choice B
    "And technological advances are reducing ever more areas of work into meaningless drudgery."

  4. CAT 2023 Slot 2 - VA RC

    The author of the passage refers to "the Davos elite" to illustrate his views on:

    1. the way the debate around liberalism has been captured by the rich who have managed to insulate themselves from economic hardships.
    2. the fact that the rise in liberalism had led to a greater interest in shared futures from unlikely social classes.
    3. the unlikelihood of a return to the liberalism of the past as long as the rich continue to benefit from the decline in liberal values.
    4. the hypocrisy of the liberal rich, who profess to subscribe to liberal values while cornering most of the wealth.
    Choice D
    the hypocrisy of the liberal rich, who profess to subscribe to liberal values while cornering most of the wealth.

The passage below is accompanied by four questions. Based on the passage, choose the best answer for each question.

The Positivists, anxious to stake out their claim for history as a science, contributed the weight of their influence to the cult of facts. First ascertain the facts, said the positivists, then draw your conclusions from them. . . . This is what may [be] called the common-sense view of history. History consists of a corpus of ascertained facts. The facts are available to the historian in documents, inscriptions, and so on . . . [Sir George Clark] contrasted the "hard core of facts" in history with the surrounding pulp of disputable interpretation forgetting perhaps that the pulpy part of the fruit is more rewarding than the hard core. . . . It recalls the favourite dictum of the great liberal journalist C. P. Scott: "Facts are sacred, opinion is free.". . .

What is a historical fact? . . . According to the common-sense view, there are certain basic facts which are the same for all historians and which form, so to speak, the backbone of history—the fact, for example, that the Battle of Hastings was fought in 1066. But this view calls for two observations. In the first place, it is not with facts like these that the historian is primarily concerned. It is no doubt important to know that the great battle was fought in 1066 and not in 1065 or 1067, and that it was fought at Hastings and not at Eastbourne or Brighton. The historian must not get these things wrong. But [to] praise a historian for his accuracy is like praising an architect for using well-seasoned timber or properly mixed concrete in his building. It is a necessary condition of his work, but not his essential function. It is precisely for matters of this kind that the historian is entitled to rely on what have been called the "auxiliary sciences" of history—archaeology, epigraphy, numismatics, chronology, and so forth. . . .

The second observation is that the necessity to establish these basic facts rests not on any quality in the facts themselves, but on an apriori decision of the historian. In spite of C. P. Scott's motto, every journalist knows today that the most effective way to influence opinion is by the selection and arrangement of the appropriate facts. It used to be said that facts speak for themselves. This is, of course, untrue. The facts speak only when the historian calls on them: it is he who decides to which facts to give the floor, and in what order or context. . . . The only reason why we are interested to know that the battle was fought at Hastings in 1066 is that historians regard it as a major historical event. . . . Professor Talcott Parsons once called [science] "a selective system of cognitive orientations to reality." It might perhaps have been put more simply. But history is, among other things, that. The historian is necessarily selective. The belief in a hard core of historical facts existing objectively and independently of the interpretation of the historian is a preposterous fallacy, but one which it is very hard to eradicate.

  1. CAT 2023 Slot 2 - VA RC

    All of the following, if true, can weaken the passage's claim that facts do not speak for themselves, EXCEPT:

    1. the truth value of a fact is independent of the historian who expresses it.
    2. facts, like truth, can be relative: what is fact for person X may not be so for person Y.
    3. a fact, by its very nature, is objective and universal, irrespective of the context in which it is placed.
    4. the order in which a series of facts is presented does not have any bearing on the production of meaning.
    Choice B
    facts, like truth, can be relative: what is fact for person X may not be so for person Y.

  2. CAT 2023 Slot 2 - VA RC

    If the author of the passage were to write a book on the Battle of Hastings along the lines of his/her own reasoning, the focus of the historical account would be on:

    1. deriving historical facts from the relevant documents and inscriptions.
    2. providing a nuanced interpretation by relying on the auxiliary sciences.
    3. exploring the socio-political and economic factors that led to the Battle.
    4. producing a detailed timeline of the various events that led to the Battle.
    Choice C
    exploring the socio-political and economic factors that led to the Battle.

  3. CAT 2023 Slot 2 - VA RC

    According to this passage, which one of the following statements best describes the significance of archaeology for historians?

    1. Archaeology helps historians to carry out their primary duty.
    2. Archaeology helps historians to interpret historical facts.
    3. Archaeology helps historians to ascertain factual accuracy.
    4. Archaeology helps historians to locate the oldest civilisations in history.
    Choice C
    Archaeology helps historians to ascertain factual accuracy.

  4. CAT 2023 Slot 2 - VA RC

    All of the following describe the "common-sense view" of history, EXCEPT:

    1. history is like science: a selective system of cognitive orientations to reality.
    2. only the positivist methods can lead to credible historical knowledge.
    3. history can be objective like the sciences if it is derived from historical facts.
    4. real history can be found in ancient engravings and archival documents.
    Choice A
    history is like science: a selective system of cognitive orientations to reality.

The passage below is accompanied by four questions. Based on the passage, choose the best answer for each question.

Umberto Eco, an Italian writer, was right when he said the language of Europe is translation. Netflix and other deep-pocketed global firms speak it well. Just as the EU employs a small army of translators and interpreters to turn intricate laws or impassioned speeches of Romanian MEPs into the EU's 24 official languages, so do the likes of Netflix. It now offers dubbing in 34 languages and subtitling in a few more. . . .

The economics of European productions are more appealing, too. American audiences are more willing than before to give dubbed or subtitled viewing a chance. This means shows such as "Lupin", a French crime caper on Netflix, can become global hits. . . . In 2015, about 75% of Netflix's original content was American; now the figure is half, according to Ampere, a media-analysis company. Netflix has about 100 productions under way in Europe, which is more than big public broadcasters in France or Germany. . . .

Not everything works across borders. Comedy sometimes struggles. Whodunits and bloodthirsty maelstroms between arch Romans and uppity tribesmen have a more universal appeal. Some do it better than others. Barbarians aside, German television is not always built for export, says one executive, being polite. A bigger problem is that national broadcasters still dominate. Streaming services, such as Netflix or Disney+, account for about a third of all viewing hours, even in markets where they are well-established. Europe is an ageing continent. The generation of teens staring at phones is outnumbered by their elders who prefer to gawp at the box.

In Brussels and national capitals, the prospect of Netflix as a cultural hegemon is seen as a threat. "Cultural sovereignty" is the watchword of European executives worried that the Americans will eat their lunch. To be fair, Netflix content sometimes seems stuck in an uncanny valley somewhere in the mid-Atlantic, with local quirks stripped out. Netflix originals tend to have fewer specific cultural references than shows produced by domestic rivals, according to Enders, a market analyst. The company used to have an imperial model of commissioning, with executives in Los Angeles cooking up ideas French people might like. Now Netflix has offices across Europe. But ultimately the big decisions rest with American executives. This makes European politicians nervous.

They should not be. An irony of European integration is that it is often American companies that facilitate it. Google Translate makes European newspapers comprehensible, even if a little clunky, for the continent's non-polyglots. American social-media companies make it easier for Europeans to talk politics across borders. (That they do not always like to hear what they say about each other is another matter.) Now Netflix and friends pump the same content into homes across a continent, making culture a cross-border endeavour, too. If Europeans are to share a currency, bail each other out in times of financial need and share vaccines in a pandemic, then they need to have something in common—even if it is just bingeing on the same series. Watching fictitious northern and southern Europeans tear each other apart 2,000 years ago beats doing so in reality.

  1. CAT 2023 Slot 2 - VA RC

    Based on information provided in the passage, all of the following are true, EXCEPT:

    1. European television productions have the potential to become global hits.
    2. only half of Netflix's original programming in the EU is now produced in America.
    3. national broadcasters dominate in the EU in terms of total television viewing hours.
    4. Netflix has been able to transform itself into a truly European entity.
    Choice D
    Netflix has been able to transform itself into a truly European entity.

  2. CAT 2023 Slot 2 - VA RC

    The author sees the rise of Netflix in Europe as:

    1. a unifying force.
    2. a looming cultural threat.
    3. filling an entertainment gap.
    4. an economic threat.
    Choice A
    a unifying force.

  3. CAT 2023 Slot 2 - VA RC

    Which one of the following research findings would weaken the author's conclusion in the final paragraph?

    1. Research shows that Netflix has been gradually losing market share to other streaming television service providers.
    2. Research shows there is a wide variance in the popularity and viewing of Netflix shows across different EU countries.
    3. Research shows that older women across the EU enjoy watching romantic comedies on Netflix, whereas younger women prefer historical fiction dramas.
    4. Research shows that Netflix hits produced in France are very popular with North American audiences.
    Choice B
    Research shows there is a wide variance in the popularity and viewing of Netflix shows across different EU countries.

  4. CAT 2023 Slot 2 - VA RC

    Based only on information provided in the passage, which one of the following hypothetical Netflix shows would be most successful with audiences across the EU?

    1. A trans-Atlantic romantic drama set in Europe and America.
    2. An original German TV science fiction production.
    3. A murder mystery drama set in North Africa and France.
    4. An Italian comedy show hosted by an international star.
    Choice C
    A murder mystery drama set in North Africa and France.

CAT VARC : CAT 2023 Question Paper Slot 3

The passage below is accompanied by four questions. Based on the passage, choose the best answer for each question.

The biggest challenge [The Nutmeg's Curse by Ghosh] throws down is to the prevailing understanding of when the climate crisis started. Most of us have accepted . . . that it started with the widespread use of coal at the beginning of the Industrial Age in the 18th century and worsened with the mass adoption of oil and natural gas in the 20th. Ghosh takes this history at least three centuries back, to the start of European colonialism in the 15th century. He [starts] the book with a 1621 massacre by Dutch invaders determined to impose a monopoly on nutmeg cultivation and trade in the Banda islands in today's Indonesia. Not only do the Dutch systematically depopulate the islands through genocide, they also try their best to bring nutmeg cultivation into plantation mode. These are the two points to which Ghosh returns through examples from around the world. One, how European colonialists decimated not only indigenous populations but also indigenous understanding of the relationship between humans and Earth. Two, how this was an invasion not only of humans but of the Earth itself, and how this continues to the present day by looking at nature as a 'resource' to exploit. . . . We know we are facing more frequent and more severe heatwaves, storms, floods, droughts and wildfires due to climate change. We know our expansion through deforestation, dam building, canal cutting – in short, terraforming, the word Ghosh uses – has brought us repeated disasters . . . Are these the responses of an angry Gaia who has finally had enough? By using the word 'curse' in the title, the author makes it clear that he thinks so. I use the pronoun 'who' knowingly, because Ghosh has quoted many non-European sources to enquire into the relationship between humans and the world around them so that he can question the prevalent way of looking at Earth as an inert object to be exploited to the maximum. As Ghosh's text, notes and bibliography show once more, none of this is new. There have always been challenges to the way European colonialists looked at other civilisations and at Earth. It is just that the invaders and their myriad backers in the fields of economics, politics, anthropology, philosophy, literature, technology, physics, chemistry, biology have dominated global intellectual discourse. . . . There are other points of view that we can hear today if we listen hard enough. Those observing global climate negotiations know about the Latin American way of looking at Earth as Pachamama (Earth Mother). They also know how such a framing is just provided lip service and is ignored in the substantive portions of the negotiations. In The Nutmeg's Curse, Ghosh explains why. He shows the extent of the vested interest in the oil economy – not only for oil-exporting countries, but also for a superpower like the US that controls oil drilling, oil prices and oil movement around the world. Many of us know power utilities are sabotaging decentralised solar power generation today because it hits their revenues and control. And how the other points of view are so often drowned out.

  1. CAT 2023 Slot 3 - VA RC

    On the basis of information in the passage, which one of the following is NOT a reason for the failure of policies seeking to address climate change?

    1. The greed of organisations benefiting from non-renewable energy resources.
    2. The marginalised status of non-European ways of looking at nature and the environment.
    3. The decentralised characteristic of renewable energy resources like solar power.
    4. The global dominance of oil economies and international politics built around it.
    Choice C
    The decentralised characteristic of renewable energy resources like solar power.

  2. CAT 2023 Slot 3 - VA RC

    Which one of the following best explains the primary purpose of the discussion of the colonisation of the Banda islands in "The Nutmeg's Curse"?

    1. To illustrate how systemic violence against the colonised constituted the cornerstone of colonialism.
    2. To illustrate the first instance in history when the processes responsible for climate change were initiated.
    3. To illustrate the role played by the cultivation of certain crops in the plantation mode in contributing to climate change.
    4. To illustrate how colonialism represented and perpetuated the mindset that has led to climate change.
    Choice D
    To illustrate how colonialism represented and perpetuated the mindset that has led to climate change.

  3. CAT 2023 Slot 3 - VA RC

    All of the following can be inferred from the reviewer's discussion of "The Nutmeg's Curse", EXCEPT:

    1. academic discourses have always served the function of raising awareness about environmental preservation.
    2. environmental preservation policy makers can learn a lot from non-European and/or pre-colonial societies.
    3. the contemporary dominant perception of nature and the environment was put in place by processes of colonialism.
    4. the history of climate change is deeply intertwined with the history of colonialism.
    Choice A
    academic discourses have always served the function of raising awareness about environmental preservation.

  4. CAT 2023 Slot 3 - VA RC

    Which one of the following, if true, would make the reviewer's choice of the pronoun "who" for Gaia inappropriate?

    1. Modern western science discovers new evidence for the Earth being an inanimate object.
    2. Ghosh's book has a different title: "The Nutmeg's Revenge".
    3. Non-European societies have perceived the Earth as a non-living source of all resources.
    4. There is a direct cause–effect relationship between human activities and global climate change.
    Choice C
    Non-European societies have perceived the Earth as a non-living source of all resources.

The passage below is accompanied by four questions. Based on the passage, choose the best answer for each question.

Steven Pinker's new book, "Rationality: What It Is, Why It Seems Scarce, Why It Matters," offers a pragmatic dose of measured optimism, presenting rationality as a fragile but achievable ideal in personal and civic life. . . . Pinker's ambition to illuminate such a crucial topic offers the welcome prospect of a return to sanity. . . . It's no small achievement to make formal logic, game theory, statistics and Bayesian reasoning delightful topics full of charm and relevance.

It's also plausible to believe that a wider application of the rational tools he analyzes would improve the world in important ways. His primer on statistics and scientific uncertainty is particularly timely and should be required reading before consuming any news about the [COVID] pandemic. More broadly, he argues that less media coverage of shocking but vanishingly rare events, from shark attacks to adverse vaccine reactions, would help prevent dangerous overreactions, fatalism and the diversion of finite resources away from solvable but less-dramatic issues, like malnutrition in the developing world.

It's a reasonable critique, and Pinker is not the first to make it. But analyzing the political economy of journalism — its funding structures, ownership concentration and increasing reliance on social media shares — would have given a fuller picture of why so much coverage is so misguided and what we might do about it.

Pinker's main focus is the sort of conscious, sequential reasoning that can track the steps in a geometric proof or an argument in formal logic. Skill in this domain maps directly onto the navigation of many real-world problems, and Pinker shows how greater mastery of the tools of rationality can improve decision-making in medical, legal, financial and many other contexts in which we must act on uncertain and shifting information. . . .

Despite the undeniable power of the sort of rationality he describes, many of the deepest insights in the history of science, math, music and art strike their originators in moments of epiphany. From the 19th-century chemist Friedrich August Kekulé's discovery of the structure of benzene to any of Mozart's symphonies, much extraordinary human achievement is not a product of conscious, sequential reasoning. Even Plato's Socrates — who anticipated many of Pinker's points by nearly 2,500 years, showing the virtue of knowing what you do not know and examining all premises in arguments, not simply trusting speakers' authority or charisma — attributed many of his most profound insights to dreams and visions. Conscious reasoning is helpful in sorting the wheat from the chaff, but it would be interesting to consider the hidden aquifers that make much of the grain grow in the first place.

The role of moral and ethical education in promoting rational behavior is also underexplored. Pinker recognizes that rationality "is not just a cognitive virtue but a moral one." But this profoundly important point, one subtly explored by ancient Greek philosophers like Plato and Aristotle, doesn't really get developed. This is a shame, since possessing the right sort of moral character is arguably a precondition for using rationality in beneficial ways.

  1. CAT 2023 Slot 3 - VA RC

    The author refers to the ancient Greek philosophers to:

    1. indicate the various similarities between their thinking and Pinker's conclusions.
    2. reveal gaps in Pinker's discussion of the importance of ethical considerations in rational behaviour.
    3. show how dreams and visions have for centuries influenced subconscious behaviour and pathbreaking inventions.
    4. highlight the influence of their thinking on the development of Pinker's arguments.
    Choice B
    reveal gaps in Pinker's discussion of the importance of ethical considerations in rational behaviour.

  2. CAT 2023 Slot 3 - VA RC

    The author mentions Kekulé's discovery of the structure of benzene and Mozart's symphonies to illustrate the point that:

    1. great innovations across various fields can stem from flashes of intuition and are not always propelled by logical thinking.
    2. unlike the sciences, human achievements in other fields are a mix of logical reasoning and spontaneous epiphanies.
    3. Pinker's conclusions on sequential reasoning are belied by European achievements which, in the past, were more rooted in unconscious bursts of genius.
    4. it is not just the creative arts, but also scientific fields that have benefitted from flashes of creativity.
    Choice A
    great innovations across various fields can stem from flashes of intuition and are not always propelled by logical thinking.

  3. CAT 2023 Slot 3 - VA RC

    According to the author, for Pinker as well as the ancient Greek philosophers, rational thinking involves all of the following EXCEPT:

    1. the belief that the ability to reason logically encompasses an ethical and moral dimension.
    2. an awareness of underlying assumptions in an argument and gaps in one's own knowledge.
    3. the primacy of conscious sequential reasoning as the basis for seminal human achievements.
    4. arriving at independent conclusions irrespective of who is presenting the argument.
    Choice C
    the primacy of conscious sequential reasoning as the basis for seminal human achievements.

  4. CAT 2023 Slot 3 - VA RC

    The author endorses Pinker's views on the importance of logical reasoning as it:

    1. equips people with the ability to tackle challenging practical problems.
    2. focuses public attention on real issues like development rather than sensational events.
    3. provides a moral compass for resolving important ethical dilemmas.
    4. helps people to gain expertise in statistics and other scientific disciplines.
    Choice A
    equips people with the ability to tackle challenging practical problems.

The passage below is accompanied by four questions. Based on the passage, choose the best answer for each question.
In 2006, the Met [art museum in the US] agreed to return the Euphronios krater, a masterpiece Greek urn that had been a museum draw since 1972. In 2007, the Getty [art museum in the US] agreed to return 40 objects to Italy, including a marble Aphrodite, in the midst of looting scandals. And in December, Sotheby's and a private owner agreed to return an ancient Khmer statue of a warrior, pulled from auction two years before, to Cambodia.

Cultural property, or patrimony, laws limit the transfer of cultural property outside the source country's territory, including outright export prohibitions and national ownership laws. Most art historians, archaeologists, museum officials and policymakers portray cultural property laws in general as invaluable tools for counteracting the ugly legacy of Western cultural imperialism.

During the late 19th and early 20th century — an era former Met director Thomas Hoving called "the age of piracy" — American and European art museums acquired antiquities by hook or by crook, from grave robbers or souvenir collectors, bounty from digs and ancient sites in impoverished but art-rich source countries. Patrimony laws were intended to protect future archaeological discoveries against Western imperialist designs. . . .

I surveyed 90 countries with one or more archaeological sites on UNESCO's World Heritage Site list, and my study shows that in most cases the number of discovered sites diminishes sharply after a country passes a cultural property law. There are 222 archaeological sites listed for those 90 countries. When you look into the history of the sites, you see that all but 21 were discovered before the passage of cultural property laws. . . .

Strict cultural patrimony laws are popular in most countries. But the downside may be that they reduce incentives for foreign governments, nongovernmental organizations and educational institutions to invest in overseas exploration because their efforts will not necessarily be rewarded by opportunities to hold, display and study what is uncovered. To the extent that source countries can fund their own archaeological projects, artifacts and sites may still be discovered. . . . The survey has far-reaching implications. It suggests that source countries, particularly in the developing world, should narrow their cultural property laws so that they can reap the benefits of new archaeological discoveries, which typically increase tourism and enhance cultural pride. This does not mean these nations should abolish restrictions on foreign excavation and foreign claims to artifacts.

China provides an interesting alternative approach for source nations eager for foreign archaeological investment. From 1935 to 2003, China had a restrictive cultural property law that prohibited foreign ownership of Chinese cultural artifacts. In those years, China's most significant archaeological discovery occurred by chance, in 1974, when peasant farmers accidentally uncovered ranks of buried terra cotta warriors, which are part of Emperor Qin's spectacular tomb system.

In 2003, the Chinese government switched course, dropping its cultural property law and embracing collaborative international archaeological research. Since then, China has nominated 11 archaeological sites for inclusion in the World Heritage Site list, including eight in 2013, the most ever for China.

  1. CAT 2023 Slot 3 - VA RC

    From the passage we can infer that the author is likely to advise poor, but archaeologically-rich source countries to do all of the following, EXCEPT:

    1. to find ways to motivate other countries to finance archaeological explorations in their country.
    2. allow foreign countries to analyse and exhibit the archaeological finds made in the source country.
    3. adopt China's strategy of dropping its cultural property laws and carrying out archaeological research through international collaboration.
    4. fund institutes in other countries to undertake archaeological exploration in the source country reaping the benefits of cutting-edge techniques.
    Choice D
    fund institutes in other countries to undertake archaeological exploration in the source country reaping the benefits of cutting-edge techniques.

  2. CAT 2023 Slot 3 - VA RC

    It can be inferred from the passage that archaeological sites are considered important by some source countries because they:

    1. give a boost to the tourism sector.
    2. are a symbol of Western imperialism.
    3. generate funds for future discoveries.
    4. are subject to strict patrimony laws.
    Choice A
    give a boost to the tourism sector.

  3. CAT 2023 Slot 3 - VA RC

    Which one of the following statements, if true, would undermine the central idea of the passage?

    1. Affluent archaeologically-rich source countries can afford to carry out their own excavations.
    2. Museums established in economically deprived archaeologically-rich source countries can display the antiques discovered there.
    3. Western countries will have to apologise to countries for looting their cultural property in the past century.
    4. UNESCO finances archaeological research in poor, but archaeologically-rich source countries.
    Choice D
    UNESCO finances archaeological research in poor, but archaeologically-rich source countries.

  4. CAT 2023 Slot 3 - VA RC

    Which one of the following statements best expresses the paradox of patrimony laws?

    1. They were intended to protect cultural property, but instead resulted in the neglect of historical sites.
    2. They were aimed at protecting cultural property, but instead reduced business for auctioneers like Sotheby's.
    3. They were aimed at protecting cultural property, but instead reduced new archaeological discoveries.
    4. They were intended to protect cultural property, but instead resulted in the withholding of national treasure from museums.
    Choice C
    They were aimed at protecting cultural property, but instead reduced new archaeological discoveries.

The passage below is accompanied by four questions. Based on the passage, choose the best answer for each question.

Understanding romantic aesthetics is not a simple undertaking for reasons that are internal to the nature of the subject. Distinguished scholars, such as Arthur Lovejoy, Northrop Frye and Isaiah Berlin, have remarked on the notorious challenges facing any attempt to define romanticism. Lovejoy, for example, claimed that romanticism is "the scandal of literary history and criticism" . . . The main difficulty in studying the romantics, according to him, is the lack of any "single real entity, or type of entity" that the concept "romanticism" designates. Lovejoy concluded, "the word 'romantic' has come to mean so many things that, by itself, it means nothing" . . .

The more specific task of characterizing romantic aesthetics adds to these difficulties an air of paradox. Conventionally, "aesthetics" refers to a theory concerning beauty and art or the branch of philosophy that studies these topics. However, many of the romantics rejected the identification of aesthetics with a circumscribed domain of human life that is separated from the practical and theoretical domains of life. The most characteristic romantic commitment is to the idea that the character of art and beauty and of our engagement with them should shape all aspects of human life. Being fundamental to human existence, beauty and art should be a central ingredient not only in a philosophical or artistic life, but also in the lives of ordinary men and women. Another challenge for any attempt to characterize romantic aesthetics lies in the fact that most of the romantics were poets and artists whose views of art and beauty are, for the most part, to be found not in developed theoretical accounts, but in fragments, aphorisms and poems, which are often more elusive and suggestive than conclusive.

Nevertheless, in spite of these challenges the task of characterizing romantic aesthetics is neither impossible nor undesirable, as numerous thinkers responding to Lovejoy's radical skepticism have noted. While warning against a reductive definition of romanticism, Berlin, for example, still heralded the need for a general characterization: "[Although] one does have a certain sympathy with Lovejoy's despair…[he is] in this instance mistaken. There was a romantic movement…and it is important to discover what it is" . . .

Recent attempts to characterize romanticism and to stress its contemporary relevance follow this path. Instead of overlooking the undeniable differences between the variety of romanticisms of different nations that Lovejoy had stressed, such studies attempt to characterize romanticism, not in terms of a single definition, a specific time, or a specific place, but in terms of "particular philosophical questions and concerns" . . .

While the German, British and French romantics are all considered, the central protagonists in the following are the German romantics. Two reasons explain this focus: first, because it has paved the way for the other romanticisms, German romanticism has a pride of place among the different national romanticisms . . . Second, the aesthetic outlook that was developed in Germany roughly between 1796 and 1801–02 — the period that corresponds to the heyday of what is known as "Early Romanticism" . . .— offers the most philosophical expression of romanticism since it is grounded primarily in the epistemological, metaphysical, ethical, and political concerns that the German romantics discerned in the aftermath of Kant's philosophy.

  1. CAT 2023 Slot 3 - VA RC

    According to the romantics, aesthetics:

    1. is widely considered to be irrelevant to human existence.
    2. permeates all aspects of human life, philosophical and mundane.
    3. should be confined to a specific domain separate from the practical and theoretical aspects of life.
    4. is primarily the concern of philosophers and artists, rather than of ordinary people.
    Choice B
    permeates all aspects of human life, philosophical and mundane.

  2. CAT 2023 Slot 3 - VA RC

    Which one of the following statements is NOT supported by the passage?

    1. Recent studies on romanticism seek to refute the differences between national romanticisms.
    2. Romantic aesthetics are primarily expressed through fragments, aphorisms, and poems.
    3. Many romantics rejected the idea of aesthetics as a domain separate from other aspects of life.
    4. Characterising romantic aesthetics is both possible and desirable, despite the challenges involved.
    Choice A
    Recent studies on romanticism seek to refute the differences between national romanticisms.

  3. CAT 2023 Slot 3 - VA RC

    The main difficulty in studying romanticism is the:

    1. absence of written accounts by romantic poets and artists.
    2. elusive and suggestive nature of romantic aesthetics.
    3. controversial and scandalous history of romantic literature.
    4. lack of clear conceptual contours of the domain.
    Choice D
    lack of clear conceptual contours of the domain.

  4. CAT 2023 Slot 3 - VA RC

    According to the passage, recent studies on romanticism avoid "a single definition, a specific time, or a specific place" because they:

    1. prefer to highlight the paradox of romantic aesthetics as a concept.
    2. seek to discredit Lovejoy's scepticism regarding romanticism.
    3. understand that the variety of romanticisms renders a general analysis impossible.
    4. prefer to focus on the fundamental concerns of the romantics.
    Choice D
    prefer to focus on the fundamental concerns of the romantics.

CAT VARC : CAT 2022 Question Paper Slot 1

The passage below is accompanied by a set of questions. Choose the best answer to each question.

The Chinese have two different concepts of a copy. Fangzhipin . . . are imitations where the difference from the original is obvious. These are small models or copies that can be purchased in a museum shop, for example. The second concept for a copy is fuzhipin . . . They are exact reproductions of the original, which, for the Chinese, are of equal value to the original. It has absolutely no negative connotations. The discrepancy with regard to the understanding of what a copy is has often led to misunderstandings and arguments between China and Western museums. The Chinese often send copies abroad instead of originals, in the firm belief that they are not essentially different from the originals. The rejection that then comes from the Western museums is perceived by the Chinese as an insult. . . .

The Far Eastern notion of identity is also very confusing to the Western observer. The Ise Grand Shrine [in Japan] is 1,300 years old for the millions of Japanese people who go there on pilgrimage every year. But in reality this temple complex is completely rebuilt from scratch every 20 years. . . .

The cathedral of Freiburg Minster in southwest Germany is covered in scaffolding almost all year round. The sandstone from which it is built is a very soft, porous material that does not withstand natural erosion by rain and wind. After a while, it crumbles. As a result, the cathedral is continually being examined for damage, and eroded stones are replaced. And in the cathedral's dedicated workshop, copies of the damaged sandstone figures are constantly being produced. Of course, attempts are made to preserve the stones from the Middle Ages for as long as possible. But at some point they, too, are removed and replaced with new stones.

Fundamentally, this is the same operation as with the Japanese shrine, except in this case the production of a replica takes place very slowly and over long periods of time. . . . In the field of art as well, the idea of an unassailable original developed historically in the Western world. Back in the 17th century [in the West], excavated artworks from antiquity were treated quite differently from today. They were not restored in a way that was faithful to the original. Instead, there was massive intervention in these works, changing their appearance. . . .

It is probably this intellectual position that explains why Asians have far fewer scruples about cloning than Europeans. The South Korean cloning researcher Hwang Woo-suk, who attracted worldwide attention with his cloning experiments in 2004, is a Buddhist. He found a great deal of support and followers among Buddhists, while Christians called for a ban on human cloning. . . . Hwang legitimised his cloning experiments with his religious affiliation: 'I am Buddhist, and I have no philosophical problem with cloning. And as you know, the basis of Buddhism is that life is recycled through reincarnation. In some ways, I think, therapeutic cloning restarts the circle of life.'

  1. CAT 2022 Slot 1 - VA RC

    Based on the passage, which one of the following copies would a Chinese museum be unlikely to consider as having less value than the original?

    1. Pablo Picasso's painting of Vincent van Gogh's original painting, identical in every respect.
    2. Pablo Picasso's photograph of Vincent van Gogh's original painting, printed to exactly the same scale.
    3. Pablo Picasso's miniaturised, but otherwise faithful and accurate painting of Vincent van Gogh's original painting.
    4. Pablo Picasso's painting of Vincent van Gogh's original painting, bearing Picasso's signature.
    Choice A
    Pablo Picasso's painting of Vincent van Gogh's original painting, identical in every respect.

  2. CAT 2022 Slot 1 - VA RC

    Which one of the following statements does not correctly express the similarity between the Ise Grand Shrine and the cathedral of Freiburg Minster?

    1. Both can be regarded as very old structures.
    2. Both will one day be completely rebuilt.
    3. Both were built as places of worship.
    4. Both are continually undergoing restoration.
    Choice D
    Both are continually undergoing restoration.

  3. CAT 2022 Slot 1 - VA RC

    Which one of the following scenarios is unlikely to follow from the arguments in the passage?

    1. A 17th century British painter would have no problem adding personal touches when restoring an ancient Roman painting.
    2. A 20th century Japanese Buddhist monk would value a reconstructed shrine as the original.
    3. A 17th century French artist who adhered to a Christian worldview would need to be completely true to the original intent of a painting when restoring it.
    4. A 21st century Christian scientist is likely to oppose cloning because of his philosophical orientation.
    Choice C
    A 17th century French artist who adhered to a Christian worldview would need to be completely true to the original intent of a painting when restoring it.

  4. CAT 2022 Slot 1 - VA RC

    The value that the modern West assigns to "an unassailable original" has resulted in all of the following EXCEPT:

    1. it discourages them from making interventions in ancient art.
    2. it discourages them from carrying out human cloning.
    3. it discourages them from simultaneous displays of multiple copies of a painting.
    4. it allows regular employment for certain craftsmen.
    Choice B
    it discourages them from carrying out human cloning.

The passage below is accompanied by a set of questions. Choose the best answer to each question.

Stoicism was founded in 300 BC by the Greek philosopher Zeno and survived into the Roman era until about AD 300. According to the Stoics, emotions consist of two movements. The first movement is the immediate feeling and other reactions (e.g., physiological response) that occur when a stimulus or event occurs. For instance, consider what could have happened if an army general accused Marcus Aurelius of treason in front of other officers. The first movement for Marcus may have been (internal) surprise and anger in response to this insult, accompanied perhaps by some involuntary physiological and expressive responses such as face flushing and a movement of the eyebrows. The second movement is what one does next about the emotion. Second movement behaviors occur after thinking and are under one's control. Examples of second movements for Marcus might have included a plot to seek revenge, actions signifying deference and appeasement, or perhaps proceeding as he would have proceeded whether or not this event occurred: continuing to lead the Romans in a way that Marcus Aurelius believed best benefited them. In the Stoic view, choosing a reasoned, unemotional response as the second movement is the only appropriate response.

The Stoics believed that to live the good life and be a good person, we need to free ourselves of nearly all desires such as too much desire for money, power, or sexual gratification. Prior to second movements, we can consider what is important in life. Money, power, and excessive sexual gratification are not important. Character, rationality, and kindness are important. The Epicureans, first associated with the Greek philosopher Epicurus . . . held a similar view, believing that people should enjoy simple pleasures, such as good conversation, friendship, food, and wine, but not be indulgent in these pursuits and not follow passion for those things that hold no real value like power and money. As Oatley (2004) states, "the Epicureans articulated a view—enjoyment of relationship with friends, of things that are real rather than illusory, simple rather than artificially inflated, possible rather than vanishingly unlikely—that is certainly relevant today" . . . In sum, these ancient Greek and Roman philosophers saw emotions, especially strong ones, as potentially dangerous. They viewed emotions as experiences that needed to be [reined] in and controlled.

As Oatley (2004) points out, the Stoic idea bears some similarity to Buddhism. Buddha, living in India in the 6th century BC, argued for cultivating a certain attitude that decreases the probability of (in Stoic terms) destructive second movements. Through meditation and the right attitude, one allows emotions to happen to oneself (it is impossible to prevent this), but one is advised to observe the emotions without necessarily acting on them; one achieves some distance and decides what has value and what does not have value. Additionally, the Stoic idea of developing virtue in oneself, of becoming a good person, which the Stoics believed we could do because we have a touch of the divine, laid the foundation for the three monotheistic religions: Judaism, Christianity, and Islam . . . As with Stoicism, tenets of these religions include controlling our emotions lest we engage in sinful behavior.

  1. CAT 2022 Slot 1 - VA RC

    On the basis of the passage, which one of the following statements can be regarded as true?

    1. The Epicureans believed in controlling all emotions.
    2. There were no Stoics in India at the time of the Roman civilisation.
    3. The Stoic influences can be seen in multiple religions.
    4. The Stoics valorised the pursuit of money, power, and sexual gratification.
    Choice C
    The Stoic influences can be seen in multiple religions.

  2. CAT 2022 Slot 1 - VA RC

    Which one of the following statements, if false, could be seen as contradicting the facts/arguments in the passage?

    1. In the Epicurean view, indulging in simple pleasures is not desirable.
    2. Despite practising meditation and cultivating the right attitude, emotions cannot ever be controlled.
    3. In the Stoic view, choosing a reasoned, unemotional response as the first movement is an appropriate response to emotional situations.
    4. The Greek philosopher Zeno survived into the Roman era until about AD 300.
    Choice A
    In the Epicurean view, indulging in simple pleasures is not desirable.

  3. CAT 2022 Slot 1 - VA RC

    "Through meditation and the right attitude, one allows emotions to happen to oneself (it is impossible to prevent this), but one is advised to observe the emotions without necessarily acting on them; one achieves some distance and decides what has value and what does not have value." In the context of the passage, which one of the following is not a possible implication of the quoted statement?

    1. Meditation allows certain out-of-body experiences that permit us to gain the distance necessary to control our emotions.
    2. Emotional responses can make it difficult to distinguish valuable experiences from valueless experiences.
    3. The observation of emotions in a distant manner corresponds to the second movement referred to earlier in the passage.
    4. "Meditation and the right attitude", in this instance, implies an initially passive reception of all experiences.
    Choice A
    Meditation allows certain out-of-body experiences that permit us to gain the distance necessary to control our emotions.

  4. CAT 2022 Slot 1 - VA RC

    Which one of the following statements would be an accurate inference from the example of Marcus Aurelius?

    1. Marcus Aurelius was humiliated by the accusation of treason in front of the other officers.
    2. Marcus Aurelius was one of the leaders of the Roman army.
    3. Marcus Aurelius was a Stoic whose philosophy survived into the Roman era.
    4. Marcus Aurelius plotted revenge in his quest for justice.
    Choice B
    Marcus Aurelius was one of the leaders of the Roman army.

The passage below is accompanied by a set of questions. Choose the best answer to each question.

Critical theory of technology is a political theory of modernity with a normative dimension. It belongs to a tradition extending from Marx to Foucault and Habermas according to which advances in the formal claims of human rights take center stage while in the background centralization of ever more powerful public institutions and private organizations imposes an authoritarian social order.

Marx attributed this trajectory to the capitalist rationalization of production. Today it marks many institutions besides the factory and every modern political system, including so-called socialist systems. This trajectory arose from the problems of command over a disempowered and deskilled labor force; but everywhere [that] masses are organized – whether it be Foucault's prisons or Habermas's public sphere – the same pattern prevails. Technological design and development is shaped by this pattern as the material base of a distinctive social order. Marcuse would later point to a "project" as the basis of what he called rather confusingly "technological rationality." Releasing technology from this project is a democratic political task.

In accordance with this general line of thought, critical theory of technology regards technologies as an environment rather than as a collection of tools. We live today with and even within technologies that determine our way of life. Along with the constant pressures to build centers of power, many other social values and meanings are inscribed in technological design. A hermeneutics of technology must make explicit the meanings implicit in the devices we use and the rituals they script. Social histories of technologies such as the bicycle, artificial lighting or firearms have made important contributions to this type of analysis. Critical theory of technology attempts to build a methodological approach on the lessons of these histories.

As an environment, technologies shape their inhabitants. In this respect, they are comparable to laws and customs. Each of these institutions can be said to represent those who live under their sway through privileging certain dimensions of their human nature. Laws of property represent the interest in ownership and control. Customs such as parental authority represent the interest of childhood in safety and growth. Similarly, the automobile represents its users in so far as they are interested in mobility. Interests such as these constitute the version of human nature sanctioned by society.

This notion of representation does not imply an eternal human nature. The concept of nature as non-identity in the Frankfurt School suggests an alternative. On these terms, nature is what lies at the limit of history, at the point at which society loses the capacity to imprint its meanings on things and control them effectively. The reference here is, of course, not to the nature of natural science, but to the lived nature in which we find ourselves and which we are. This nature reveals itself as that which cannot be totally encompassed by the machinery of society. For the Frankfurt School, human nature, in all its transcending force, emerges out of a historical context as that context is [depicted] in illicit joys, struggles and pathologies. We can perhaps admit a less romantic . . . conception in which those dimensions of human nature recognized by society are also granted theoretical legitimacy.

  1. CAT 2022 Slot 1 - VA RC

    All of the following claims can be inferred from the passage, EXCEPT:

    1. technologies seek to privilege certain dimensions of human nature at a high cost to lived nature.
    2. the significance of parental authority to children's safety does not therefore imply that parental authority is a permanent aspect of human nature.
    3. analyses of technologies must engage with their social histories to be able to reveal their implicit and explicit meanings for us.
    4. the critical theory of technology argues that, as issues of human rights become more prominent, we lose sight of the ways in which the social order becomes more authoritarian.
    Choice A
    technologies seek to privilege certain dimensions of human nature at a high cost to lived nature.

  2. CAT 2022 Slot 1 - VA RC

    Which one of the following statements best reflects the main argument of the fourth paragraph of the passage?

    1. Technological environments privilege certain dimensions of human nature as effectively as laws and customs.
    2. Technology, laws, and customs are not unlike each other if considered as institutions.
    3. Technology, laws, and customs are comparable, but dissimilar phenomena.
    4. Automobiles represent the interest in mobility present in human nature.
    Choice B
    Technology, laws, and customs are not unlike each other if considered as institutions.

  3. CAT 2022 Slot 1 - VA RC

    Which one of the following statements contradicts the arguments of the passage?

    1. Marx's understanding of the capitalist rationalisation of production and Marcuse's understanding of a "project" of "technological rationality" share theoretical inclinations.
    2. Masses are organised in patterns set by Foucault's prisons and Habermas' public sphere.
    3. Paradoxically, the capitalist rationalisation of production is a mark of so-called socialist systems as well.
    4. The problems of command over a disempowered and deskilled labour force gave rise to similar patterns of the capitalist rationalisation of production wherever masses were organised.
    Choice B
    Masses are organised in patterns set by Foucault's prisons and Habermas' public sphere.

  4. CAT 2022 Slot 1 - VA RC

    Which one of the following statements could be inferred as supporting the arguments of the passage?

    1. The romantic conception of nature referred to by the passage is the one that requires theoretical legitimacy.
    2. Technologies form the environmental context and shape the contours of human society.
    3. It is not human nature, but human culture that is represented by institutions such as law and custom.
    4. Nature decides the point at which society loses its capacity to control history.
    Choice B
    Technologies form the environmental context and shape the contours of human society.

The passage below is accompanied by a set of questions. Choose the best answer to each question.

Stories concerning the Undead have always been with us. From out of the primal darkness of Mankind's earliest years, come whispers of eerie creatures, not quite alive (or alive in a way which we can understand), yet not quite dead either. These may have been ancient and primitive deities who dwelt deep in the surrounding forests and in remote places, or simply those deceased who refused to remain in their tombs and who wandered about the countryside, physically tormenting and frightening those who were still alive. Mostly they were ill-defined—strange sounds in the night beyond the comforting glow of the fire, or a shape, half-glimpsed in the twilight along the edge of an encampment. They were vague and indistinct, but they were always there with the power to terrify and disturb. They had the power to touch the minds of our early ancestors and to fill them with dread. Such fear formed the basis of the earliest tales although the source and exact nature of such terrors still remained very vague.

And as Mankind became more sophisticated, leaving the gloom of their caves and forming themselves into recognizable communities—towns, cities, whole cultures—so the Undead travelled with them, inhabiting their folklore just as they had in former times. Now they began to take on more definite shapes. They became walking cadavers; the physical embodiment of former deities and things which had existed alongside Man since the Creation. Some still remained vague and ill-defined but, as Mankind strove to explain the horror which it felt towards them, such creatures emerged more readily into the light.

In order to confirm their abnormal status, many of the Undead were often accorded attributes, which defied the natural order of things—the power to transform themselves into other shapes, the ability to sustain themselves by drinking human blood, and the ability to influence human minds across a distance. Such powers—described as supernatural—only [lent] an added dimension to the terror that humans felt regarding them.

And it was only natural, too, that the Undead should become connected with the practice of magic. From very early times, Shamans and witchdoctors had claimed at least some power and control over the spirits of departed ancestors, and this has continued down into more "civilized" times. Formerly, the invisible spirits and forces that thronged around men's earliest encampments, had spoken "through" the tribal Shamans but now, as entities in their own right, they were subject to magical control and could be physically summoned by a competent sorcerer. However, the relationship between the magician and an Undead creature was often a very tenuous and uncertain one. Some sorcerers might have even become Undead entities once they died, but they might also have been susceptible to the powers of other magicians when they did.

From the Middle Ages and into the Age of Enlightenment, theories of the Undead continued to grow and develop. Their names became more familiar—werewolf, vampire, ghoul—each one certain to strike fear into the hearts of ordinary humans.

  1. CAT 2022 Slot 1 - VA RC

    Which one of the following statements best describes what the passage is about?

    1. The passage discusses the evolution of theories of the Undead from primitive thinking to the Age of Enlightenment.
    2. The passage describes the failure of human beings to fully comprehend their environment.
    3. The writer discusses the transition from primitive thinking to the Age of Enlightenment.
    4. The writer describes the ways in which the Undead come to be associated with Shamans and the practice of magic.
    Choice A
    The passage discusses the evolution of theories of the Undead from primitive thinking to the Age of Enlightenment.
    >

  2. CAT 2022 Slot 1 - VA RC

    "In order to confirm their abnormal status, many of the Undead were often accorded attributes, which defied the natural order of things . . ." Which one of the following best expresses the claim made in this statement?

    1. The Undead are deified in nature's order by giving them divine attributes.
    2. According the Undead an abnormal status is to reject the natural order of things.
    3. Human beings conceptualise the Undead as possessing abnormal features.
    4. The natural attributes of the Undead are rendered abnormal by changing their status.
    Choice C
    Human beings conceptualise the Undead as possessing abnormal features.

  3. CAT 2022 Slot 1 - VA RC

    All of the following statements, if false, could be seen as being in accordance with the passage, EXCEPT:

    1. the Undead remained vague and ill-defined, even as Mankind strove to understand the horror they inspired.
    2. the relationship between Shamans and the Undead was believed to be a strong and stable one.
    3. the growing sophistication of Mankind meant that humans stopped believing in the Undead.
    4. the transition from the Middle Ages to the Age of Enlightenment saw new theories of the Undead.
    Choice D
    the transition from the Middle Ages to the Age of Enlightenment saw new theories of the Undead.

  4. CAT 2022 Slot 1 - VA RC

    Which one of the following observations is a valid conclusion to draw from the statement, "From out of the primal darkness of Mankind's earliest years, come whispers of eerie creatures, not quite alive (or alive in a way which we can understand), yet not quite dead either."?

    1. Mankind's primal years were marked by creatures alive with eerie whispers, but seen only in the darkness.
    2. Mankind's early years were marked by a belief in the existence of eerie creatures that were neither quite alive nor dead.
    3. We can understand the lives of the eerie creatures in Mankind's early years through their whispers in the darkness.
    4. Long ago, eerie creatures used to whisper in the primal darkness that they were not quite dead.
    Choice B
    Mankind's early years were marked by a belief in the existence of eerie creatures that were neither quite alive nor dead.

CAT VARC : CAT 2022 Question Paper Slot 2

The passage below is accompanied by a set of questions. Choose the best answer to each question.

[Octopuses are] misfits in their own extended families . . . They belong to the Mollusca class Cephalopoda. But they don't look like their cousins at all. Other molluscs include sea snails, sea slugs, bivalves - most are shelled invertebrates with a dorsal foot. Cephalopods are all arms, and can be as tiny as 1 centimetre and as large at 30 feet. Some of them have brains the size of a walnut, which is large for an invertebrate. . . .

It makes sense for these molluscs to have added protection in the form of a higher cognition; they don't have a shell covering them, and pretty much everything feeds on cephalopods, including humans. But how did cephalopods manage to secure their own invisibility cloak? Cephalopods fire from multiple cylinders to achieve this in varying degrees from species to species. There are four main catalysts - chromatophores, iridophores, papillae and leucophores. . . .

[Chromatophores] are organs on their bodies that contain pigment sacs, which have red, yellow and brown pigment granules. These sacs have a network of radial muscles, meaning muscles arranged in a circle radiating outwards. These are connected to the brain by a nerve. When the cephalopod wants to change colour, the brain carries an electrical impulse through the nerve to the muscles that expand outwards, pulling open the sacs to display the colours on the skin. Why these three colours? Because these are the colours the light reflects at the depths they live in (the rest is absorbed before it reaches those depths). . . .

Well, what about other colours? Cue the iridophores. Think of a second level of skin that has thin stacks of cells. These can reflect light back at different wavelengths. . . . It's using the same properties that we've seen in hologram stickers, or rainbows on puddles of oil. You move your head and you see a different colour. The sticker isn't doing anything but reflecting light - it's your movement that's changing the appearance of the colour. This property of holograms, oil and other such surfaces is called "iridescence". . . .

Papillae are sections of the skin that can be deformed to make a texture bumpy. Even humans possess them (goosebumps) but cannot use them in the manner that cephalopods can. For instance, the use of these cells is how an octopus can wrap itself over a rock and appear jagged or how a squid or cuttlefish can imitate the look of a coral reef by growing miniature towers on its skin. It actually matches the texture of the substrate it chooses.

Finally, the leucophores: According to a paper, published in Nature, cuttlefish and octopuses possess an additional type of reflector cell called a leucophore. They are cells that scatter full spectrum light so that they appear white in a similar way that a polar bear's fur appears white. Leucophores will also reflect any filtered light shown on them . . . If the water appears blue at a certain depth, the octopuses and cuttlefish can appear blue; if the water appears green, they appear green, and so on and so forth.

  1. CAT 2022 Slot 2 - VA RC

    All of the following are reasons for octopuses being "misfits" EXCEPT that they:

    1. have several arms.
    2. are consumed by humans and other animals.
    3. do not possess an outer protective shell.
    4. exhibit higher intelligence than other molluscs.
    Choice B
    are consumed by humans and other animals.

  2. CAT 2022 Slot 2 - VA RC

    Based on the passage, we can infer that all of the following statements, if true, would weaken the camouflaging adeptness of Cephalopods EXCEPT:

    1. the hydrostatic pressure at the depths at which Cephalopods reside renders radial muscle movements difficult.
    2. the temperature of water at the depths at which Cephalopods reside renders the transmission of neural signals difficult.
    3. light reflects the colours red, green, and yellow at the depths at which Cephalopods reside.
    4. the number of chromatophores in Cephalopods is half the number of iridophores and leucophores.
    Choice D
    the number of chromatophores in Cephalopods is half the number of iridophores and leucophores.

  3. CAT 2022 Slot 2 - VA RC

    Based on the passage, it can be inferred that camouflaging techniques in an octopus are most dissimilar to those in:

    1. polar bears
    2. sea snails
    3. squids
    4. cuttlefish
    Choice B
    sea snails

  4. CAT 2022 Slot 2 - VA RC

    Which one of the following statements is not true about the camouflaging ability of Cephalopods?

    1. Cephalopods can take on the colour of their predator.
    2. Cephalopods can blend into the colour of their surroundings.
    3. Cephalopods can change their colour.
    4. Cephalopods can change their texture.
    Choice A
    Cephalopods can take on the colour of their predator.

The passage below is accompanied by a set of questions. Choose the best answer to each question.

We begin with the emergence of the philosophy of the social sciences as an arena of thought and as a set of social institutions. The two characterisations overlap but are not congruent. Academic disciplines are social institutions. . . . My view is that institutions are all those social entities that organise action: they link acting individuals into social structures. There are various kinds of institutions. Hegelians and Marxists emphasise universal institutions such as the family, rituals, governance, economy and the military. These are mostly institutions that just grew. Perhaps in some imaginary beginning of time they spontaneously appeared. In their present incarnations, however, they are very much the product of conscious attempts to mould and plan them. We have family law, established and disestablished churches, constitutions and laws, including those governing the economy and the military. Institutions deriving from statute, like joint-stock companies are formal by contrast with informal ones such as friendships. There are some institutions that come in both informal and formal variants, as well as in mixed ones. Consider the fact that the stock exchange and the black market are both market institutions, one formal one not. Consider further that there are many features of the work of the stock exchange that rely on informal, noncodifiable agreements, not least the language used for communication. To be precise, mixtures are the norm . . . From constitutions at the top to by-laws near the bottom we are always adding to, or tinkering with, earlier institutions, the grown and the designed are intertwined.

It is usual in social thought to treat culture and tradition as different from, although alongside, institutions. The view taken here is different. Culture and tradition are sub-sets of institutions analytically isolated for explanatory or expository purposes. Some social scientists have taken all institutions, even purely local ones, to be entities that satisfy basic human needs - under local conditions . . . Others differed and declared any structure of reciprocal roles and norms an institution. Most of these differences are differences of emphasis rather than disagreements. Let us straddle all these versions and present institutions very generally . . . as structures that serve to coordinate the actions of individuals. . . . Institutions themselves then have no aims or purpose other than those given to them by actors or used by actors to explain them . . .

Language is the formative institution for social life and for science . . . Both formal and informal language is involved, naturally grown or designed. (Language is all of these to varying degrees.) Languages are paradigms of institutions or, from another perspective, nested sets of institutions. Syntax, semantics, lexicon and alphabet/character-set are all institutions within the larger institutional framework of a written language. Natural languages are typical examples of what Ferguson called 'the result of human action, but not the execution of any human design'[;] reformed natural languages and artificial languages introduce design into their modifications or refinements of natural language. Above all, languages are paradigms of institutional tools that function to coordinate.

  1. CAT 2022 Slot 2 - VA RC

    "Consider the fact that the stock exchange and the black market are both market institutions, one formal one not." Which one of the following statements best explains this quote, in the context of the passage?

    1. The stock exchange and the black market are both organised to function by rules.
    2. Market instruments can be formally traded in the stock exchange and informally traded in the black market.
    3. The stock exchange and the black market are both dependent on the market to survive.
    4. The stock exchange and the black market are examples of how, even within the same domain, different kinds of institutions can co-exist.
    Choice D
    The stock exchange and the black market are examples of how, even within the same domain, different kinds of institutions can co-exist.

  2. CAT 2022 Slot 2 - VA RC

    All of the following inferences from the passage are false, EXCEPT:

    1. "natural language" refers to that stage of language development where no conscious human intent is evident in the formation of language.
    2. the institution of friendship cannot be found in the institution of joint-stock companies because the first is an informal institution, while the second is a formal one.
    3. as concepts, "culture" and "tradition" have no analytical, explanatory or expository power, especially when they are treated in isolation.
    4. institutions like the family, rituals, governance, economy, and the military are natural and cannot be consciously modified.
    Choice A
    "natural language" refers to that stage of language development where no conscious human intent is evident in the formation of language.

  3. CAT 2022 Slot 2 - VA RC

    Which of the following statements best represents the essence of the passage?

    1. Institutions are structures that serve to coordinate the actions of individuals.
    2. It is usual in social thought to treat culture and tradition as different from institutions.
    3. Language is the fundamental formal institution for social life and for science.
    4. The stock exchange and the black market are both market institutions.
    Choice A
    Institutions are structures that serve to coordinate the actions of individuals.

  4. CAT 2022 Slot 2 - VA RC

    In the first paragraph of the passage, what are the two "characterisations" that are seen as overlapping but not congruent?

    1. "the philosophy of the social sciences" and "a set of social institutions".
    2. "academic disciplines" and "institutions".
    3. "an arena of thought" and "academic disciplines".
    4. "individuals" and "social structures".
    Choice B
    "academic disciplines" and "institutions".

The passage below is accompanied by a set of questions. Choose the best answer to each question.

When we teach engineering problems now, we ask students to come to a single "best" solution defined by technical ideals like low cost, speed to build, and ability to scale. This way of teaching primes students to believe that their decision-making is purely objective, as it is grounded in math and science. This is known as technical-social dualism, the idea that the technical and social dimensions of engineering problems are readily separable and remain distinct throughout the problem-definition and solution process.

Nontechnical parameters such as access to a technology, cultural relevancy or potential harms are deemed political and invalid in this way of learning. But those technical ideals are at their core social and political choices determined by a dominant culture focused on economic growth for the most privileged segments of society. By choosing to downplay public welfare as a critical parameter for engineering design, we risk creating a culture of disengagement from societal concerns amongst engineers that is antithetical to the ethical code of engineering.

In my field of medical devices, ignoring social dimensions has real consequences. . . . Most FDA-approved drugs are incorrectly dosed for people assigned female at birth, leading to unexpected adverse reactions. This is because they have been inadequately represented in clinical trials.

Beyond physical failings, subjective beliefs treated as facts by those in decision-making roles can encode social inequities. For example, spirometers, routinely used devices that measure lung capacity, still have correction factors that automatically assume smaller lung capacity in Black and Asian individuals. These racially based adjustments are derived from research done by eugenicists who thought these racial differences were biologically determined and who considered nonwhite people as inferior. These machines ignore the influence of social and environmental factors on lung capacity.

Many technologies for systemically marginalized people have not been built because they were not deemed important such as better early diagnostics and treatment for diseases like endometriosis, a disease that afflicts 10 percent of people with uteruses. And we hardly question whether devices are built sustainably, which has led to a crisis of medical waste and health care accounting for 10 percent of U.S. greenhouse gas emissions.

Social justice must be made core to the way engineers are trained. Some universities are working on this. . . . Engineers taught this way will be prepared to think critically about what problems we choose to solve, how we do so responsibly and how we build teams that challenge our ways of thinking.

Individual engineering professors are also working to embed societal needs in their pedagogy. Darshan Karwat at the University of Arizona developed activist engineering to challenge engineers to acknowledge their full moral and social responsibility through practical self-reflection. Khalid Kadir at the University of California, Berkeley, created the popular course Engineering, Environment, and Society that teaches engineers how to engage in place-based knowledge, an understanding of the people, context and history, to design better technical approaches in collaboration with communities. When we design and build with equity and justice in mind, we craft better solutions that respond to the complexities of entrenched systemic problems.

  1. CAT 2022 Slot 2 - VA RC

    In this passage, the author is making the claim that:

    1. technical-social dualism has emerged as a technique for engineering students to incorporate social considerations into their technical problem-solving processes.
    2. the objective of best solutions in engineering has shifted the focus of pedagogy from humanism and social obligations to technological perfection.
    3. engineering students today are trained to be non-subjective in their reasoning as this best enables them to develop much-needed universal solutions.
    4. engineering students today are taught to focus on objective technical outcomes, independent of the social dimensions of their work.
    Choice D
    engineering students today are taught to focus on objective technical outcomes, independent of the social dimensions of their work.

  2. CAT 2022 Slot 2 - VA RC

    The author gives all of the following reasons for why marginalised people are systematically discriminated against in technology-related interventions EXCEPT:

    1. "And we hardly question whether devices are built sustainably, which has led to a crisis of medical waste and health care accounting for 10 percent of U.S. greenhouse gas emissions."
    2. "These racially based adjustments are derived from research done by eugenicists who thought these racial differences were biologically determined and who considered nonwhite people as inferior."
    3. "Beyond physical failings, subjective beliefs treated as facts by those in decision-making roles can encode social inequities."
    4. "But those technical ideals are at their core social and political choices determined by a dominant culture focused on economic growth for the most privileged segments of society."
    Choice A
    "And we hardly question whether devices are built sustainably, which has led to a crisis of medical waste and health care accounting for 10 percent of U.S. greenhouse gas emissions."

  3. CAT 2022 Slot 2 - VA RC

    All of the following are examples of the negative outcomes of focusing on technical ideals in the medical sphere EXCEPT the:

    1. incorrect assignment of people as female at birth which has resulted in faulty drug interventions.
    2. exclusion of non-privileged groups in clinical trials which leads to incorrect drug dosages.
    3. neglect of research and development of medical technologies for the diagnosis and treatment of diseases that typically afflict marginalised communities.
    4. continuing calibration of medical devices based on past racial biases that have remained unadjusted for changes.
    Choice A
    incorrect assignment of people as female at birth which has resulted in faulty drug interventions.

  4. CAT 2022 Slot 2 - VA RC

    We can infer that the author would approve of a more evolved engineering pedagogy that includes all of the following EXCEPT:

    1. moving towards technical-social dualism where social community needs are incorporated in problem-definition and solutions.
    2. making considerations of environmental sustainability intrinsic to the development of technological solutions.
    3. design that is based on the needs of communities using local knowledge and responding to local priorities.
    4. a more responsible approach to technical design and problem-solving than a focus on speed in developing and bringing to scale.
    Choice A
    moving towards technical-social dualism where social community needs are incorporated in problem-definition and solutions.

The passage below is accompanied by a set of questions. Choose the best answer to each question.

Humans today make music. Think beyond all the qualifications that might trail after this bald statement: that only certain humans make music, that extensive training is involved, that many societies distinguish musical specialists from nonmusicians, that in today's societies most listen to music rather than making it, and so forth. These qualifications, whatever their local merit, are moot in the face of the overarching truth that making music, considered from a cognitive and psychological vantage, is the province of all those who perceive and experience what is made. We are, almost all of us, musicians - everyone who can entrain (not necessarily dance) to a beat, who can recognize a repeated tune (not necessarily sing it), who can distinguish one instrument or one singing voice from another. I will often use an antique word, recently revived, to name this broader musical experience. Humans are musicking creatures. . . .

The set of capacities that enables musicking is a principal marker of modern humanity. There is nothing polemical in this assertion except a certain insistence, which will figure often in what follows, that musicking be included in our thinking about fundamental human commonalities. Capacities involved in musicking are many and take shape in complicated ways, arising from innate dispositions . . . Most of these capacities overlap with nonmusical ones, though a few may be distinct and dedicated to musical perception and production. In the area of overlap, linguistic capacities seem to be particularly important, and humans are (in principle) language-makers in addition to music-makers - speaking creatures as well as musicking ones.

Humans are symbol-makers too, a feature tightly bound up with language, not so tightly with music. The species Cassirer dubbed Homo symbolicus cannot help but tangle musicking in webs of symbolic thought and expression, habitually making it a component of behavioral complexes that form such expression. But in fundamental features musicking is neither language-like nor symbol-like, and from these differences come many clues to its ancient emergence.

If musicking is a primary, shared trait of modern humans, then to describe its emergence must be to detail the coalescing of that modernity. This took place, archaeologists are clear, over a very long durée: at least 50,000 years or so, more likely something closer to 200,000, depending in part on what that coalescence is taken to comprise. If we look back 20,000 years, a small portion of this long period, we reach the lives of humans whose musical capacities were probably little different from our own. As we look farther back we reach horizons where this similarity can no longer hold - perhaps 40,000 years ago, perhaps 70,000, perhaps 100,000. But we never cross a line before which all the cognitive capacities recruited in modern musicking abruptly disappear. Unless we embrace the incredible notion that music sprang forth in full-blown glory, its emergence will have to be tracked in gradualist terms across a long period.

This is one general feature of a history of music's emergence . . . The history was at once sociocultural and biological . . . The capacities recruited in musicking are many, so describing its emergence involves following several or many separate strands.

  1. CAT 2022 Slot 2 - VA RC

    Which one of the following sets of terms best serves as keywords to the passage?

    1. Musicking; Cognitive psychology; Antique; Symbol-makers; Modernity.
    2. Humans; Psychological vantage; Musicking; Cassirer; Emergence of music.
    3. Humans; Capacities; Language; Symbols; Modernity.
    4. Humans; Musicking; Linguistic capacities; Symbol-making; Modern humanity.
    Choice D
    Humans; Musicking; Linguistic capacities; Symbol-making; Modern humanity.

  2. CAT 2022 Slot 2 - VA RC

    Based on the passage, which one of the following statements is a valid argument about the emergence of music/musicking?

    1. Anyone who can perceive and experience music must be considered capable of musicking.
    2. Although musicking is not language-like, it shares the quality of being a form of expression.
    3. 20,000 years ago, human musical capacities were not very different from what they are today.
    4. All musical work is located in the overlap between linguistic capacity and music production.
    Choice C
    20,000 years ago, human musical capacities were not very different from what they are today.

  3. CAT 2022 Slot 2 - VA RC

    "Think beyond all the qualifications that might trail after this bald statement . . ." In the context of the passage, what is the author trying to communicate in this quoted extract?

    1. A bald statement is one that is trailed by a series of qualifying clarifications and caveats.
    2. Although there may be many caveats and other considerations, the statement is essentially true.
    3. Thinking beyond qualifications allows us to give free reign to musical expressions.
    4. A bald statement is one that requires no qualifications to infer its meaning.
    Choice B
    Although there may be many caveats and other considerations, the statement is essentially true.

  4. CAT 2022 Slot 2 - VA RC

    Which one of the following statements, if true, would weaken the author's claim that humans are musicking creatures?

    1. Musical capacities are primarily socio-cultural, which explains the wide diversity of musical forms.
    2. Nonmusical capacities are of far greater consequence to human survival than the capacity for music.
    3. As musicking is neither language-like nor symbol-like, it is a much older form of expression.
    4. From a cognitive and psychological vantage, musicking arises from unconscious dispositions, not conscious ones.
    Choice A
    Musical capacities are primarily socio-cultural, which explains the wide diversity of musical forms.

CAT VARC : CAT 2022 Question Paper Slot 3

The passage below is accompanied by a set of questions. Choose the best answer to each question.

As software improves, the people using it become less likely to sharpen their own know-how. Applications that offer lots of prompts and tips are often to blame; simpler, less solicitous programs push people harder to think, act and learn.

Ten years ago, information scientists at Utrecht University in the Netherlands had a group of people carry out complicated analytical and planning tasks using either rudimentary software that provided no assistance or sophisticated software that offered a great deal of aid. The researchers found that the people using the simple software developed better strategies, made fewer mistakes and developed a deeper aptitude for the work. The people using the more advanced software, meanwhile, would often "aimlessly click around" when confronted with a tricky problem. The supposedly helpful software actually short-circuited their thinking and learning.

[According to] philosopher Hubert Dreyfus . . . . our skills get sharper only through practice, when we use them regularly to overcome different sorts of difficult challenges. The goal of modern software, by contrast, is to ease our way through such challenges. Arduous, painstaking work is exactly what programmers are most eager to automate-after all, that is where the immediate efficiency gains tend to lie. In other words, a fundamental tension ripples between the interests of the people doing the automation and the interests of the people doing the work.

Nevertheless, automation's scope continues to widen. With the rise of electronic health records, physicians increasingly rely on software templates to guide them through patient exams. The programs incorporate valuable checklists and alerts, but they also make medicine more routinized and formulaic-and distance doctors from their patients. . . . Harvard Medical School professor Beth Lown, in a 2012 journal article . . . warned that when doctors become "screen-driven," following a computer's prompts rather than "the patient's narrative thread," their thinking can become constricted. In the worst cases, they may miss important diagnostic signals. . . .

In a recent paper published in the journal Diagnosis, three medical researchers . . . examined the misdiagnosis of Thomas Eric Duncan, the first person to die of Ebola in the U.S., at Texas Health Presbyterian Hospital Dallas. They argue that the digital templates used by the hospital's clinicians to record patient information probably helped to induce a kind of tunnel vision. "These highly constrained tools," the researchers write, "are optimized for data capture but at the expense of sacrificing their utility for appropriate triage and diagnosis, leading users to miss the forest for the trees." Medical software, they write, is no "replacement for basic history-taking, examination skills, and critical thinking." . . .

There is an alternative. In "human-centered automation," the talents of people take precedence. . . . In this model, software plays an essential but secondary role. It takes over routine functions that a human operator has already mastered, issues alerts when unexpected situations arise, provides fresh information that expands the operator's perspective and counters the biases that often distort human thinking. The technology becomes the expert's partner, not the expert's replacement.

  1. CAT 2022 Slot 3 - VA RC

    From the passage, we can infer that the author is apprehensive about the use of sophisticated automation for all of the following reasons EXCEPT that:

    1. it stops users from exercising their minds.
    2. it could mislead people.
    3. computers could replace humans.
    4. it stunts the development of its users.
    Choice C
    computers could replace humans.

  2. CAT 2022 Slot 3 - VA RC

    In the Ebola misdiagnosis case, we can infer that doctors probably missed the forest for the trees because:

    1. they were led by the data processed by digital templates.
    2. the digital templates forced them to acquire tunnel vision.
    3. they used the wrong type of digital templates for the case.
    4. the data collected were not sufficient for appropriate triage.
    Choice A
    they were led by the data processed by digital templates.

  3. CAT 2022 Slot 3 - VA RC

    In the context of the passage, all of the following can be considered examples of human-centered automation EXCEPT:

    1. software that offers interpretations when requested by the human operator.
    2. software that auto-completes text when the user writes an email.
    3. medical software that provides optional feedback on the doctor's analysis of the medical situation.
    4. a smart-home system that changes the temperature as instructed by the resident.
    Choice B
    software that auto-completes text when the user writes an email.

  4. CAT 2022 Slot 3 - VA RC

    It can be inferred that in the Utrecht University experiment, one group of people was "aimlessly clicking around" because:

    1. they were hoping that the software would help carry out the tasks.
    2. they did not have the skill-set to address complicated tasks.
    3. the other group was carrying out the tasks more efficiently.
    4. they wanted to avoid making mistakes.
    Choice A
    they were hoping that the software would help carry out the tasks.

The passage below is accompanied by a set of questions. Choose the best answer to each question.

Nature has all along yielded her flesh to humans. First, we took nature's materials as food, fibers, and shelter. Then we learned to extract raw materials from her biosphere to create our own new synthetic materials. Now Bios is yielding us her mind-we are taking her logic.

Clockwork logic-the logic of the machines-will only build simple contraptions. Truly complex systems such as a cell, a meadow, an economy, or a brain (natural or artificial) require a rigorous nontechnological logic. We now see that no logic except bio-logic can assemble a thinking device, or even a workable system of any magnitude.

It is an astounding discovery that one can extract the logic of Bios out of biology and have something useful. Although many philosophers in the past have suspected one could abstract the laws of life and apply them elsewhere, it wasn't until the complexity of computers and human-made systems became as complicated as living things, that it was possible to prove this. It's eerie how much of life can be transferred. So far, some of the traits of the living that have successfully been transported to mechanical systems are: self-replication, self-governance, limited self-repair, mild evolution, and partial learning.

We have reason to believe yet more can be synthesized and made into something new. Yet at the same time that the logic of Bios is being imported into machines, the logic of Technos is being imported into life. The root of bioengineering is the desire to control the organic long enough to improve it. Domesticated plants and animals are examples of technos-logic applied to life. The wild aromatic root of the Queen Anne's lace weed has been fine-tuned over generations by selective herb gatherers until it has evolved into a sweet carrot of the garden; the udders of wild bovines have been selectively enlarged in a "unnatural" way to satisfy humans rather than calves. Milk cows and carrots, therefore, are human inventions as much as steam engines and gunpowder are. But milk cows and carrots are more indicative of the kind of inventions humans will make in the future: products that are grown rather than manufactured.

Genetic engineering is precisely what cattle breeders do when they select better strains of Holsteins, only bioengineers employ more precise and powerful control. While carrot and milk cow breeders had to rely on diffuse organic evolution, modern genetic engineers can use directed artificial evolution-purposeful design-which greatly accelerates improvements.

The overlap of the mechanical and the lifelike increases year by year. Part of this bionic convergence is a matter of words. The meanings of "mechanical" and "life" are both stretching until all complicated things can be perceived as machines, and all self-sustaining machines can be perceived as alive. Yet beyond semantics, two concrete trends are happening: (1) Human-made things are behaving more lifelike, and (2) Life is becoming more engineered. The apparent veil between the organic and the manufactured has crumpled to reveal that the two really are, and have always been, of one being.

  1. CAT 2022 Slot 3 - VA RC

    The author claims that, "Part of this bionic convergence is a matter of words". Which one of the following statements best expresses the point being made by the author?

    1. "Bios" and "Technos" are both convergent forms of logic, but they generate meanings about the world that are mutually exclusive.
    2. A bionic convergence indicates the meeting ground of genetic engineering and artificial intelligence.
    3. "Mechanical" and "life" are words from different logical systems and are, therefore, fundamentally incompatible in meaning.
    4. "Mechanical" and "life" were earlier seen as opposite in meaning, but the difference between the two is increasingly blurred.
    Choice D
    "Mechanical" and "life" were earlier seen as opposite in meaning, but the difference between the two is increasingly blurred.

  2. CAT 2022 Slot 3 - VA RC

    Which one of the following sets of words/phrases best serves as keywords to the passage?

    1. Complex systems; Carrots; Milk cows; Convergence; Technos-logic
    2. Nature; Bios; Technos; Self-repair; Holsteins
    3. Nature; Computers; Carrots; Milk cows; Genetic engineering
    4. Complex systems; Bio-logic; Bioengineering; Technos-logic; Convergence
    Choice D
    Complex systems; Bio-logic; Bioengineering; Technos-logic; Convergence

  3. CAT 2022 Slot 3 - VA RC

    None of the following statements is implied by the arguments of the passage, EXCEPT:

    1. purposeful design represents the pinnacle of scientific expertise in the service of human betterment and civilisational progress.
    2. the biological realm is as complex as the mechanical one; which is why the logic of Bios is being imported into machines.
    3. genetic engineers and bioengineers are the same insofar as they both seek to force evolution in an artificial way.
    4. historically, philosophers have known that the laws of life can be abstracted and applied elsewhere.
    Choice C
    genetic engineers and bioengineers are the same insofar as they both seek to force evolution in an artificial way.

  4. CAT 2022 Slot 3 - VA RC

    The author claims that, "The apparent veil between the organic and the manufactured has crumpled to reveal that the two really are, and have always been, of one being." Which one of the following statements best expresses the point being made by the author here?

    1. Scientific advances are making it increasingly difficult to distinguish between organic reality and manufactured reality.
    2. Apparent reality and organic reality are distinguished by the fact that the former is manufactured.
    3. Organic reality has crumpled under the veil of manufacturing, rendering the apparent and the real as the same being.
    4. The crumpling of the organic veil between apparent and manufactured reality reveals them to have the same being.
    Choice A
    Scientific advances are making it increasingly difficult to distinguish between organic reality and manufactured reality.

The passage below is accompanied by a set of questions. Choose the best answer to each question.

Interpretations of the Indian past . . . were inevitably influenced by colonial concerns and interests, and also by prevalent European ideas about history, civilization and the Orient. Orientalist scholars studied the languages and the texts with selected Indian scholars, but made little attempt to understand the world-view of those who were teaching them. The readings therefore are something of a disjuncture from the traditional ways of looking at the Indian past. . . .

Orientalism [which we can understand broadly as Western perceptions of the Orient] fuelled the fantasy and the freedom sought by European Romanticism, particularly in its opposition to the more disciplined Neo-Classicism. The cultures of Asia were seen as bringing a new Romantic paradigm. Another Renaissance was anticipated through an acquaintance with the Orient, and this, it was thought, would be different from the earlier Greek Renaissance. It was believed that this Oriental Renaissance would liberate European thought and literature from the increasing focus on discipline and rationality that had followed from the earlier Enlightenment. . . . [The Romantic English poets, Wordsworth and Coleridge,] were apprehensive of the changes introduced by industrialization and turned to nature and to fantasies of the Orient.

However, this enthusiasm gradually changed, to conform with the emphasis later in the nineteenth century on the innate superiority of European civilization. Oriental civilizations were now seen as having once been great but currently in decline. The various phases of Orientalism tended to mould European understanding of the Indian past into a particular pattern. . . . There was an attempt to formulate Indian culture as uniform, such formulations being derived from texts that were given priority. The so-called 'discovery' of India was largely through selected literature in Sanskrit. This interpretation tended to emphasize non-historical aspects of Indian culture, for example the idea of an unchanging continuity of society and religion over 3,000 years; and it was believed that the Indian pattern of life was so concerned with metaphysics and the subtleties of religious belief that little attention was given to the more tangible aspects.

German Romanticism endorsed this image of India, and it became the mystic land for many Europeans, where even the most ordinary actions were imbued with a complex symbolism. This was the genesis of the idea of the spiritual east, and also, incidentally, the refuge of European intellectuals seeking to distance themselves from the changing patterns of their own societies. A dichotomy in values was maintained, Indian values being described as 'spiritual' and European values as 'materialistic', with little attempt to juxtapose these values with the reality of Indian society. This theme has been even more firmly endorsed by a section of Indian opinion during the last hundred years.

It was a consolation to the Indian intelligentsia for its perceived inability to counter the technical superiority of the west, a superiority viewed as having enabled Europe to colonize Asia and other parts of the world. At the height of anti-colonial nationalism it acted as a salve for having been made a colony of Britain.

  1. CAT 2022 Slot 3 - VA RC

    It can be inferred from the passage that to gain a more accurate view of a nation's history and culture, scholars should do all of the following EXCEPT:

    1. develop an oppositional framework to grasp cultural differences.
    2. examine the complex reality of that nation's society.
    3. read widely in the country's literature.
    4. examine their own beliefs and biases.
    Choice A
    develop an oppositional framework to grasp cultural differences.

  2. CAT 2022 Slot 3 - VA RC

    It can be inferred from the passage that the author is not likely to support the view that:

    1. the Orientalist view of Asia fired the imagination of some Western poets.
    2. Indian culture acknowledges the material aspects of life.
    3. India's culture has evolved over the centuries.
    4. India became a colony although it matched the technical knowledge of the West.
    Choice D
    India became a colony although it matched the technical knowledge of the West.

  3. CAT 2022 Slot 3 - VA RC

    Which one of the following styles of research is most similar to the Orientalist scholars' method of understanding Indian history and culture?

    1. Reading about the life of early American settlers and later waves of migration to understand the evolution of American culture.
    2. Reading 18th century accounts by travellers to India to see how they viewed Indian life and culture of the time.
    3. Studying artefacts excavated at a palace to understand the lifestyle of those who lived there.
    4. Analysing Hollywood action movies that depict violence and sex to understand contemporary America.
    Choice D
    Analysing Hollywood action movies that depict violence and sex to understand contemporary America.

  4. CAT 2022 Slot 3 - VA RC

    In the context of the passage, all of the following statements are true EXCEPT:

    1. Orientalist scholarship influenced Indians.
    2. Indian texts influenced Orientalist scholars.
    3. India's spiritualism served as a salve for European colonisers.
    4. Orientalists' understanding of Indian history was linked to colonial concerns.
    Choice C
    India's spiritualism served as a salve for European colonisers.

The passage below is accompanied by a set of questions. Choose the best answer to each question.

Sociologists working in the Chicago School tradition have focused on how rapid or dramatic social change causes increases in crime. Just as Durkheim, Marx, Toennies, and other European sociologists thought that the rapid changes produced by industrialization and urbanization produced crime and disorder, so too did the Chicago School theorists. The location of the University of Chicago provided an excellent opportunity for Park, Burgess, and McKenzie to study the social ecology of the city. Shaw and McKay found . . . that areas of the city characterized by high levels of social disorganization had higher rates of crime and delinquency.

In the 1920s and 1930s Chicago, like many American cities, experienced considerable immigration. Rapid population growth is a disorganizing influence, but growth resulting from in-migration of very different people is particularly disruptive. Chicago's in-migrants were both native-born whites and blacks from rural areas and small towns, and foreign immigrants. The heavy industry of cities like Chicago, Detroit, and Pittsburgh drew those seeking opportunities and new lives. Farmers and villagers from America's hinterland, like their European cousins of whom Durkheim wrote, moved in large numbers into cities. At the start of the twentieth century, Americans were predominately a rural population, but by the century's mid-point most lived in urban areas. The social lives of these migrants, as well as those already living in the cities they moved to, were disrupted by the differences between urban and rural life. According to social disorganization theory, until the social ecology of the ''new place'' can adapt, this rapid change is a criminogenic influence. But most rural migrants, and even many of the foreign immigrants to the city, looked like and eventually spoke the same language as the natives of the cities into which they moved. These similarities allowed for more rapid social integration for these migrants than was the case for African Americans and most foreign immigrants.

In these same decades America experienced what has been called ''the great migration'': the massive movement of African Americans out of the rural South and into northern (and some southern) cities. The scale of this migration is one of the most dramatic in human history. These migrants, unlike their white counterparts, were not integrated into the cities they now called home. In fact, most American cities at the end of the twentieth century were characterized by high levels of racial residential segregation . . . Failure to integrate these migrants, coupled with other forces of social disorganization such as crowding, poverty, and illness, caused crime rates to climb in the cities, particularly in the segregated wards and neighborhoods where the migrants were forced to live.

Foreign immigrants during this period did not look as dramatically different from the rest of the population as blacks did, but the migrants from eastern and southern Europe who came to American cities did not speak English, and were frequently Catholic, while the native born were mostly Protestant. The combination of rapid population growth with the diversity of those moving into the cities created what the Chicago School sociologists called social disorganization.

  1. CAT 2022 Slot 3 - VA RC

    A fundamental conclusion by the author is that:

    1. the best circumstances for crime to flourish are when there are severe racial disparities.
    2. to prevent crime, it is important to maintain social order through maintaining social segregation.
    3. according to European sociologists, crime in America is mainly in Chicago.
    4. rapid population growth and demographic diversity give rise to social disorganisation that can feed the growth of crime.
    Choice D
    rapid population growth and demographic diversity give rise to social disorganisation that can feed the growth of crime.

  2. CAT 2022 Slot 3 - VA RC

    The author notes that, "At the start of the twentieth century, Americans were predominately a rural population, but by the century's mid-point most lived in urban areas." Which one of the following statements, if true, does not contradict this statement?

    1. Demographic transition in America in the twentieth century is strongly marked by an out-migration from rural areas.
    2. A population census conducted in 1952 showed that more Americans lived in rural areas than in urban ones.
    3. The estimation of per capita income in America in the mid-twentieth century primarily required data from rural areas.
    4. Economists have found that throughout the twentieth century, the size of the labour force in America has always been largest in rural areas.
    Choice A
    Demographic transition in America in the twentieth century is strongly marked by an out-migration from rural areas.

  3. CAT 2022 Slot 3 - VA RC

    Which one of the following is not a valid inference from the passage?

    1. The failure to integrate in-migrants, along with social problems like poverty, was a significant reason for the rise in crime in American cities.
    2. According to social disorganisation theory, fast-paced social change provides fertile ground for the rapid growth of crime.
    3. The differences between urban and rural lifestyles were crucial factors in the disruption experienced by migrants to American cities.
    4. According to social disorganisation theory, the social integration of African American migrants into Chicago was slower because they were less organised.
    Choice D
    According to social disorganisation theory, the social integration of African American migrants into Chicago was slower because they were less organised.

  4. CAT 2022 Slot 3 - VA RC

    Which one of the following sets of words/phrases best encapsulates the issues discussed in the passage?

    1. Durkheim; Marx; Toennies; Shaw
    2. Chicago School; Native-born Whites; European immigrants; Poverty
    3. Chicago School; Social organisation; Migration; Crime
    4. Rapid population growth; Heavy industry; Segregation; Crime
    Choice C
    Chicago School; Social organisation; Migration; Crime

CAT VARC : CAT 2021 Question Paper Slot 1

The passage below is accompanied by a set of questions. Choose the best answer to each question.

The sleights of hand that conflate consumption with virtue are a central theme in A Thirst for Empire, a sweeping and richly detailed history of tea by the historian Erika Rappaport. How did tea evolve from an obscure "China drink" to a universal beverage imbued with civilising properties? The answer, in brief, revolves around this conflation, not only by profit-motivated marketers but by a wide variety of interest groups. While abundant historical records have allowed the study of how tea itself moved from east to west, Rappaport is focused on the movement of the idea of tea to suit particular purposes.

Beginning in the 1700s, the temperance movement advocated for tea as a pleasure that cheered but did not inebriate, and industrialists soon borrowed this moral argument in advancing their case for free trade in tea (and hence more open markets for their textiles). Factory owners joined in, compelled by the cause of a sober workforce, while Christian missionaries discovered that tea "would soothe any colonial encounter". During the Second World War, tea service was presented as a social and patriotic activity that uplifted soldiers and calmed refugees.

But it was tea's consumer-directed marketing by importers and retailers “ and later by brands “ that most closely portends current trade debates. An early version of the "farm to table" movement was sparked by anti-Chinese sentiment and concerns over trade deficits, as well as by the reality and threat of adulterated tea containing dirt and hedge clippings. Lipton was soon advertising "from the Garden to Tea Cup" supply chains originating in British India and supervised by "educated Englishmen". While tea marketing always presented direct consumer benefits (health, energy, relaxation), tea drinkers were also assured that they were participating in a larger noble project that advanced the causes of family, nation and civilization. . . .

Rappaport's treatment of her subject is refreshingly apolitical. Indeed, it is a virtue that readers will be unable to guess her political orientation: both the miracle of markets and capitalism's dark underbelly are evident in tea's complex story, as are the complicated effects of British colonialism. . . . Commodity histories are now themselves commodities: recent works investigate cotton, salt, cod, sugar, chocolate, paper and milk. And morality marketing is now a commodity as well, applied to food, "fair trade" apparel and eco-tourism. Yet tea is, Rappaport makes clear, a world apart “ an astonishing success story in which tea marketers not only succeeded in conveying a sense of moral elevation to the consumer but also arguably did advance the cause of civilisation and community.

I have been offered tea at a British garden party, a Bedouin campfire, a Turkish carpet shop and a Japanese chashitsu, to name a few settings. In each case the offering was more an idea “ friendship, community, respect “ than a drink, and in each case the idea then created a reality. It is not a stretch to say that tea marketers have advanced the particularly noble cause of human dialogue and friendship.

  1. CAT 2021 Slot 1 - VA RC

    The author of this book review is LEAST likely to support the view that:

    1. tea drinking was sometimes promoted as a patriotic duty.
    2. the ritual of drinking tea promotes congeniality and camaraderie.
    3. tea drinking has become a social ritual worldwide.
    4. tea became the leading drink in Britain in the nineteenth century.
    Choice D
    tea became the leading drink in Britain in the nineteenth century.

  2. CAT 2021 Slot 1 - VA RC

    This book review argues that, according to Rappaport, tea is unlike other "morality" products because it:

    1. appealed to a universal group and not just to a niche section of people.
    2. had an actual beneficial effect on social interaction and society in general.
    3. was actively encouraged by interest groups in the government.
    4. was marketed by a wide range of interest groups.
    Choice B
    had an actual beneficial effect on social interaction and society in general.

  3. CAT 2021 Slot 1 - VA RC

    According to this book review, A Thirst for Empire says that, in addition to "profit-motivated marketers", tea drinking was promoted in Britain by all of the following EXCEPT:

    1. factories to instill sobriety in their labour.
    2. tea drinkers lobbying for product diversity.
    3. manufacturers who were pressing for duty-free imports.
    4. the anti-alcohol lobby as a substitute for the consumption of liquor.
    Choice B
    tea drinkers lobbying for product diversity.

  4. CAT 2021 Slot 1 - VA RC

    Today, "conflat[ing] consumption with virtue" can be seen in the marketing of:

    1. sustainably farmed foods.
    2. ergonomically designed products.
    3. travel to pristine destinations.
    4. natural health supplements.
    Choice A
    sustainably farmed foods.

The passage below is accompanied by a set of questions. Choose the best answer to each question.

For the Maya of the Classic period, who lived in Southern Mexico and Central America between 250 and 900 CE, the category of 'persons' was not coincident with human beings, as it is for us. That is, human beings were persons “ but other, nonhuman entities could be persons, too. . . . In order to explore the slippage of categories between 'humans' and 'persons', I examined a very specific category of ancient Maya images, found painted in scenes on ceramic vessels. I sought out instances in which faces (some combination of eyes, nose, and mouth) are shown on inanimate objects. . . . Consider my iPhone, which needs to be fed with electricity every night, swaddled in a protective bumper, and enjoys communicating with other fellow-phone-beings. Does it have personhood (if at all) because it is connected to me, drawing this resource from me as an owner or source? For the Maya (who did have plenty of other communicating objects, if not smartphones), the answer was no. Nonhuman persons were not tethered to specific humans, and they did not derive their personhood from a connection with a human. . . . It's a profoundly democratising way of understanding the world. Humans are not more important persons “ we are just one of many kinds of persons who inhabit this world. . . .

The Maya saw personhood as 'activated' by experiencing certain bodily needs and through participation in certain social activities. For example, among the faced objects that I examined, persons are marked by personal requirements (such as hunger, tiredness, physical closeness), and by community obligations (communication, interaction, ritual observance). In the images I examined, we see, for instance, faced objects being cradled in humans' arms; we also see them speaking to humans. These core elements of personhood are both turned inward, what the body or self of a person requires, and outward, what a community expects of the persons who are a part of it, underlining the reciprocal nature of community membership. . . .

Personhood was a nonbinary proposition for the Maya. Entities were able to be persons while also being something else. The faced objects I looked at indicate that they continue to be functional, doing what objects do (a stone implement continues to chop, an incense burner continues to do its smoky work). Furthermore, the Maya visually depicted many objects in ways that indicated the material category to which they belonged “ drawings of the stone implement show that a person-tool is still made of stone. One additional complexity: the incense burner (which would have been made of clay, and decorated with spiky appliques representing the sacred ceiba tree found in this region) is categorised as a person “ but also as a tree. With these Maya examples, we are challenged to discard the person/nonperson binary that constitutes our basic ontological outlook. . . . The porousness of boundaries that we have seen in the Maya world points towards the possibility of living with a certain uncategorisability of the world.

  1. CAT 2021 Slot 1 - VA RC

    Which one of the following, if true about the Classic Maya, would invalidate the purpose of the iPhone example in the passage?

    1. The personhood of the incense burner and the stone chopper was a function of their usefulness to humans.
    2. Classic Maya songs represent both humans and non-living objects as characters, talking and interacting with each other.
    3. The clay incense burner with spiky appliques was categorised only as a person and not as a tree by the Classic Maya.
    4. Unlike modern societies equipped with mobile phones, the Classic Maya did not have any communicating objects.
    Choice A
    The personhood of the incense burner and the stone chopper was a function of their usefulness to humans.

  2. CAT 2021 Slot 1 - VA RC

    Which one of the following, if true, would not undermine the democratising potential of the Classic Maya worldview?

    1. They believed that animals like cats and dogs that live in proximity to humans have a more clearly articulated personhood.
    2. They understood the stone implement and the incense burner in a purely human form.
    3. While they believed in the personhood of objects and plants, they did not believe in the personhood of rivers and animals.
    4. They depicted their human healers with physical attributes of local medicinal plants.
    Choice D
    They depicted their human healers with physical attributes of local medicinal plants.

  3. CAT 2021 Slot 1 - VA RC

    On the basis of the passage, which one of the following worldviews can be inferred to be closest to that of the Classic Maya?

    1. A tribe that perceives its hunting weapons as sacred person-artefacts because of their significance to its survival.
    2. A futuristic society that perceives robots to be persons as well as robots because of their similarity to humans.
    3. A tribe that perceives plants as person-plants because they form an ecosystem and are marked by needs of nutrition.
    4. A tribe that perceives its utensils as person-utensils in light of their functionality and bodily needs.
    Choice C
    A tribe that perceives plants as person-plants because they form an ecosystem and are marked by needs of nutrition.

  4. CAT 2021 Slot 1 - VA RC

    Which one of the following best explains the "additional complexity" that the example of the incense burner illustrates regarding personhood for the Classic Maya?

    1. The example adds a new layer to the nonbinary understanding of personhood by bringing in a third category that shares a dissimilar relation with the previous two.
    2. The example complicates the nonbinary understanding of personhood by bringing in the sacred, establishing the porosity of the divine and the profane.
    3. The example provides an exception to the nonbinary understanding of personhood that the passage had hitherto established.
    4. The example adds a new layer to the nonbinary understanding of personhood by bringing in a third category that shares a similar relation with the previous two.
    Choice D
    The example adds a new layer to the nonbinary understanding of personhood by bringing in a third category that shares a similar relation with the previous two.

The passage below is accompanied by a set of questions. Choose the best answer to each question.

We cannot travel outside our neighbourhood without passports. We must wear the same plain clothes. We must exchange our houses every ten years. We cannot avoid labour. We all go to bed at the same time . . . We have religious freedom, but we cannot deny that the soul dies with the body, since 'but for the fear of punishment, they would have nothing but contempt for the laws and customs of society'. . . . In More's time, for much of the population, given the plenty and security on offer, such restraints would not have seemed overly unreasonable. For modern readers, however, Utopia appears to rely upon relentless transparency, the repression of variety, and the curtailment of privacy. Utopia provides security: but at what price? In both its external and internal relations, indeed, it seems perilously dystopian.

Such a conclusion might be fortified by examining selectively the tradition which follows More on these points. This often portrays societies where . . . 'it would be almost impossible for man to be depraved, or wicked'. . . . This is achieved both through institutions and mores, which underpin the common life. . . . The passions are regulated and inequalities of wealth and distinction are minimized. Needs, vanity, and emulation are restrained, often by prizing equality and holding riches in contempt. The desire for public power is curbed. Marriage and sexual intercourse are often controlled: in Tommaso Campanella's The City of the Sun (1623), the first great literary utopia after More's, relations are forbidden to men before the age of twenty-one and women before nineteen. Communal child-rearing is normal; for Campanella this commences at age two. Greater simplicity of life, 'living according to nature', is often a result: the desire for simplicity and purity are closely related. People become more alike in appearance, opinion, and outlook than they often have been. Unity, order, and homogeneity thus prevail at the cost of individuality and diversity. This model, as J. C. Davis demonstrates, dominated early modern utopianism. . . . And utopian homogeneity remains a familiar theme well into the twentieth century.

Given these considerations, it is not unreasonable to take as our starting point here the hypothesis that utopia and dystopia evidently share more in common than is often supposed. Indeed, they might be twins, the progeny of the same parents. Insofar as this proves to be the case, my linkage of both here will be uncomfortably close for some readers. Yet we should not mistake this argument for the assertion that all utopias are, or tend to produce, dystopias. Those who defend this proposition will find that their association here is not nearly close enough. For we have only to acknowledge the existence of thousands of successful intentional communities in which a cooperative ethos predominates and where harmony without coercion is the rule to set aside such an assertion. Here the individual's submersion in the group is consensual (though this concept is not unproblematic). It results not in enslavement but voluntary submission to group norms. Harmony is achieved without . . . harming others.

  1. CAT 2021 Slot 1 - VA RC

    Following from the passage, which one of the following may be seen as a characteristic of a utopian society?

    1. A society without any laws to restrain one's individuality.
    2. Institutional surveillance of every individual to ensure his/her security and welfare.
    3. A society where public power is earned through merit rather than through privilege.
    4. The regulation of homogeneity through promoting competitive heterogeneity.
    Choice B
    Institutional surveillance of every individual to ensure his/her security and welfare.

  2. CAT 2021 Slot 1 - VA RC

    Which sequence of words below best captures the narrative of the passage?

    1. Curtailment of privacy - Dystopia - Utopia - Intentional community.
    2. Relentless transparency - Homogeneity - Utopia - Dystopia.
    3. Utopia - Security - Dystopia - Coercion.
    4. Utopia - Security - Homogeneity - Intentional community.
    Choice D
    Utopia - Security - Homogeneity - Intentional community.

  3. CAT 2021 Slot 1 - VA RC

    All of the following statements can be inferred from the passage EXCEPT that:

    1. it is possible to see utopias as dystopias, with a change in perspective, because one person's utopia could be seen as another's dystopia.
    2. utopian and dystopian societies are twins, the progeny of the same parents.
    3. utopian societies exist in a long tradition of literature dealing with imaginary people practicing imaginary customs, in imaginary worlds.
    4. many conceptions of utopian societies emphasise the importance of social uniformity and cultural homogeneity.
    Choice B
    utopian and dystopian societies are twins, the progeny of the same parents.

  4. CAT 2021 Slot 1 - VA RC

    All of the following arguments are made in the passage EXCEPT that:

    1. in More's time, there was plenty and security, so people did not need restraints that could appear unreasonable.
    2. the tradition of utopian literature has often shown societies in which it would be nearly impossible for anyone to be sinful or criminal.
    3. there have been thousands of communities where homogeneity and stability have been achieved through choice, rather than by force.
    4. in early modern utopianism, the stability of utopian societies was seen to be achieved only with individuals surrendering their sense of self.
    Choice A
    in More's time, there was plenty and security, so people did not need restraints that could appear unreasonable.

The passage below is accompanied by a set of questions. Choose the best answer to each question.

Cuttlefish are full of personality, as behavioral ecologist Alexandra Schnell found out while researching the cephalopod's potential to display self-control. . . . "Self-control is thought to be the cornerstone of intelligence, as it is an important prerequisite for complex decision-making and planning for the future," says Schnell . . .

[Schnell's] study used a modified version of the "marshmallow test" . . . During the original marshmallow test, psychologist Walter Mischel presented children between age four and six with one marshmallow. He told them that if they waited 15 minutes and didn't eat it, he would give them a second marshmallow. A long-term follow-up study showed that the children who waited for the second marshmallow had more success later in life. . . . The cuttlefish version of the experiment looked a lot different. The researchers worked with six cuttlefish under nine months old and presented them with seafood instead of sweets. (Preliminary experiments showed that cuttlefishes' favorite food is live grass shrimp, while raw prawns are so-so and Asian shore crab is nearly unacceptable.) Since the researchers couldn't explain to the cuttlefish that they would need to wait for their shrimp, they trained them to recognize certain shapes that indicated when a food item would become available. The symbols were pasted on transparent drawers so that the cuttlefish could see the food that was stored inside. One drawer, labeled with a circle to mean "immediate," held raw king prawn. Another drawer, labeled with a triangle to mean "delayed," held live grass shrimp. During a control experiment, square labels meant "never."

"If their self-control is flexible and I hadn't just trained them to wait in any context, you would expect the cuttlefish to take the immediate reward [in the control], even if it's their second preference," says Schnell . . . and that's what they did. That showed the researchers that cuttlefish wouldn't reject the prawns if it was the only food available. In the experimental trials, the cuttlefish didn't jump on the prawns if the live grass shrimp were labeled with a triangle”many waited for the shrimp drawer to open up. Each time the cuttlefish showed it could wait, the researchers tacked another ten seconds on to the next round of waiting before releasing the shrimp. The longest that a cuttlefish waited was 130 seconds.

Schnell [says] that the cuttlefish usually sat at the bottom of the tank and looked at the two food items while they waited, but sometimes, they would turn away from the king prawn "as if to distract themselves from the temptation of the immediate reward." In past studies, humans, chimpanzees, parrots and dogs also tried to distract themselves while waiting for a reward.

Not every species can use self-control, but most of the animals that can share another trait in common: long, social lives. Cuttlefish, on the other hand, are solitary creatures that don't form relationships even with mates or young. . . . "We don't know if living in a social group is important for complex cognition unless we also show those abilities are lacking in less social species," says . . . comparative psychologist Jennifer Vonk.

  1. CAT 2021 Slot 1 - VA RC

    All of the following constitute a point of difference between the "original" and "modified" versions of the marshmallow test EXCEPT that:

    1. the former correlated self-control and future success, while the latter correlated self-control and survival advantages.
    2. the former was performed over a longer time span than the latter.
    3. the former had human subjects, while the latter had cuttlefish.
    4. the former used verbal communication with its subjects, while the latter had to develop a symbolic means of communication.
    Choice A
    the former correlated self-control and future success, while the latter correlated self-control and survival advantages.

  2. CAT 2021 Slot 1 - VA RC

    Which one of the following, if true, would best complement the passage's findings?

    1. Cuttlefish wait longer than 100 seconds for the shrimp drawer to open up.
    2. Cuttlefish live in big groups that exhibit sociability.
    3. Cuttlefish cannot distinguish between geometrical shapes.
    4. Cuttlefish are equally fond of live grass shrimp and raw prawn.
    Choice B
    Cuttlefish live in big groups that exhibit sociability.

  3. CAT 2021 Slot 1 - VA RC

    In which one of the following scenarios would the cuttlefish's behaviour demonstrate self-control?

    1. Asian shore crabs and raw prawns are simultaneously released while a live grass shrimp drawer labelled with a triangle is placed in front of the cuttlefish, to be opened after one minute.
    2. raw prawns are released while a live grass shrimp drawer labelled with a square is placed in front of the cuttlefish.
    3. live grass shrimp are released while two raw prawn drawers labelled with a circle and a triangle respectively are placed in front of the cuttlefish; the triangle-labelled drawer is opened after 50 seconds.
    4. raw prawns are released while an Asian shore crab drawer labelled with a triangle is placed in front of the cuttlefish, to be opened after one minute.
    Choice A
    Asian shore crabs and raw prawns are simultaneously released while a live grass shrimp drawer labelled with a triangle is placed in front of the cuttlefish, to be opened after one minute.

  4. CAT 2021 Slot 1 - VA RC

    Which one of the following cannot be inferred from Alexandra Schnell's experiment?

    1. Intelligence in a species is impossible without sociability.
    2. Like human children, cuttlefish are capable of self-control.
    3. Cuttlefish exert self-control with the help of diversions.
    4. Cuttlefish exercise choice when it comes to food.
    Choice A
    Intelligence in a species is impossible without sociability.

CAT VARC : CAT 2021 Question Paper Slot 2

The passage below is accompanied by a set of questions. Choose the best answer to each question.

It has been said that knowledge, or the problem of knowledge, is the scandal of philosophy. The scandal is philosophy's apparent inability to show how, when and why we can be sure that we know something or, indeed, that we know anything. Philosopher Michael Williams writes: 'Is it possible to obtain knowledge at all? This problem is pressing because there are powerful arguments, some very ancient, for the conclusion that it is not . . . Scepticism is the skeleton in Western rationalism's closet'. While it is not clear that the scandal matters to anyone but philosophers, philosophers point out that it should matter to everyone, at least given a certain conception of knowledge. For, they explain, unless we can ground our claims to knowledge as such, which is to say, distinguish it from mere opinion, superstition, fantasy, wishful thinking, ideology, illusion or delusion, then the actions we take on the basis of presumed knowledge – boarding an airplane, swallowing a pill, finding someone guilty of a crime – will be irrational and unjustifiable.

That is all quite serious-sounding but so also are the rattlings of the skeleton: that is, the sceptic's contention that we cannot be sure that we know anything – at least not if we think of knowledge as something like having a correct mental representation of reality, and not if we think of reality as something like things-as-they-are-in-themselves, independent of our perceptions, ideas or descriptions. For, the sceptic will note, since reality, under that conception of it, is outside our ken (we cannot catch a glimpse of things-in-themselves around the corner of our own eyes; we cannot form an idea of reality that floats above the processes of our conceiving it), we have no way to compare our mental representations with things-as-they-are-in-themselves and therefore no way to determine whether they are correct or incorrect. Thus the sceptic may repeat (rattling loudly), you cannot be sure you 'know' something or anything at all – at least not, he may add (rattling softly before disappearing), if that is the way you conceive 'knowledge'.

There are a number of ways to handle this situation. The most common is to ignore it. Most people outside the academy – and, indeed, most of us inside it – are unaware of or unperturbed by the philosophical scandal of knowledge and go about our lives without too many epistemic anxieties. We hold our beliefs and presumptive knowledges more or less confidently, usually depending on how we acquired them (I saw it with my own eyes; I heard it on Fox News; a guy at the office told me) and how broadly and strenuously they seem to be shared or endorsed by various relevant people: experts and authorities, friends and family members, colleagues and associates. And we examine our convictions more or less closely, explain them more or less extensively, and defend them more or less vigorously, usually depending on what seems to be at stake for ourselves and/or other people and what resources are available for reassuring ourselves or making our beliefs credible to others (look, it's right here on the page; add up the figures yourself; I happen to be a heart specialist).

  1. CAT 2021 Slot 2 - VA RC

    The author of the passage is most likely to support which one of the following statements?

    1. The confidence with which we maintain something to be true is usually independent of the source of the alleged truth.
    2. The actions taken on the basis of presumed knowledge are rational and justifiable if we are confident that that knowledge is widely held.
    3. The scandal of philosophy is that we might not know anything at all about reality if we think of reality as independent of our perceptions, ideas or descriptions.
    4. For the sceptic, if we think of reality as independent of our perceptions, ideas or descriptions, we should aim to know that reality independently too.
    Choice C
    The scandal of philosophy is that we might not know anything at all about reality if we think of reality as independent of our perceptions, ideas or descriptions.

  2. CAT 2021 Slot 2 - VA RC

    ". . . we cannot catch a glimpse of things-in-themselves around the corner of our own eyes; we cannot form an idea of reality that floats above the processes of our conceiving it . . ." Which one of the following statements best reflects the argument being made in this sentence?

    1. Our knowledge of reality cannot be merged with our process of conceiving it.
    2. If the reality of things is independent of our perception, logically we cannot perceive that reality.
    3. If the reality of things is independent of our eyesight, logically we cannot perceive our perception.
    4. Our knowledge of reality floats above our subjective perception of it.
    Choice B
    If the reality of things is independent of our perception, logically we cannot perceive that reality.

  3. CAT 2021 Slot 2 - VA RC

    According to the last paragraph of the passage, "We hold our beliefs and presumptive knowledges more or less confidently, usually depending on" something. Which one of the following most broadly captures what we depend on?

    1. All of the options listed here.
    2. How much of a stake we have in them; what resources there are to support them.
    3. Remaining outside the academy; ignoring epistemic anxieties.
    4. How we come to hold them; how widely they are held in our social circles.
    Choice D
    How we come to hold them; how widely they are held in our social circles.

  4. CAT 2021 Slot 2 - VA RC

    The author discusses all of the following arguments in the passage, EXCEPT:

    1. if we cannot distinguish knowledge from opinion or delusion, we will not be able to justify our actions.
    2. the best way to deal with scepticism about the veracity of knowledge is to ignore it.
    3. philosophers maintain that the scandal of philosophy should be of concern to everyone.
    4. sceptics believe that we can never fully know anything, if by "knowing" we mean knowledge of a reality that is independent of the knower.
    Choice B
    the best way to deal with scepticism about the veracity of knowledge is to ignore it.

The passage below is accompanied by a set of questions. Choose the best answer to each question.

It's easy to forget that most of the world's languages are still transmitted orally with no widely established written form. While speech communities are increasingly involved in projects to protect their languages – in print, on air and online – orality is fragile and contributes to linguistic vulnerability. But indigenous languages are about much more than unusual words and intriguing grammar: They function as vehicles for the transmission of cultural traditions, environmental understandings and knowledge about medicinal plants, all at risk when elders die and livelihoods are disrupted.

Both push and pull factors lead to the decline of languages. Through war, famine and natural disasters, whole communities can be destroyed, taking their language with them to the grave, such as the indigenous populations of Tasmania who were wiped out by colonists. More commonly, speakers live on but abandon their language in favor of another vernacular, a widespread process that linguists refer to as "language shift" from which few languages are immune. Such trading up and out of a speech form occurs for complex political, cultural and economic reasons – sometimes voluntary for economic and educational reasons, although often amplified by state coercion or neglect. Welsh, long stigmatized and disparaged by the British state, has rebounded with vigor.

Many speakers of endangered, poorly documented languages have embraced new digital media with excitement. Speakers of previously exclusively oral tongues are turning to the web as a virtual space for languages to live on. Internet technology offers powerful ways for oral traditions and cultural practices to survive, even thrive, among increasingly mobile communities. I have watched as videos of traditional wedding ceremonies and songs are recorded on smartphones in London by Nepali migrants, then uploaded to YouTube and watched an hour later by relatives in remote Himalayan villages . . .

Globalization is regularly, and often uncritically, pilloried as a major threat to linguistic diversity. But in fact, globalization is as much process as it is ideology, certainly when it comes to language. The real forces behind cultural homogenization are unbending beliefs, exchanged through a globalized delivery system, reinforced by the historical monolingualism prevalent in much of the West.

Monolingualism – the condition of being able to speak only one language – is regularly accompanied by a deep-seated conviction in the value of that language over all others. Across the largest economies that make up the G8, being monolingual is still often the norm, with multilingualism appearing unusual and even somewhat exotic. The monolingual mindset stands in sharp contrast to the lived reality of most the world, which throughout its history has been more multilingual than unilingual. Monolingualism, then, not globalization, should be our primary concern.

Multilingualism can help us live in a more connected and more interdependent world. By widening access to technology, globalization can support indigenous and scholarly communities engaged in documenting and protecting our shared linguistic heritage. For the last 5,000 years, the rise and fall of languages was intimately tied to the plow, sword and book. In our digital age, the keyboard, screen and web will play a decisive role in shaping the future linguistic diversity of our species.

  1. CAT 2021 Slot 2 - VA RC

    From the passage, we can infer that the author is in favour of:

    1. "language shifts" across languages.
    2. cultural homogenisation.
    3. an expanded state role in the preservation of languages.
    4. greater multilingualism.
    Choice D
    greater multilingualism.

  2. CAT 2021 Slot 2 - VA RC

    The author lists all of the following as reasons for the decline or disappearance of a language EXCEPT:

    1. the focus on only a few languages as a result of widespread internet use.
    2. governments promoting certain languages over others.
    3. people shifting away from their own language to study or work in another language.
    4. a catastrophic event that entirely eliminates a people and their culture.
    Choice A
    the focus on only a few languages as a result of widespread internet use.

  3. CAT 2021 Slot 2 - VA RC

    The author mentions the Welsh language to show that:

    1. efforts to integrate Welsh speakers in the English-speaking fold have been fruitless.
    2. while often pilloried, globalisation can, in fact, support linguistic revival.
    3. languages can revive even after their speakers have gone through a "language shift".
    4. vulnerable languages can rebound with state effort.
    Choice C
    languages can revive even after their speakers have gone through a "language shift".

  4. CAT 2021 Slot 2 - VA RC

    We can infer all of the following about indigenous languages from the passage EXCEPT that:

    1. their vocabulary and grammatical constructs have been challenging to document.
    2. they are repositories of traditional knowledge about the environment and culture.
    3. people are increasingly working on documenting these languages.
    4. they are in danger of being wiped out as most can only be transmitted orally.
    Choice A
    their vocabulary and grammatical constructs have been challenging to document.

The passage below is accompanied by a set of questions. Choose the best answer to each question.

I have elaborated . . . a framework for analyzing the contradictory pulls on [Indian] nationalist ideology in its struggle against the dominance of colonialism and the resolution it offered to those contradictions. Briefly, this resolution was built around a separation of the domain of culture into two spheres—the material and the spiritual. It was in the material sphere that the claims of Western civilization were the most powerful. Science, technology, rational forms of economic organization, modern methods of statecraft—these had given the European countries the strength to subjugate the non-European people . . . To overcome this domination, the colonized people had to learn those superior techniques of organizing material life and incorporate them within their own cultures. . . . But this could not mean the imitation of the West in every aspect of life, for then the very distinction between the West and the East would vanish—the self-identity of national culture would itself be threatened. . . .

The discourse of nationalism shows that the material/spiritual distinction was condensed into an analogous, but ideologically far more powerful, dichotomy: that between the outer and the inner. . . . Applying the inner/outer distinction to the matter of concrete day-to-day living separates the social space into ghar and bahir, the home and the world. The world is the external, the domain of the material; the home represents one's inner spiritual self, one's true identity. The world is a treacherous terrain of the pursuit of material interests, where practical considerations reign supreme. It is also typically the domain of the male. The home in its essence must remain unaffected by the profane activities of the material world—and woman is its representation. And so one gets an identification of social roles by gender to correspond with the separation of the social space into ghar and bahir. . . .

The colonial situation, and the ideological response of nationalism to the critique of Indian tradition, introduced an entirely new substance to [these dichotomies] and effected their transformation. The material/spiritual dichotomy, to which the terms world and home corresponded, had acquired . . . a very special significance in the nationalist mind. The world was where the European power had challenged the non-European peoples and, by virtue of its superior material culture, had subjugated them. But, the nationalists asserted, it had failed to colonize the inner, essential, identity of the East which lay in its distinctive, and superior, spiritual culture. . . . [I]n the entire phase of the national struggle, the crucial need was to protect, preserve and strengthen the inner core of the national culture, its spiritual essence. . . .

Once we match this new meaning of the home/world dichotomy with the identification of social roles by gender, we get the ideological framework within which nationalism answered the women's question. It would be a grave error to see in this, as liberals are apt to in their despair at the many marks of social conservatism in nationalist practice, a total rejection of the West. Quite the contrary: the nationalist paradigm in fact supplied an ideological principle of selection.

  1. CAT 2021 Slot 2 - VA RC

    Which one of the following, if true, would weaken the author's claims in the passage?

    1. Indian nationalists rejected the cause of English education for women during the colonial period.
    2. Forces of colonial modernity played an important role in shaping anti-colonial Indian nationalism.
    3. The Industrial Revolution played a crucial role in shaping the economic prowess of Britain in the eighteenth century.
    4. The colonial period saw the hybridisation of Indian culture in all realms as it came in contact with British/European culture.
    Choice D
    The colonial period saw the hybridisation of Indian culture in all realms as it came in contact with British/European culture.

  2. CAT 2021 Slot 2 - VA RC

    Which one of the following best describes the liberal perception of Indian nationalism?

    1. Indian nationalism embraced the changes brought about by colonialism in Indian women's traditional gender roles.
    2. Indian nationalist discourses reaffirmed traditional gender roles for Indian women.
    3. Indian nationalism's sophistication resided in its distinction of the material from the spiritual spheres.
    4. Indian nationalist discourses provided an ideological principle of selection.
    Choice B
    Indian nationalist discourses reaffirmed traditional gender roles for Indian women.

  3. CAT 2021 Slot 2 - VA RC

    On the basis of the information in the passage, all of the following are true about the spiritual/material dichotomy of Indian nationalism EXCEPT that it:

    1. helped in safeguarding the identity of Indian nationalism.
    2. constituted the premise of the ghar/bahir dichotomy.
    3. represented a continuation of age-old oppositions in Indian culture.
    4. was not as ideologically powerful as the inner/outer dichotomy.
    Choice C
    represented a continuation of age-old oppositions in Indian culture.

  4. CAT 2021 Slot 2 - VA RC

    Which one of the following explains the "contradictory pulls" on Indian nationalism?

    1. Despite its spiritual superiority, Indian nationalism had to fight against colonial domination.
    2. Despite its fight against colonial domination, Indian nationalism had to borrow from the coloniser in the spiritual sphere.
    3. Despite its fight against colonial domination, Indian nationalism had to borrow from the coloniser in the material sphere.
    4. Despite its scientific and technological inferiority, Indian nationalism had to fight against colonial domination.
    Choice C
    Despite its fight against colonial domination, Indian nationalism had to borrow from the coloniser in the material sphere.

The passage below is accompanied by a set of questions. Choose the best answer to each question.

Many people believe that truth conveys power. . . . Hence sticking with the truth is the best strategy for gaining power. Unfortunately, this is just a comforting myth. In fact, truth and power have a far more complicated relationship, because in human society, power means two very different things.

 On the one hand, power means having the ability to manipulate objective realities: to hunt animals, to construct bridges, to cure diseases, to build atom bombs. This kind of power is closely tied to truth. If you believe a false physical theory, you won't be able to build an atom bomb. On the other hand, power also means having the ability to manipulate human beliefs, thereby getting lots of people to cooperate effectively. Building atom bombs requires not just a good understanding of physics, but also the coordinated labor of millions of humans. Planet Earth was conquered by Homo sapiens rather than by chimpanzees or elephants, because we are the only mammals that can cooperate in very large numbers. And large-scale cooperation depends on believing common stories. But these stories need not be true. You can unite millions of people by making them believe in completely fictional stories about God, about race or about economics. The dual nature of power and truth results in the curious fact that we humans know many more truths than any other animal, but we also believe in much more nonsense. . . .

When it comes to uniting people around a common story, fiction actually enjoys three inherent advantages over the truth. First, whereas the truth is universal, fictions tend to be local. Consequently if we want to distinguish our tribe from foreigners, a fictional story will serve as a far better identity marker than a true story. . . . The second huge advantage of fiction over truth has to do with the handicap principle, which says that reliable signals must be costly to the signaler. Otherwise, they can easily be faked by cheaters. . . . If political loyalty is signaled by believing a true story, anyone can fake it. But believing ridiculous and outlandish stories exacts greater cost, and is therefore a better signal of loyalty. . . . Third, and most important, the truth is often painful and disturbing. Hence if you stick to unalloyed reality, few people will follow you. An American presidential candidate who tells the American public the truth, the whole truth and nothing but the truth about American history has a 100 percent guarantee of losing the elections. . . . An uncompromising adherence to the truth is an admirable spiritual practice, but it is not a winning political strategy. . . .

Even if we need to pay some price for deactivating our rational faculties, the advantages of increased social cohesion are often so big that fictional stories routinely triumph over the truth in human history. Scholars have known this for thousands of years, which is why scholars often had to decide whether they served the truth or social harmony. Should they aim to unite people by making sure everyone believes in the same fiction, or should they let people know the truth even at the price of disunity?

  1. CAT 2021 Slot 2 - VA RC

    Regarding which one of the following quotes could we argue that the author overemphasises the importance of fiction?

    1. ". . . scholars often had to decide whether they served the truth or social harmony. Should they aim to unite people by making sure everyone believes in the same fiction, or should they let people know the truth . . .?"
    2. "Hence sticking with the truth is the best strategy for gaining power. Unfortunately, this is just a comforting myth."
    3. "In fact, truth and power have a far more complicated relationship, because in human society, power means two very different things."
    4. "On the one hand, power means having the ability to manipulate objective realities: to hunt animals, to construct bridges, to cure diseases, to build atom bombs."
    Choice A
    ". . . scholars often had to decide whether they served the truth or social harmony. Should they aim to unite people by making sure everyone believes in the same fiction, or should they let people know the truth . . .?"

  2. CAT 2021 Slot 2 - VA RC

    The central theme of the passage is about the choice between:

    1. stories that unite people and those that distinguish groups from each other.
    2. leaders who unknowingly spread fictions and those who intentionally do so.
    3. attaining social cohesion and propagating objective truth.
    4. truth and power.
    Choice C
    attaining social cohesion and propagating objective truth.

  3. CAT 2021 Slot 2 - VA RC

    The author implies that, like scholars, successful leaders:

    1. today know how to create social cohesion better than in the past.
    2. know how to balance truth and social unity.
    3. need to leverage both types of power to remain in office.
    4. use myths to attain the first type of power.
    Choice B
    know how to balance truth and social unity.

  4. CAT 2021 Slot 2 - VA RC

    The author would support none of the following statements about political power EXCEPT that:

    1. while unalloyed truth is not recommended, leaders should stay as close as possible to it.
    2. people cannot handle the unvarnished truth, so leaders retain power by deviating from it.
    3. there are definite advantages to promoting fiction, but there needs to be some limit to a pervasive belief in myths.
    4. manipulating people's beliefs is politically advantageous, but a leader who propagates only myths is likely to lose power.
    Choice B
    people cannot handle the unvarnished truth, so leaders retain power by deviating from it.

CAT VARC : CAT 2021 Question Paper Slot 3

The passage below is accompanied by a set of questions. Choose the best answer to each question.

Today we can hardly conceive of ourselves without an unconscious. Yet between 1700 and 1900, this notion developed as a genuinely original thought. The "unconscious" burst the shell of conventional language, coined as it had been to embody the fleeting ideas and the shifting conceptions of several generations until, finally, it became fixed and defined in specialized terms within the realm of medical psychology and Freudian psychoanalysis.

The vocabulary concerning the soul and the mind increased enormously in the course of the nineteenth century. The enrichments of literary and intellectual language led to an altered understanding of the meanings that underlie time-honored expressions and traditional catchwords. At the same time, once coined, powerful new ideas attracted to themselves a whole host of seemingly unrelated issues, practices, and experiences, creating a peculiar network of preoccupations that as a group had not existed before. The drawn-out attempt to approach and define the unconscious brought together the spiritualist and the psychical researcher of borderline phenomena (such as apparitions, spectral illusions, haunted houses, mediums, trance, automatic writing); the psychiatrist or alienist probing the nature of mental disease, of abnormal ideation, hallucination, delirium, melancholia, mania; the surgeon performing operations with the aid of hypnotism; the magnetizer claiming to correct the disequilibrium in the universal flow of magnetic fluids but who soon came to be regarded as a clever manipulator of the imagination; the physiologist and the physician who puzzled over sleep, dreams, sleepwalking, anesthesia, the influence of the mind on the body in health and disease; the neurologist concerned with the functions of the brain and the physiological basis of mental life; the philosopher interested in the will, the emotions, consciousness, knowledge, imagination and the creative genius; and, last but not least, the psychologist.

Significantly, most if not all of these practices (for example, hypnotism in surgery or psychological magnetism) originated in the waning years of the eighteenth century and during the early decades of the nineteenth century, as did some of the disciplines (such as psychology and psychical research). The majority of topics too were either new or assumed hitherto unknown colors. Thus, before 1790, few if any spoke, in medical terms, of the affinity between creative genius and the hallucinations of the insane . . .

Striving vaguely and independently to give expression to a latent conception, various lines of thought can be brought together by some novel term. The new concept then serves as a kind of resting place or stocktaking in the development of ideas, giving satisfaction and a stimulus for further discussion or speculation. Thus, the massive introduction of the term unconscious by Hartmann in 1869 appeared to focalize many stray thoughts, affording a temporary feeling that a crucial step had been taken forward, a comprehensive knowledge gained, a knowledge that required only further elaboration, explication, and unfolding in order to bring in a bounty of higher understanding. Ultimately, Hartmann's attempt at defining the unconscious proved fruitless because he extended its reach into every realm of organic and inorganic, spiritual, intellectual, and instinctive existence, severely diluting the precision and compromising the impact of the concept.

  1. CAT 2021 Slot 3 - VA RC

    Which one of the following sets of words is closest to mapping the main arguments of the passage?

    1. Imagination; Magnetism; Psychiatry.
    2. Unconscious; Latent conception; Dreams.
    3. Language; Unconscious; Psychoanalysis.
    4. Literary language; Unconscious; Insanity.
    Choice C
    Language; Unconscious; Psychoanalysis.

  2. CAT 2021 Slot 3 - VA RC

    Which one of the following statements best describes what the passage is about?

    1. The discovery of the unconscious as a part of the human mind.
    2. The identification of the unconscious as an object of psychical research.
    3. The collating of diverse ideas under the single term: unconscious.
    4. The growing vocabulary of the soul and the mind, as diverse processes.
    Choice C
    The collating of diverse ideas under the single term: unconscious.

  3. CAT 2021 Slot 3 - VA RC

    "The enrichments of literary and intellectual language led to an altered understanding of the meanings that underlie time-honored expressions and traditional catchwords." Which one of the following interpretations of this sentence would be closest in meaning to the original?

    1. The meanings of time-honored expressions were changed by innovations in literary and intellectual language.
    2. Time-honored expressions and traditional catchwords were enriched by literary and intellectual language.
    3. All of the options listed here.
    4. Literary and intellectual language was altered by time-honored expressions and traditional catchwords.
    Choice A
    The meanings of time-honored expressions were changed by innovations in literary and intellectual language.

  4. CAT 2021 Slot 3 - VA RC

    All of the following statements may be considered valid inferences from the passage, EXCEPT:

    1. New conceptions in the nineteenth century could provide new knowledge because of the establishment of fields such as anaesthesiology.
    2. Unrelated practices began to be treated as related to each other, as knowledge of the mind grew in the nineteenth century.
    3. Eighteenth century thinkers were the first to perceive a connection between creative genius and insanity.
    4. Without the linguistic developments of the nineteenth century, the growth of understanding of the soul and the mind may not have happened.
    Choice A
    New conceptions in the nineteenth century could provide new knowledge because of the establishment of fields such as anaesthesiology.

The passage below is accompanied by a set of questions. Choose the best answer to each question.

Keeping time accurately comes with a price. The maximum accuracy of a clock is directly related to how much disorder, or entropy, it creates every time it ticks. Natalia Ares at the University of Oxford and her colleagues made this discovery using a tiny clock with an accuracy that can be controlled. The clock consists of a 50-nanometre-thick membrane of silicon nitride, vibrated by an electric current. Each time the membrane moved up and down once and then returned to its original position, the researchers counted a tick, and the regularity of the spacing between the ticks represented the accuracy of the clock. The researchers found that as they increased the clock's accuracy, the heat produced in the system grew, increasing the entropy of its surroundings by jostling nearby particles . . . "If a clock is more accurate, you are paying for it somehow," says Ares. In this case, you pay for it by pouring more ordered energy into the clock, which is then converted into entropy. "By measuring time, we are increasing the entropy of the universe," says Ares. The more entropy there is in the universe, the closer it may be to its eventual demise. "Maybe we should stop measuring time," says Ares. The scale of the additional entropy is so small, though, that there is no need to worry about its effects, she says.

The increase in entropy in timekeeping may be related to the "arrow of time", says Marcus Huber at the Austrian Academy of Sciences in Vienna, who was part of the research team. It has been suggested that the reason that time only flows forward, not in reverse, is that the total amount of entropy in the universe is constantly increasing, creating disorder that cannot be put in order again.

The relationship that the researchers found is a limit on the accuracy of a clock, so it doesn't mean that a clock that creates the most possible entropy would be maximally accurate - hence a large, inefficient grandfather clock isn't more precise than an atomic clock. "It's a bit like fuel use in a car. Just because I'm using more fuel doesn't mean that I'm going faster or further," says Huber.

When the researchers compared their results with theoretical models developed for clocks that rely on quantum effects, they were surprised to find that the relationship between accuracy and entropy seemed to be the same for both. . . . We can't be sure yet that these results are actually universal, though, because there are many types of clocks for which the relationship between accuracy and entropy haven't been tested. "It's still unclear how this principle plays out in real devices such as atomic clocks, which push the ultimate quantum limits of accuracy," says Mark Mitchison at Trinity College Dublin in Ireland. Understanding this relationship could be helpful for designing clocks in the future, particularly those used in quantum computers and other devices where both accuracy and temperature are crucial, says Ares. This finding could also help us understand more generally how the quantum world and the classical world are similar and different in terms of thermodynamics and the passage of time.

  1. CAT 2021 Slot 3 - VA RC

    "It's a bit like fuel use in a car. Just because I'm using more fuel doesn't mean that I'm going faster or further . . ." What is the purpose of this example?

    1. If you go faster in a car, you will tend to consume more fuel, but the converse is not necessarily true. In the same way, increased entropy does not necessarily mean greater accuracy of a clock.
    2. The further you go in a car, the more fuel you use. In the same way, the faster you go in a car, the less time you use.
    3. If you measure the speed of a car with a grandfather clock, the result will be different than if you measured it with an atomic clock.
    4. The further and faster you go in a car, the greater the amount of fuel you will use, the greater the amount of heat produced and, hence, the greater the entropy.
    Choice A
    If you go faster in a car, you will tend to consume more fuel, but the converse is not necessarily true. In the same way, increased entropy does not necessarily mean greater accuracy of a clock.

  2. CAT 2021 Slot 3 - VA RC

    The author makes all of the following arguments in the passage, EXCEPT that:

    1. The relationship between accuracy and entropy may not apply to all clocks.
    2. Researchers found that the heat produced in a system is the price paid for increased accuracy of measurement.
    3. There is no difference in accuracy between an inefficient grandfather clock and an atomic clock.
    4. In designing clocks for quantum computers, both precision and heat have to be taken into account.
    Choice B
    Researchers found that the heat produced in a system is the price paid for increased accuracy of measurement.

  3. CAT 2021 Slot 3 - VA RC

    Which one of the following sets of words and phrases serves best as keywords of the passage?

    1. Electric current; Heat; Quantum effects.
    2. Silicon Nitride; Energy; Grandfather Clock.
    3. Measuring Time; Accuracy; Entropy.
    4. Membrane; Arrow of time; Entropy.
    Choice C
    Measuring Time; Accuracy; Entropy.

  4. CAT 2021 Slot 3 - VA RC

    None of the following statements can be inferred from the passage EXCEPT that:

    1. the arrow of time has not yet been tested for atomic clocks.
    2. quantum computers are likely to produce more heat and, hence, more entropy, because of the emphasis on their clocks' accuracy.
    3. grandfather clocks are likely to produce less heat and, hence, less entropy, because they are not as accurate.
    4. a clock with a 50-nanometre-thick membrane of silicon nitride has been made to vibrate, producing electric currents.
    Choice B
    quantum computers are likely to produce more heat and, hence, more entropy, because of the emphasis on their clocks' accuracy.

The passage below is accompanied by a set of questions. Choose the best answer to each question.

Starting in 1957, [Noam Chomsky] proclaimed a new doctrine: Language, that most human of all attributes, was innate. The grammatical faculty was built into the infant brain, and your average 3-year-old was not a mere apprentice in the great enterprise of absorbing English from his or her parents, but a "linguistic genius." Since this message was couched in terms of Chomskyan theoretical linguistics, in discourse so opaque that it was nearly incomprehensible even to some scholars, many people did not hear it. Now, in a brilliant, witty and altogether satisfying book, Mr. Chomsky's colleague Steven Pinker . . . has brought Mr. Chomsky's findings to everyman. In "The Language Instinct" he has gathered persuasive data from such diverse fields as cognitive neuroscience, developmental psychology and speech therapy to make his points, and when he disagrees with Mr. Chomsky he tells you so. . . .

For Mr. Chomsky and Mr. Pinker, somewhere in the human brain there is a complex set of neural circuits that have been programmed with "super-rules" (making up what Mr. Chomsky calls "universal grammar"), and that these rules are unconscious and instinctive. A half-century ago, this would have been pooh-poohed as a "black box" theory, since one could not actually pinpoint this grammatical faculty in a specific part of the brain, or describe its functioning. But now things are different. Neurosurgeons [have now found that this] "black box" is situated in and around Broca's area, on the left side of the forebrain. . . .

Unlike Mr. Chomsky, Mr. Pinker firmly places the wiring of the brain for language within the framework of Darwinian natural selection and evolution. He effectively disposes of all claims that intelligent nonhuman primates like chimps have any abilities to learn and use language. It is not that chimps lack the vocal apparatus to speak; it is just that their brains are unable to produce or use grammar. On the other hand, the "language instinct," when it first appeared among our most distant hominid ancestors, must have given them a selective reproductive advantage over their competitors (including the ancestral chimps). . . .

So according to Mr. Pinker, the roots of language must be in the genes, but there cannot be a "grammar gene" any more than there can be a gene for the heart or any other complex body structure. This proposition will undoubtedly raise the hackles of some behavioral psychologists and anthropologists, for it apparently contradicts the liberal idea that human behavior may be changed for the better by improvements in culture and environment, and it might seem to invite the twin bugaboos of biological determinism and racism. Yet Mr. Pinker stresses one point that should allay such fears. Even though there are 4,000 to 6,000 languages today, they are all sufficiently alike to be considered one language by an extraterrestrial observer. In other words, most of the diversity of the world's cultures, so beloved to anthropologists, is superficial and minor compared to the similarities. Racial differences are literally only "skin deep." The fundamental unity of humanity is the theme of Mr. Chomsky's universal grammar, and of this exciting book.

  1. CAT 2021 Slot 3 - VA RC

    Which one of the following statements best summarises the author's position about Pinker's book?

    1. Anatomical developments like the voice box play a key role in determining language acquisition skills.
    2. The universality of the "language instinct" counters claims that Pinker's book is racist.
    3. The evolutionary and deterministic framework of Pinker's book makes it racist.
    4. Culture and environment play a key role in shaping our acquisition of language.
    Choice B
    The universality of the "language instinct" counters claims that Pinker's book is racist.

  2. CAT 2021 Slot 3 - VA RC

    According to the passage, all of the following are true about the language instinct EXCEPT that:

    1. all intelligent primates are gifted with it.
    2. not all intelligent primates are gifted with it.
    3. developments in neuroscience have increased its acceptance.
    4. it confers an evolutionary reproductive advantage.
    Choice A
    all intelligent primates are gifted with it.

  3. CAT 2021 Slot 3 - VA RC

    From the passage, it can be inferred that all of the following are true about Pinker's book, "The Language Instinct", EXCEPT that Pinker:

    1. writes in a different style from Chomsky.
    2. disagrees with Chomsky on certain grounds.
    3. draws extensively from Chomsky's propositions.
    4. draws from behavioural psychology theories.
    Choice D
    draws from behavioural psychology theories.

  4. CAT 2021 Slot 3 - VA RC

    On the basis of the information in the passage, Pinker and Chomsky may disagree with each other on which one of the following points?

    1. The possibility of a universal grammar.
    2. The Darwinian explanatory paradigm for language.
    3. The language instinct.
    4. The inborn language acquisition skills of humans.
    Choice B
    The Darwinian explanatory paradigm for language.

The passage below is accompanied by a set of questions. Choose the best answer to each question.

Back in the early 2000s, an awesome thing happened in the New X-Men comics. Our mutant heroes had been battling giant robots called Sentinels for years, but suddenly these mechanical overlords spawned a new threat: Nano-Sentinels! Not content to rule Earth with their metal fists, these tiny robots invaded our bodies at the microscopic level. Infected humans were slowly converted into machines, cell by cell.

Now, a new wave of extremely odd robots is making at least part of the Nano-Sentinels story come true. Using exotic fabrication materials like squishy hydrogels and elastic polymers, researchers are making autonomous devices that are often tiny and that could turn out to be more powerful than an army of Terminators. Some are 1-centimetre blobs that can skate over water. Others are flat sheets that can roll themselves into tubes, or matchstick-sized plastic coils that act as powerful muscles. No, they won't be invading our bodies and turning us into Sentinels - which I personally find a little disappointing - but some of them could one day swim through our bloodstream to heal us. They could also clean up pollutants in water or fold themselves into different kinds of vehicles for us to drive. . . .

Unlike a traditional robot, which is made of mechanical parts, these new kinds of robots are made from molecular parts. The principle is the same: both are devices that can move around and do things independently. But a robot made from smart materials might be nothing more than a pink drop of hydrogel. Instead of gears and wires, it's assembled from two kinds of molecules - some that love water and some that avoid it - which interact to allow the bot to skate on top of a pond.

Sometimes these materials are used to enhance more conventional robots. One team of researchers, for example, has developed a different kind of hydrogel that becomes sticky when exposed to a low-voltage zap of electricity and then stops being sticky when the electricity is switched off. This putty-like gel can be pasted right onto the feet or wheels of a robot. When the robot wants to climb a sheer wall or scoot across the ceiling, it can activate its sticky feet with a few volts. Once it is back on a flat surface again, the robot turns off the adhesive like a light switch.

Robots that are wholly or partly made of gloop aren't the future that I was promised in science fiction. But it's definitely the future I want. I'm especially keen on the nanometre-scale "soft robots" that could one day swim through our bodies. Metin Sitti, a director at the Max Planck Institute for Intelligent Systems in Germany, worked with colleagues to prototype these tiny, synthetic beasts using various stretchy materials, such as simple rubber, and seeding them with magnetic microparticles. They are assembled into a finished shape by applying magnetic fields. The results look like flowers or geometric shapes made from Tinkertoy ball and stick modelling kits. They're guided through tubes of fluid using magnets, and can even stop and cling to the sides of a tube.

  1. CAT 2021 Slot 3 - VA RC

    Which one of the following statements best summarises the central point of the passage?

    1. Once the stuff of science fiction, nano-robots now feature in cutting-edge scientific research.
    2. Robots will use nano-robots on their feet and wheels to climb walls or move on ceilings.
    3. The field of robotics is likely to be featured more and more in comics like the New X-Men.
    4. Nano-robots made from molecules that react to water have become increasingly useful.
    Choice A
    Once the stuff of science fiction, nano-robots now feature in cutting-edge scientific research.

  2. CAT 2021 Slot 3 - VA RC

    Which one of the following statements best captures the sense of the first paragraph?

    1. People who were infected by Nano-Sentinel robots became mutants who were called X-Men.
    2. None of the options listed here.
    3. Tiny sentinels called X-Men infected people, turning them into mutant robot overlords.
    4. The X-Men were mutant heroes who now had to battle tiny robots called Nano-Sentinels.
    Choice D
    The X-Men were mutant heroes who now had to battle tiny robots called Nano-Sentinels.

  3. CAT 2021 Slot 3 - VA RC

    Which one of the following scenarios, if false, could be seen as supporting the passage?

    1. Nano-Sentinel-like robots are likely to be used to inject people to convert them into robots, cell by cell.
    2. There are two kinds of molecules used to make some nano-robots: one that reacts positively to water and the other negatively.
    3. Robots made from smart materials are likely to become part of our everyday lives in the future.
    4. Some hydrogels turn sticky when an electric current is passed through them; this potentially has very useful applications.
    Choice A
    Nano-Sentinel-like robots are likely to be used to inject people to convert them into robots, cell by cell.

  4. CAT 2021 Slot 4 - VA RC

    Which one of the following statements, if true, would be the most direct extension of the arguments in the passage?

    1. In the future, robots will be used to search and destroy diseases even in the deepest recesses of the human body.
    2. Sentinel robots will be used in warfare to cause large-scale destructive mutations amongst civilians.
    3. X-Men may be created by injecting people with mutant nano-gels that will respond to the brain's magnetic field.
    4. 1-centimetre blobs of gel that have nano-robots in them will be used to send messages.
    Choice A
    In the future, robots will be used to search and destroy diseases even in the deepest recesses of the human body.

CAT VARC : CAT 2020 Question Paper Slot 1

The passage below is accompanied by a set of questions. Choose the best answer to each question.

The word ‘anarchy’ comes from the Greek anarkhia, meaning contrary to authority or without a ruler, and was used in a derogatory sense until 1840, when it was adopted by Pierre-Joseph Proudhon to describe his political and social ideology. Proudhon argued that organization without government was both possible and desirable. In the evolution of political ideas, anarchism can be seen as an ultimate projection of both liberalism and socialism, and the differing strands of anarchist thought can be related to their emphasis on one or the other of these. 

Historically, anarchism arose not only as an explanation of the gulf between the rich and the poor in any community, and of the reason why the poor have been obliged to fight for their share of a common inheritance, but as a radical answer to the question ‘What went wrong?’ that followed the ultimate outcome of the French Revolution. It had ended not only with a reign of terror and the emergence of a newly rich ruling caste, but with a new adored emperor, Napoleon Bonaparte, strutting through his conquered territories.

The anarchists and their precursors were unique on the political Left in affirming that workers and peasants, grasping the chance that arose to bring an end to centuries of exploitation and tyranny, were inevitably betrayed by the new class of politicians, whose first priority was to re-establish a centralized state power. After every revolutionary uprising, usually won at a heavy cost for ordinary populations, the new rulers had no hesitation in applying violence and terror, a secret police, and a professional army to maintain their control.

For anarchists the state itself is the enemy, and they have applied the same interpretation to the outcome of every revolution of the 19th and 20th centuries. This is not merely because every state keeps a watchful and sometimes punitive eye on its dissidents, but because every state protects the privileges of the powerful.

The mainstream of anarchist propaganda for more than a century has been anarchist-communism, which argues that property in land, natural resources, and the means of production should be held in mutual control by local communities, federating for innumerable joint purposes with other communes. It differs from state socialism in opposing the concept of any central authority. Some anarchists prefer to distinguish between anarchist-communism and collectivist anarchism in order to stress the obviously desirable freedom of an individual or family to possess the resources needed for living, while not implying the right to own the resources needed by others. . . . 

There are, unsurprisingly, several traditions of individualist anarchism, one of them deriving from the ‘conscious egoism’ of the German writer Max Stirner (1806–56), and another from a remarkable series of 19th-century American figures who argued that in protecting our own autonomy and associating with others for common advantages, we are promoting the good of all. These thinkers differed from free-market liberals in their absolute mistrust of American capitalism, and in their emphasis on mutualism. 

  1. CAT 2020 Question Paper Slot 1

    Which one of the following best expresses the similarity between American individualist anarchists and free-market liberals as well as the difference between the former and the latter?

    1. Both reject the regulatory power of the state; but the former favour a people’s state, while the latter favour state intervention in markets.
    2. Both prioritise individual autonomy; but the former also emphasise mutual dependence, while the latter do not do so.
    3. Both are sophisticated arguments for capitalism; but the former argue for a morally upright capitalism, while the latter argue that the market is the only morality.
    4. Both are founded on the moral principles of altruism; but the latter conceive of the market as a force too mystical for the former to comprehend.
    Choice B
    Both prioritise individual autonomy; but the former also emphasise mutual dependence, while the latter do not do so.

  2. CAT 2020 Question Paper Slot 1

    The author makes all of the following arguments in the passage, EXCEPT:

    1. Individualist anarchism is actually constituted of many streams, all of which focus on the autonomy of the individual.
    2. The popular perception of anarchism as espousing lawlessness and violence comes from a mainstream mistrust of collectivism.
    3. For anarchists, the state is the enemy because all states apply violence and terror to maintain their control.
    4. The failure of the French Revolution was because of its betrayal by the new class of politicians who emerged from it.
    Choice B
    The popular perception of anarchism as espousing lawlessness and violence comes from a mainstream mistrust of collectivism.

  3. CAT 2020 Question Paper Slot 1

    According to the passage, what is the one idea that is common to all forms of anarchism?

    1. There is no idea common to all forms of anarchism; that is why it is anarchic.
    2. They all focus on the primacy of the power of the individual.
    3. They all derive from the work of Pierre-Joseph Proudhon.
    4. They are all opposed to the centralisation of power in the state.
    Choice D
    They are all opposed to the centralisation of power in the state.

  4. CAT 2020 Question Paper Slot 1

    The author believes that the new ruling class of politicians betrayed the principles of the French Revolution, but does not specify in what way. In the context of the passage, which statement below is the likeliest explanation of that betrayal?

    1. The new ruling class rode to power on the strength of the workers’ revolutionary anger, but then turned to oppress that very class.
    2. The anarchists did not want a new ruling class, but were not politically strong enough to stop them.
    3. The new ruling class was constituted mainly of anarchists who were against the destructive impact of the Revolution on the market.
    4. The new ruling class struck a deal with the old ruling class to share power between them.
    Choice A
    The new ruling class rode to power on the strength of the workers’ revolutionary anger, but then turned to oppress that very class.

  5. CAT 2020 Question Paper Slot 1

    Of the following sets of concepts, identify the set that is conceptually closest to the concerns of the passage.

    1. Anarchism, Betrayal, Power, State.
    2. Revolution, State, Strike, Egoism.
    3. Revolution, State, Protection, Liberals.
    4. Anarchism, State, Individual, Freedom.
    Choice D
    Anarchism, State, Individual, Freedom.

The passage below is accompanied by a set of questions. Choose the best answer to each question.

In the late 1960s, while studying the northern-elephant-seal population along the coasts of Mexico and California, Burney Le Boeuf and his colleagues couldn’t help but notice that the threat calls of males at some sites sounded different from those of males at other sites. . . . That was the first time dialects were documented in a nonhuman mammal. . . .

All the northern elephant seals that exist today are descendants of the small herd that survived on Isla Guadalupe [after the near extinction of the species in the nineteenth century]. As that tiny population grew, northern elephant seals started to recolonize former breeding locations. It was precisely on the more recently colonized islands where Le Boeuf found that the tempos of the male vocal displays showed stronger differences to the ones from Isla Guadalupe, the founder colony. 

In order to test the reliability of these dialects over time, Le Boeuf and other researchers visited Año Nuevo Island in California—the island where males showed the slowest pulse rates in their calls—every winter from 1968 to 1972. “What we found is that the pulse rate increased, but it still remained relatively slow compared to the other colonies we had measured in the past” Le Boeuf told me.

At the individual level, the pulse of the calls stayed the same: A male would maintain his vocal signature throughout his lifetime. But the average pulse rate was changing. Immigration could have been responsible for this increase, as in the early 1970s, 43 percent of the males on Año Nuevo had come from southern rookeries that had a faster pulse rate. This led Le Boeuf and his collaborator, Lewis Petrinovich, to deduce that the dialects were, perhaps, a result of isolation over time, after the breeding sites had been recolonized. For instance, the first settlers of Año Nuevo could have had, by chance, calls with low pulse rates. At other sites, where the scientists found faster pulse rates, the opposite would have happened—seals with faster rates would have happened to arrive first.

As the population continued to expand and the islands kept on receiving immigrants from the original population, the calls in all locations would have eventually regressed to the average pulse rate of the founder colony. In the decades that followed, scientists noticed that the geographical variations reported in 1969 were not obvious anymore. . . . In the early 2010s, while studying northern elephant seals on Año Nuevo Island, [researcher Caroline] Casey noticed, too, that what Le Boeuf had heard decades ago was not what she heard now. . . . By performing more sophisticated statistical analyses on both sets of data, [Casey and Le Boeuf] confirmed that dialects existed back then but had vanished. Yet there are other differences between the males from the late 1960s and their great-great-grandsons: Modern males exhibit more individual diversity, and their calls are more complex. While 50 years ago the drumming pattern was quite simple and the dialects denoted just a change in tempo, Casey explained, the calls recorded today have more complex structures, sometimes featuring doublets or triplets. . . .

  1. CAT 2020 Question Paper Slot 1

    Which one of the following conditions, if true, could have ensured that male northern elephant seal dialects did not disappear?

    1. Besides Isla Guadalupe, there was one more surviving colony with the same average male call tempo from which no migration took place.
    2. The call tempo of individual male seals in host colonies changed to match the average call tempo of immigrant male seals.
    3. Besides Isla Guadalupe, there was one more founder colony with the same average male call tempo from which male seals migrated to various other colonies.
    4. The call tempo of individual immigrant male seals changed to match the average tempo of resident male seals in the host colony.
    Choice D
    The call tempo of individual immigrant male seals changed to match the average tempo of resident male seals in the host colony.

  2. CAT 2020 Question Paper Slot 1

    All of the following can be inferred from Le Boeuf’s study as described in the passage EXCEPT that:

    1. changes in population and migration had no effect on the call pulse rate of individual male northern elephant seals.
    2. the influx of new northern elephant seals into Año Nuevo Island would have soon made the call pulse rate of its male seals exceed that of those at Isla Guadalupe.
    3. male northern elephant seals might not have exhibited dialects had they not become nearly extinct in the nineteenth century.
    4. the average call pulse rate of male northern elephant seals at Año Nuevo Island increased from the early 1970s till the disappearance of dialects.
    Choice B
    the influx of new northern elephant seals into Año Nuevo Island would have soon made the call pulse rate of its male seals exceed that of those at Isla Guadalupe.

  3. CAT 2020 Question Paper Slot 1

    Which one of the following best sums up the overall history of transformation of male northern elephant seal calls?

    1. Owing to migrations in the aftermath of near species extinction, the calls have transformed from exhibiting complex composition, less individual variety, and great regional variety to simple composition, less individual variety, and great regional variety.
    2. The calls have transformed from exhibiting simple composition, great individual variety, and less regional variety to complex composition, less individual variety, and great regional variety.
    3. Owing to migrations in the aftermath of near species extinction, the average call pulse rates in the recolonised breeding locations exhibited a gradual increase until they matched the tempo at the founding colony.
    4. The calls have transformed from exhibiting simple composition, less individual variety, and great regional variety to complex composition, great individual variety, and less regional variety.
    Choice D
    The calls have transformed from exhibiting simple composition, less individual variety, and great regional variety to complex composition, great individual variety, and less regional variety.

  4. CAT 2020 Question Paper Slot 1

    From the passage it can be inferred that the call pulse rate of male northern elephant seals in the southern rookeries was faster because:

    1. a large number of male northern elephant seals migrated from the southern rookeries to Año Nuevo Island in the early 1970s.
    2. the male northern elephant seals of Isla Guadalupe with faster call pulse rates might have been the original settlers of the southern rookeries.
    3. the calls of male northern elephant seals in the southern rookeries have more sophisticated structures, containing doublets and triplets.
    4. a large number of male northern elephant seals from Año Nuevo Island might have migrated to the southern rookeries to recolonise them.
    Choice B
    the male northern elephant seals of Isla Guadalupe with faster call pulse rates might have been the original settlers of the southern rookeries.

The passage below is accompanied by a set of questions. Choose the best answer to each question.

Few realise that the government of China, governing an empire of some 60 million people during the Tang dynasty (618–907), implemented a complex financial system that recognised grain, coins and textiles as money. . . . Coins did have certain advantages: they were durable, recognisable and provided a convenient medium of exchange, especially for smaller transactions. However, there were also disadvantages. A continuing shortage of copper meant that government mints could not produce enough coins for the entire empire, to the extent that for most of the dynasty’s history, coins constituted only a tenth of the money supply. One of the main objections to calls for taxes to be paid in coin was that peasant producers who could weave cloth or grow grain – the other two major currencies of the Tang – would not be able to produce coins, and therefore would not be able to pay their taxes. . . . 

As coins had advantages and disadvantages, so too did textiles. If in circulation for a long period of time, they could show signs of wear and tear. Stained, faded and torn bolts of textiles had less value than a brand new bolt. Furthermore, a full bolt had a particular value. If consumers cut textiles into smaller pieces to buy or sell something worth less than a full bolt, that, too, greatly lessened the value of the textiles. Unlike coins, textiles could not be used for small transactions; as [an official] noted, textiles could not “be exchanged by the foot and the inch” . . . 

But textiles had some advantages over coins. For a start, textile production was widespread and there were fewer problems with the supply of textiles. For large transactions, textiles weighed less than their equivalent in coins since a string of coins . . .  could weigh as much as 4 kg. Furthermore, the dimensions of a bolt of silk held remarkably steady from the third to the tenth century: 56 cm wide and 12 m long . . . The values of different textiles were also more stable than the fluctuating values of coins. . . .  

The government also required the use of textiles for large transactions. Coins, on the other hand, were better suited for smaller transactions, and possibly, given the costs of transporting coins, for a more local usage. Grain, because it rotted easily, was not used nearly as much as coins and textiles, but taxpayers were required to pay grain to the government as a share of their annual tax obligations, and official salaries were expressed in weights of grain. . . . 

In actuality, our own currency system today has some similarities even as it is changing in front of our eyes. . . . We have cash – coins for small transactions like paying for parking at a meter, and banknotes for other items; cheques and debit/credit cards for other, often larger, types of payments. At the same time, we are shifting to electronic banking and making payments online. Some young people never use cash [and] do not know how to write a cheque . . . 

  1. CAT 2020 Question Paper Slot 1

    In the context of the passage, which one of the following can be inferred with regard to the use of currency during the Tang era?

    1. Currency usage was similar to that of modern times.
    2. Currency that deteriorated easily was not used for official work.
    3. Copper coins were more valuable and durable than textiles.
    4. Grains were the most used currency because of government requirements.
    Choice A
    Currency usage was similar to that of modern times.

  2. CAT 2020 Question Paper Slot 1

    According to the passage, the modern currency system shares all the following features with that of the Tang, EXCEPT that:

    1. it uses different materials as currency.
    2. its currencies fluctuate in value over time. 
    3. it uses different currencies for different situations.
    4. it is undergoing transformation.
    Choice D
    it is undergoing transformation.

  3. CAT 2020 Question Paper Slot 1

    When discussing textiles as currency in the Tang period, the author uses the words “steady” and “stable” to indicate all of the following EXCEPT:

    1. reliable supply.
    2. reliable measurements.
    3. reliable quality.
    4. reliable transportation.
    Choice D
    reliable transportation.

  4. CAT 2020 Question Paper Slot 1

    During the Tang period, which one of the following would not be an economically sound decision for a small purchase in the local market that is worth one-eighth of a bolt of cloth?

    1. Cutting one-eighth of the fabric from a new bolt to pay the amount.
    2. Making the payment with the appropriate weight of grain.
    3. Using coins issued by the government to make the payment.
    4. Paying with a faded bolt of cloth that has approximately the same value.
    Choice A
    Cutting one-eighth of the fabric from a new bolt to pay the amount.

The passage below is accompanied by a set of questions. Choose the best answer to each question.

Vocabulary used in speech or writing organizes itself in seven parts of speech (eight, if you count interjections such as Oh! and Gosh! and Fuhgeddaboudit!). Communication composed of these parts of speech must be organized by rules of grammar upon which we agree. When these rules break down, confusion and misunderstanding result. Bad grammar produces bad sentences. My favorite example from Strunk and White is this one: “As a mother of five, with another one on the way, my ironing board is always up.”

Nouns and verbs are the two indispensable parts of writing. Without one of each, no group of words can be a sentence, since a sentence is, by definition, a group of words containing a subject (noun) and a predicate (verb); these strings of words begin with a capital letter, end with a period, and combine to make a complete thought which starts in the writer’s head and then leaps to the reader’s.

Must you write complete sentences each time, every time? Perish the thought. If your work consists only of fragments and floating clauses, the Grammar Police aren’t going to come and take you away. Even William Strunk, that Mussolini of rhetoric, recognized the delicious pliability of language. “It is an old observation,” he writes, “that the best writers sometimes disregard the rules of rhetoric.” Yet he goes on to add this thought, which I urge you to consider: “Unless he is certain of doing well, [the writer] will probably do best to follow the rules.”

The telling clause here is Unless he is certain of doing well. If you don’t have a rudimentary grasp of how the parts of speech translate into coherent sentences, how can you be certain that you are doing well? How will you know if you’re doing ill, for that matter? The answer, of course, is that you can’t, you won’t. One who does grasp the rudiments of grammar finds a comforting simplicity at its heart, where there need be only nouns, the words that name, and verbs, the words that act.

Take any noun, put it with any verb, and you have a sentence. It never fails. Rocks explode. Jane transmits. Mountains float. These are all perfect sentences. Many such thoughts make little rational sense, but even the stranger ones (Plums deify!) have a kind of poetic weight that’s nice. The simplicity of noun-verb construction is useful—at the very least it can provide a safety net for your writing. Strunk and White caution against too many simple sentences in a row, but simple sentences provide a path you can follow when you fear getting lost in the tangles of rhetoric—all those restrictive and nonrestrictive clauses, those modifying phrases, those appositives and compound-complex sentences. If you start to freak out at the sight of such unmapped territory (unmapped by you, at least), just remind yourself that rocks explode, Jane transmits, mountains float, and plums deify. Grammar is . . . the pole you grab to get your thoughts up on their feet and walking.

  1. CAT 2020 Question Paper Slot 1

    Which one of the following quotes best captures the main concern of the passage?

    1. “Nouns and verbs are the two indispensable parts of writing. Without one of each, no group of words can be a sentence . . .”
    2. “Strunk and White caution against too many simple sentences in a row, but simple sentences provide a path you can follow when you fear getting lost in the tangles of rhetoric . . .”
    3. “The telling clause here is Unless he is certain of doing well.”
    4. “Bad grammar produces bad sentences.”
    Choice D
    “Bad grammar produces bad sentences.”

  2. CAT 2020 Question Paper Slot 1

    Which one of the following statements, if false, could be seen as supporting the arguments in the passage?

    1. An understanding of grammar helps a writer decide if she/he is writing well or not.
    2. Perish the thought that complete sentences necessarily need nouns and verbs!
    3. It has been observed that writers sometimes disregard the rules of rhetoric.
    4. Regarding grammar, women writers tend to be more attentive to method and accuracy.
    Choice B
    Perish the thought that complete sentences necessarily need nouns and verbs!

  3. CAT 2020 Question Paper Slot 1

    All of the following statements can be inferred from the passage EXCEPT that:

    1. the subject–predicate relation is the same as the noun–verb relation.
    2. the primary purpose of grammar is to ensure that sentences remain simple.
    3. sentences do not always have to be complete.
    4. “Grammar Police” is a metaphor for critics who focus on linguistic rules.
    Choice B
    the primary purpose of grammar is to ensure that sentences remain simple.

  4. CAT 2020 Question Paper Slot 1

    “Take any noun, put it with any verb, and you have a sentence. It never fails. Rocks explode. Jane transmits. Mountains float.” None of the following statements can be seen as similar EXCEPT:

    1. A collection of people with the same sports equipment is a sports team.
    2. Take an apple tree, plant it in a field, and you have an orchard.
    3. A group of nouns arranged in a row becomes a sentence.
    4. Take any vegetable, put some spices in it, and you have a dish.
    Choice D
    Take any vegetable, put some spices in it, and you have a dish.

  5. CAT 2020 Question Paper Slot 1

    Inferring from the passage, the author could be most supportive of which one of the following practices?

    1. A Creative Writing course that focuses on how to avoid the use of rhetoric.
    2. The critique of standardised rules of punctuation and capitalisation.
    3. A campaign demanding that a writer’s creative license should allow the breaking of grammatical rules.
    4. The availability of language software that will standardise the rules of grammar as an aid to writers.
    Choice D
    The availability of language software that will standardise the rules of grammar as an aid to writers.

CAT VARC : CAT 2020 Question Paper Slot 2

The passage below is accompanied by a set of questions. Choose the best answer to each question.

The claims advanced here may be condensed into two assertions: [first, that visual] culture is what images, acts of seeing, and attendant intellectual, emotional, and perceptual sensibilities do to build, maintain, or transform the worlds in which people live. [And second, that the] study of visual culture is the analysis and interpretation of images and the ways of seeing (or gazes) that configure the agents, practices, conceptualities, and institutions that put images to work. . . .

Accordingly, the study of visual culture should be characterized by several concerns. First, scholars of visual culture need to examine any and all imagery – high and low, art and nonart. . . . They must not restrict themselves to objects of a particular beauty or aesthetic value. Indeed, any kind of imagery may be found to offer up evidence of the visual construction of reality. . . . 

Second, the study of visual culture must scrutinize visual practice as much as images themselves, asking what images do when they are put to use. If scholars engaged in this enterprise inquire what makes an image beautiful or why this image or that constitutes a masterpiece or a work of genius, they should do so with the purpose of investigating an artist’s or a work’s contribution to the experience of beauty, taste, value, or genius. No amount of social analysis can account fully for the existence of Michelangelo or Leonardo. They were unique creators of images that changed the way their contemporaries thought and felt and have continued to shape the history of art, artists, museums, feeling, and aesthetic value. But study of the critical, artistic, and popular reception of works by such artists as Michelangelo and Leonardo can shed important light on the meaning of these artists and their works for many different people. And the history of meaning-making has a great deal to do with how scholars as well as lay audiences today understand these artists and their achievements. 

Third, scholars studying visual culture might properly focus their interpretative work on lifeworlds by examining images, practices, visual technologies, taste, and artistic style as constitutive of social relations. The task is to understand how artifacts contribute to the construction of a world. . . . Important methodological implications follow: ethnography and reception studies become productive forms of gathering information, since these move beyond the image as a closed and fixed meaning-event. . . .

Fourth, scholars may learn a great deal when they scrutinize the constituents of vision, that is, the structures of perception as a physiological process as well as the epistemological frameworks informing a system of visual representation. Vision is a socially and a biologically constructed operation, depending on the design of the human body and how it engages the interpretive devices developed by a culture in order to see intelligibly. . . . Seeing . . . operates on the foundation of covenants with images that establish the conditions for meaningful visual experience. 

Finally, the scholar of visual culture seeks to regard images as evidence for explanation, not as epiphenomena.  

  1. CAT 2020 Question Paper Slot 2

    Which one of the following best describes the word “epiphenomena” in the last sentence of the passage?

    1. Phenomena amenable to analysis.
    2. Phenomena supplemental to the evidence.
    3. Overarching collections of images.
    4. Visual phenomena of epic proportions.
    Choice B
    Phenomena supplemental to the evidence.

  2. CAT 2020 Question Paper Slot 2

    All of the following statements may be considered valid inferences from the passage, EXCEPT:

    1. artifacts are meaningful precisely because they help to construct the meanings of the world for us.
    2. studying visual culture requires institutional structures without which the structures of perception cannot be analysed.
    3. understanding the structures of perception is an important part of understanding how visual cultures work.
    4. visual culture is not just about how we see, but also about how our visual practices can impact and change the world.
    Choice B
    studying visual culture requires institutional structures without which the structures of perception cannot be analysed.

  3. CAT 2020 Question Paper Slot 2

    “No amount of social analysis can account fully for the existence of Michelangelo or Leonardo.” In light of the passage, which one of the following interpretations of this sentence is the most accurate?

    1. Social analytical accounts of people like Michelangelo or Leonardo cannot explain their genius.
    2. Michelangelo or Leonardo cannot be subjected to social analysis because of their genius.
    3. No analyses exist of Michelangelo’s or Leonardo’s social accounts.
    4. Socially existing beings cannot be analysed, unlike the art of Michelangelo or Leonardo which can.
    Choice A
    Social analytical accounts of people like Michelangelo or Leonardo cannot explain their genius.

  4. CAT 2020 Question Paper Slot 2

    “Seeing . . . operates on the foundation of covenants with images that establish the conditions for meaningful visual experience.” In light of the passage, which one of the following statements best conveys the meaning of this sentence?

    1. Sight becomes a meaningful visual experience because of covenants of meaningfulness that we establish with the images we see.
    2. The way we experience sight is through images operated on by meaningful covenants.
    3. Images are meaningful visual experiences when they have a foundation of covenants seeing them.
    4. Sight as a meaningful visual experience is possible when there is a foundational condition established in images of covenants.
    Choice A
    Sight becomes a meaningful visual experience because of covenants of meaningfulness that we establish with the images we see.

  5. CAT 2020 Question Paper Slot 2

    Which set of keywords below most closely captures the arguments of the passage?

    1. Visual Culture, Aesthetic Value, Lay Audience, Visual Experience.
    2. Scholars, Social Analysis, Michelangelo and Leonardo, Interpretive Devices.
    3. Imagery, Visual Practices, Lifeworlds, Structures of Perception.
    4. Visual Construction of Reality, Work of Genius, Ethnography, Epiphenomena.
    Choice C
    Imagery, Visual Practices, Lifeworlds, Structures of Perception.

The passage below is accompanied by a set of questions. Choose the best answer to each question.

174 incidents of piracy were reported to the International Maritime Bureau last year, with Somali pirates responsible for only three. The rest ranged from the discreet theft of coils of rope in the Yellow Sea to the notoriously ferocious Nigerian gunmen attacking and hijacking oil tankers in the Gulf of Guinea, as well as armed robbery off Singapore and the Venezuelan coast and kidnapping in the Sundarbans in the Bay of Bengal. For [Dr. Peter] Lehr, an expert on modern-day piracy, the phenomenon’s history should be a source of instruction rather than entertainment, piracy past offering lessons for piracy present. . . . 

But . . . where does piracy begin or end? According to St Augustine, a corsair captain once told Alexander the Great that in the forceful acquisition of power and wealth at sea, the difference between an emperor and a pirate was simply one of scale. By this logic, European empire-builders were the most successful pirates of all time. A more eclectic history might have included the conquistadors, Vasco da Gama and the East India Company. But Lehr sticks to the disorganised small fry, making comparisons with the renegades of today possible. 

The main motive for piracy has always been a combination of need and greed. Why toil away as a starving peasant in the 16th century when a successful pirate made up to £4,000 on each raid? Anyone could turn to freebooting if the rewards were worth the risk . . . .

Increased globalisation has done more to encourage piracy than suppress it. European colonialism weakened delicate balances of power, leading to an influx of opportunists on the high seas. A rise in global shipping has meant rich pickings for freebooters. Lehr writes: “It quickly becomes clear that in those parts of the world that have not profited from globalisation and modernisation, and where abject poverty and the daily struggle for survival are still a reality, the root causes of piracy are still the same as they were a couple of hundred years ago.” . . . 

Modern pirate prevention has failed. After the French yacht Le Gonant was ransomed for $2 million in 2008, opportunists from all over Somalia flocked to the coast for a piece of the action. . . . A consistent rule, even today, is there are never enough warships to patrol pirate-infested waters. Such ships are costly and only solve the problem temporarily; Somali piracy is bound to return as soon as the warships are withdrawn. Robot shipping, eliminating hostages, has been proposed as a possible solution; but as Lehr points out, this will only make pirates switch their targets to smaller carriers unable to afford the technology.

His advice isn’t new. Proposals to end illegal fishing are often advanced but they are difficult to enforce. Investment in local welfare put a halt to Malaysian piracy in the 1970s, but was dependent on money somehow filtering through a corrupt bureaucracy to the poor on the periphery. Diplomatic initiatives against piracy are plagued by mutual distrust: the Russians execute pirates, while the EU and US are reluctant to capture them for fear they’ll claim asylum. 

  1. CAT 2020 Question Paper Slot 2

    “A more eclectic history might have included the conquistadors, Vasco da Gama and the East India Company. But Lehr sticks to the disorganised small fry . . .” From this statement we can infer that the author believes that:

    1. colonialism should be considered an organised form of piracy.
    2. the disorganised piracy of today is no match for the organised piracy of the past.
    3. Lehr does not assign adequate blame to empire builders for their past deeds.
    4. Vasco da Gama and the East India Company laid the ground for modern piracy.   
    Choice A
    colonialism should be considered an organised form of piracy.

  2. CAT 2020 Question Paper Slot 2

    We can deduce that the author believes that piracy can best be controlled in the long run:

    1. through international cooperation in enforcing stringent deterrents.
    2. if we eliminate poverty and income disparities in affected regions.
    3. through the extensive deployment of technology to track ships and cargo. 
    4. through lucrative welfare schemes to improve the lives of people in affected regions.
    Choice B
    if we eliminate poverty and income disparities in affected regions.

  3. CAT 2020 Question Paper Slot 2

    “Why toil away as a starving peasant in the 16th century when a successful pirate made up to £4,000 on each raid?” In this sentence, the author’s tone can best be described as being:

    1. facetious, about the hardships of peasant life in medieval England.
    2. ironic, about the reasons why so many took to piracy in medieval times.
    3. analytical, to explain the contrasts between peasant and pirate life in medieval England.
    4. indignant, at the scale of wealth successful pirates could amass in medieval times. 
    Choice B
    ironic, about the reasons why so many took to piracy in medieval times.

  4. CAT 2020 Question Paper Slot 2

    The author ascribes the rise in piracy today to all of the following factors EXCEPT:

    1. the growth in international shipping with globalisation.
    2. colonialism’s disruption of historic ties among countries. 
    3. the high rewards via ransoms for successful piracy attempts. 
    4. decreased surveillance of the high seas.
    Choice D
    decreased surveillance of the high seas.

The passage below is accompanied by a set of questions. Choose the best answer to each question.

In a low-carbon world, renewable energy technologies are hot business. For investors looking to redirect funds, wind turbines and solar panels, among other technologies, seem a straightforward choice. But renewables need to be further scrutinized before being championed as forging a path toward a low-carbon future. Both the direct and indirect impacts of renewable energy must be examined to ensure that a climate-smart future does not intensify social and environmental harm. As renewable energy production requires land, water, and labor, among other inputs, it imposes costs on people and the environment. Hydropower projects, for instance, have led to community dispossession and exclusion . . . Renewable energy supply chains are also intertwined with mining, and their technologies contribute to growing levels of electronic waste . . . Furthermore, although renewable energy can be produced and distributed through small-scale, local systems, such an approach might not generate the high returns on investment needed to attract capital.

Although an emerging sector, renewables are enmeshed in long-standing resource extraction through their dependence on minerals and metals . . . Scholars document the negative consequences of mining . . . even for mining operations that commit to socially responsible practices[:] “many of the world’s largest reservoirs of minerals like cobalt, copper, lithium, [and] rare earth minerals”—the ones needed for renewable technologies—“are found in fragile states and under communities of marginalized peoples in Africa, Asia, and Latin America.” Since the demand for metals and minerals will increase substantially in a renewable-powered future . . . this intensification could exacerbate the existing consequences of extractive activities.

Among the connections between climate change and waste, O’Neill . . . highlights that “devices developed to reduce our carbon footprint, such as lithium batteries for hybrid and electric cars or solar panels[,] become potentially dangerous electronic waste at the end of their productive life.” The disposal of toxic waste has long perpetuated social injustice through the flows of waste to the Global South and to marginalized communities in the Global North . . .

While renewable energy is a more recent addition to financial portfolios, investments in the sector must be considered in light of our understanding of capital accumulation. As agricultural finance reveals, the concentration of control of corporate activity facilitates profit generation. For some climate activists, the promise of renewables rests on their ability not only to reduce emissions but also to provide distributed, democratized access to energy . . . But Burke and Stephens . . . caution that “renewable energy systems offer a possibility but not a certainty for more democratic energy futures.” Small-scale, distributed forms of energy are only highly profitable to institutional investors if control is consolidated somewhere in the financial chain. Renewable energy can be produced at the household or neighborhood level. However, such small-scale, localized production is unlikely to generate high returns for investors. For financial growth to be sustained and expanded by the renewable sector, production and trade in renewable energy technologies will need to be highly concentrated, and large asset management firms will likely drive those developments.

  1. CAT 2020 Question Paper Slot 2

    All of the following statements, if true, could be seen as supporting the arguments in the passage, EXCEPT:

    1. Marginalised people in Africa, Asia and Latin America have often been the main sufferers of corporate mineral extraction projects.
    2. The example of agricultural finance helps us to see how to concentrate corporate activity in the renewable energy sector.
    3. One reason for the perpetuation of social injustice lies in the problem of the disposal of toxic waste.
    4. The possible negative impacts of renewable energy need to be studied before it can be offered as a financial investment opportunity.
    Choice D
    The possible negative impacts of renewable energy need to be studied before it can be offered as a financial investment opportunity.

  2. CAT 2020 Question Paper Slot 2

    Which one of the following statements, if false, could be seen as best supporting the arguments in the passage?

    1. Renewable energy systems are not as profitable as non-renewable energy systems.
    2. Renewable energy systems are as expensive as non-renewable energy systems.
    3. The production and distribution of renewable energy through small-scale, local systems is not economically sustainable.
    4. Renewable energy systems have little or no environmental impact.
    Choice D
    Renewable energy systems have little or no environmental impact.

  3. CAT 2020 Question Paper Slot 2

    Which one of the following statements, if true, could be an accurate inference from the first paragraph of the passage?

    1. The author’s only reservation is about the profitability of renewable energy systems.
    2. The author does not think renewable energy systems can be as efficient as non-renewable energy systems.
    3. The author has reservations about the consequences of non-renewable energy systems.
    4. The author has reservations about the consequences of renewable energy systems.
    Choice D
    The author has reservations about the consequences of renewable energy systems.

  4. CAT 2020 Question Paper Slot 2

    Which one of the following statements best captures the main argument of the last paragraph of the passage?

    1. Renewable energy systems are not democratic unless they are corporate-controlled.
    2. Renewable energy produced at the household or neighbourhood level is more efficient than mass-produced forms of energy.
    3. The development of the renewable energy sector is a double-edged sword.
    4. Most forms of renewable energy are not profitable investments for institutional investors.
    Choice C
    The development of the renewable energy sector is a double-edged sword.

  5. CAT 2020 Question Paper Slot 2

    Based on the passage, we can infer that the author would be most supportive of which one of the following practices?

    1. The study of the coexistence of marginalised people with their environments.
    2. Encouragement for the development of more environment-friendly carbon-based fuels.
    3. More stringent global policies and regulations to ensure a more just system of toxic waste disposal.
    4. The localised, small-scale development of renewable energy systems.
    Choice C
    More stringent global policies and regulations to ensure a more just system of toxic waste disposal.

The passage below is accompanied by a set of questions. Choose the best answer to each question.

Aggression is any behavior that is directed toward injuring, harming, or inflicting pain on another living being or group of beings. Generally, the victim(s) of aggression must wish to avoid such behavior in order for it to be considered true aggression. Aggression is also categorized according to its ultimate intent. Hostile aggression is an aggressive act that results from anger, and is intended to inflict pain or injury because of that anger. Instrumental aggression is an aggressive act that is regarded as a means to an end other than pain or injury. For example, an enemy combatant may be subjected to torture in order to extract useful intelligence, though those inflicting the torture may have no real feelings of anger or animosity toward their subject. The concept of aggression is very broad, and includes many categories of behavior (e.g., verbal aggression, street crime, child abuse, spouse abuse, group conflict, war, etc.). A number of theories and models of aggression have arisen to explain these diverse forms of behavior, and these theories/models tend to be categorized according to their specific focus. The most common system of categorization groups the various approaches to aggression into three separate areas, based upon the three key variables that are present whenever any aggressive act or set of acts is committed. The first variable is the aggressor him/herself. The second is the social situation or circumstance in which the aggressive act(s) occur. The third variable is the target or victim of aggression.

Regarding theories and research on the aggressor, the fundamental focus is on the factors that lead an individual (or group) to commit aggressive acts. At the most basic level, some argue that aggressive urges and actions are the result of inborn, biological factors. Sigmund Freud (1930) proposed that all individuals are born with a death instinct that predisposes us to a variety of aggressive behaviors, including suicide (self directed aggression) and mental illness (possibly due to an unhealthy or unnatural suppression of aggressive urges). Other influential perspectives supporting a biological basis for aggression conclude that humans evolved with an abnormally low neural inhibition of aggressive impulses (in comparison to other species), and that humans possess a powerful instinct for property accumulation and territorialism. It is proposed that this instinct accounts for hostile behaviors ranging from minor street crime to world wars. Hormonal factors also appear to play a significant role in fostering aggressive tendencies. For example, the hormone testosterone has been shown to increase aggressive behaviors when injected into animals. Men and women convicted of violent crimes also possess significantly higher levels of testosterone than men and women convicted of non violent crimes. Numerous studies comparing different age groups, racial/ethnic groups, and cultures also indicate that men, overall, are more likely to engage in a variety of aggressive behaviors (e.g., sexual assault, aggravated assault, etc.) than women. One explanation for higher levels of aggression in men is based on the assumption that, on average, men have higher levels of testosterone than women.

  1. CAT 2020 Question Paper Slot 2

    “[A]n enemy combatant may be subjected to torture in order to extract useful intelligence, though those inflicting the torture may have no real feelings of anger or animosity toward their subject.” Which one of the following best explicates the larger point being made by the author here?

    1. In certain kinds of aggression, inflicting pain is not the objective, and is no more than a utilitarian means to achieve another end.
    2. When an enemy combatant refuses to reveal information, the use of torture can sometimes involve real feelings of hostility.
    3. Information revealed by subjecting an enemy combatant to torture is not always reliable because of the animosity involved.
    4. The use of torture to extract information is most effective when the torturer is not emotionally involved in the torture.
    Choice A
    In certain kinds of aggression, inflicting pain is not the objective, and is no more than a utilitarian means to achieve another end.

  2. CAT 2020 Question Paper Slot 2

    All of the following statements can be seen as logically implied by the arguments of the passage EXCEPT:

    1. Freud’s theory of aggression proposes that aggression results from the suppression of aggressive urges.
    2. the Freudian theory of suicide as self-inflicted aggression implies that an aggressive act need not be sought to be avoided in order for it to be considered aggression.
    3. a common theory of aggression is that it is the result of an abnormally low neural regulation of testosterone.
    4. if the alleged aggressive act is not sought to be avoided, it cannot really be considered aggression.
    Choice C
    a common theory of aggression is that it is the result of an abnormally low neural regulation of testosterone.

  3. CAT 2020 Question Paper Slot 2

    The author identifies three essential factors according to which theories of aggression are most commonly categorised. Which of the following options is closest to the factors identified by the author?

    1. Extreme – Moderate – Mild.
    2. Hostile – Instrumental – Hormonal.
    3. Aggressor – Circumstances of aggression – Victim.
    4. Psychologically – Sociologically – Medically.
    Choice C
    Aggressor – Circumstances of aggression – Victim.

  4. CAT 2020 Question Paper Slot 2

    The author discusses all of the following arguments in the passage EXCEPT that:

    1. aggression in most societies is kept under control through moderating the death instinct identified by Freud.
    2. the nature of aggression can vary depending on several factors, including intent.
    3. men in general are believed to be more hormonally driven to exhibit violence than women.
    4. several studies indicate that aggression may have roots in the biological condition of humanity.
    Choice A
    aggression in most societies is kept under control through moderating the death instinct identified by Freud.

CAT VARC : CAT 2020 Question Paper Slot 3

The passage below is accompanied by a set of questions. Choose the best answer to each question.

Mode of transportation affects the travel experience and thus can produce new types of travel writing and perhaps even new “identities.” Modes of transportation determine the types and duration of social encounters; affect the organization and passage of space and time; . . . and also affect perception and knowledge—how and what the traveler comes to know and write about. The completion of the first U.S. transcontinental highway during the 1920s . . . for example, inaugurated a new genre of travel literature about the United States—the automotive or road narrative. Such narratives highlight the experiences of mostly male protagonists “discovering themselves” on their journeys, emphasizing the independence of road travel and the value of rural folk traditions.

Travel writing’s relationship to empire building— as a type of “colonialist discourse”—has drawn the most attention from academicians. Close connections have been observed between European (and American) political, economic, and administrative goals for the colonies and their manifestations in the cultural practice of writing travel books. Travel writers’ descriptions of foreign places have been analyzed as attempts to validate, promote, or challenge the ideologies and practices of colonial or imperial domination and expansion. Mary Louise Pratt’s study of the genres and conventions of 18th- and 19th-century exploration narratives about South America and Africa (e.g., the “monarch of all I survey” trope) offered ways of thinking about travel writing as embedded within relations of power between metropole and periphery, as did Edward Said’s theories of representation and cultural imperialism. Particularly Said’s book, Orientalism, helped scholars understand ways in which representations of people in travel texts were intimately bound up with notions of self, in this case, that the Occident defined itself through essentialist, ethnocentric, and racist representations of the Orient. Said’s work became a model for demonstrating cultural forms of imperialism in travel texts, showing how the political, economic, or administrative fact of dominance relies on legitimating discourses such as those articulated through travel writing. . . .

Feminist geographers’ studies of travel writing challenge the masculinist history of geography by questioning who and what are relevant subjects of geographic study and, indeed, what counts as geographic knowledge itself. Such questions are worked through ideological constructs that posit men as explorers and women as travelers—or, conversely, men as travelers and women as tied to the home. Studies of Victorian women who were professional travel writers, tourists, wives of colonial administrators, and other (mostly) elite women who wrote narratives about their experiences abroad during the 19th century have been particularly revealing. From a “liberal” feminist perspective, travel presented one means toward female liberation for middle- and upper-class Victorian women. Many studies from the 1970s onward demonstrated the ways in which women’s gendered identities were negotiated differently “at home” than they were “away,” thereby showing women’s self-development through travel. The more recent poststructural turn in studies of Victorian travel writing has focused attention on women’s diverse and fragmented identities as they narrated their travel experiences, emphasizing women’s sense of themselves as women in new locations, but only as they worked through their ties to nation, class, whiteness, and colonial and imperial power structures.

  1. CAT 2020 Question Paper Slot 3

    From the passage, we can infer that feminist scholars’ understanding of the experiences of Victorian women travellers is influenced by all of the following EXCEPT scholars':

    1. awareness of the ways in which identity is formed.
    2. perspective that they bring to their research.
    3. knowledge of class tensions in Victorian society.
    4. awareness of gender issues in Victorian society.
    Choice C
    knowledge of class tensions in Victorian society.

  2. CAT 2020 Question Paper Slot 3

    From the passage, we can infer that travel writing is most similar to:

    1. political journalism.
    2. historical fiction.
    3. autobiographical writing.
    4. feminist writing.
    Choice C
    autobiographical writing.

  3. CAT 2020 Question Paper Slot 3

    From the passage, it can be inferred that scholars argue that Victorian women experienced self-development through their travels because:

    1. their identity was redefined when they were away from home.
    2. they were from the progressive middle- and upper-classes of society.
    3. they were on a quest to discover their diverse identities.
    4. they developed a feminist perspective of the world.
    Choice A
    their identity was redefined when they were away from home.

  4. CAT 2020 Question Paper Slot 3

    American travel literature of the 1920s:

    1. celebrated the freedom that travel gives.
    2. developed the male protagonists’ desire for independence.
    3. presented travellers’ discovery of their identity as different from others.
    4. showed participation in local traditions.
    Choice A
    celebrated the freedom that travel gives.

  5. CAT 2020 Question Paper Slot 3

    According to the passage, Said’s book, “Orientalism”:

    1. explained the difference between the representation of people and the actual fact.
    2. demonstrated how cultural imperialism was used to justify colonial domination.
    3. argued that cultural imperialism was more significant than colonial domination.
    4. illustrated how narrow minded and racist westerners were.
    Choice B
    demonstrated how cultural imperialism was used to justify colonial domination.

The passage below is accompanied by a set of questions. Choose the best answer to each question.

Although one of the most contested concepts in political philosophy, human nature is something on which most people seem to agree. By and large, according to Rutger Bregman in his new book Humankind, we have a rather pessimistic view – not of ourselves exactly, but of everyone else. We see other people as selfish, untrustworthy and dangerous and therefore we behave towards them with defensiveness and suspicion. This was how the 17th-century philosopher Thomas Hobbes conceived our natural state to be, believing that all that stood between us and violent anarchy was a strong state and firm leadership.

But in following Hobbes, argues Bregman, we ensure that the negative view we have of human nature is reflected back at us. He instead puts his faith in Jean-Jacques Rousseau, the 18th-century French thinker, who famously declared that man was born free and it was civilisation – with its coercive powers, social classes and restrictive laws – that put him in chains.

Hobbes and Rousseau are seen as the two poles of the human nature argument and it’s no surprise that Bregman strongly sides with the Frenchman. He takes Rousseau’s intuition and paints a picture of a prelapsarian idyll in which, for the better part of 300,000 years, Homo sapiens lived a fulfilling life in harmony with nature . . . Then we discovered agriculture and for the next 10,000 years it was all property, war, greed and injustice. . . . 

It was abandoning our nomadic lifestyle and then domesticating animals, says Bregman, that brought about infectious diseases such as measles, smallpox, tuberculosis, syphilis, malaria, cholera and plague. This may be true, but what Bregman never really seems to get to grips with is that pathogens were not the only things that grew with agriculture – so did the number of humans. It’s one thing to maintain friendly relations and a property-less mode of living when you’re 30 or 40 hunter-gatherers following the food. But life becomes a great deal more complex and knowledge far more extensive when there are settlements of many thousands. 

“Civilisation has become synonymous with peace and progress and wilderness with war and decline,” writes Bregman. “In reality, for most of human existence, it was the other way around.” Whereas traditional history depicts the collapse of civilisations as “dark ages” in which everything gets worse, modern scholars, he claims, see them more as a reprieve, in which the enslaved gain their freedom and culture flourishes. Like much else in this book, the truth is probably somewhere between the two stated positions.

In any case, the fear of civilisational collapse, Bregman believes, is unfounded. It’s the result of what the Dutch biologist Frans de Waal calls “veneer theory” – the idea that just below the surface, our bestial nature is waiting to break out. . . . There’s a great deal of reassuring human decency to be taken from this bold and thought-provoking book and a wealth of evidence in support of the contention that the sense of who we are as a species has been deleteriously distorted. But it seems equally misleading to offer the false choice of Rousseau and Hobbes when, clearly, humanity encompasses both.

  1. CAT 2020 Question Paper Slot 3

    The author has differing views from Bregman regarding:

    1. the role of pathogens in the spread of infectious diseases.
    2. a civilised society being coercive and unjust.
    3. a property-less mode of living being socially harmonious.
    4. the role of agriculture in the advancement of knowledge.
    Choice B
    a civilised society being coercive and unjust.

  2. CAT 2020 Question Paper Slot 3

    According to the passage, the “collapse of civilisations” is viewed by Bregman as:

    1. a sign of regression in society’s trajectory. 
    2. resulting from a breakdown in the veneer of human nature.  
    3. a time that enables changes in societies and cultures.
    4. a temporary phase which can be rectified by social action.
    Choice C
    a time that enables changes in societies and cultures.

  3. CAT 2020 Question Paper Slot 3

    None of the following views is expressed in the passage EXCEPT that:

    1. Hobbes and Rousseau disagreed on the fundamental nature of humans, but both believed in the need for a strong state. 
    2. Bregman agrees with Hobbes that firm leadership is needed to ensure property rights and regulate strife.
    3. the author of the review believes in the veneer theory of human nature.   
    4. most people agree with Hobbes’ pessimistic view of human nature as being intrinsically untrustworthy and selfish.   
    Choice D
    most people agree with Hobbes’ pessimistic view of human nature as being intrinsically untrustworthy and selfish.

  4. CAT 2020 Question Paper Slot 3

    According to the author, the main reason why Bregman contrasts life in pre-agricultural societies with agricultural societies is to:

    1. advocate the promotion of less complex societies as a basis for greater security and prosperity.
    2. bolster his argument that people are basically decent, but progress as we know it can make them selfish.
    3. highlight the enormous impact that settled farming had on population growth.
    4. make the argument that an environmentally conscious lifestyle is a more harmonious way of living.
    Choice C
    bolster his argument that people are basically decent, but progress as we know it can make them selfish.

The passage below is accompanied by a set of questions. Choose the best answer to each question.

[There is] a curious new reality: Human contact is becoming a luxury good. As more screens appear in the lives of the poor, screens are disappearing from the lives of the rich. The richer you are, the more you spend to be off-screen. . . .  

The joy — at least at first — of the internet revolution was its democratic nature. Facebook is the same Facebook whether you are rich or poor. Gmail is the same Gmail. And it’s all free. There is something mass market and unappealing about that. And as studies show that time on these advertisement-support platforms is unhealthy, it all starts to seem déclassé, like drinking soda or smoking cigarettes, which wealthy people do less than poor people.  The wealthy can afford to opt out of having their data and their attention sold as a product. The poor and middle class don’t have the same kind of resources to make that happen.

Screen exposure starts young. And children who spent more than two hours a day looking at a screen got lower scores on thinking and language tests, according to early results of a landmark study on brain development of more than 11,000 children that the National Institutes of Health is supporting. Most disturbingly, the study is finding that the brains of children who spend a lot of time on screens are different. For some kids, there is premature thinning of their cerebral cortex. In adults, one study found an association between screen time and depression. . . .

Tech companies worked hard to get public schools to buy into programs that required schools to have one laptop per student, arguing that it would better prepare children for their screen-based future. But this idea isn’t how the people who actually build the screen-based future raise their own children. In Silicon Valley, time on screens is increasingly seen as unhealthy. Here, the popular elementary school is the local Waldorf School, which promises a back-to-nature, nearly screen-free education. So as wealthy kids are growing up with less screen time, poor kids are growing up with more. How comfortable someone is with human engagement could become a new class marker.

Human contact is, of course, not exactly like organic food . . . . But with screen time, there has been a concerted effort on the part of Silicon Valley behemoths to confuse the public. The poor and the middle class are told that screens are good and important for them and their children. There are fleets of psychologists and neuroscientists on staff at big tech companies working to hook eyes and minds to the screen as fast as possible and for as long as possible. And so human contact is rare. . . . 

There is a small movement to pass a “right to disconnect” bill, which would allow workers to turn their phones off, but for now a worker can be punished for going offline and not being available. There is also the reality that in our culture of increasing isolation, in which so many of the traditional gathering places and social structures have disappeared, screens are filling a crucial void.

  1. CAT 2020 Question Paper Slot 3

    Which of the following statements about the negative effects of screen time is the author least likely to endorse?

    1. It can cause depression in viewers.
    2. It increases human contact as it fills an isolation void.
    3. It is shown to have adverse effects on young children’s learning.
    4. It is designed to be addictive.
    Choice B
    It increases human contact as it fills an isolation void.

  2. CAT 2020 Question Paper Slot 3

    The statement “The richer you are, the more you spend to be off-screen” is supported by which other line from the passage?

    1. “. . . studies show that time on these advertisement-support platforms is unhealthy . . .”
    2. “Gmail is the same Gmail. And it’s all free.” 
    3. “How comfortable someone is with human engagement could become a new class marker.”
    4.  “. . . screens are filling a crucial void.”
    Choice C
    “How comfortable someone is with human engagement could become a new class marker.”

  3. CAT 2020 Question Paper Slot 3

    The author is least likely to agree with the view that the increase in screen-time is fuelled by the fact that:

    1. screens provide social contact in an increasingly isolating world.
    2. some workers face punitive action if they are not online.
    3. with falling costs, people are streaming more content on their devices.
    4. there is a growth in computer-based teaching in public schools.
    Choice C
    with falling costs, people are streaming more content on their devices.

  4. CAT 2020 Question Paper Slot 3

    The author claims that Silicon Valley tech companies have tried to “confuse the public” by:

    1. promoting screen time in public schools while opting for a screen-free education for their own children. 
    2. developing new work-efficiency programmes while lobbying for the “right to disconnect” bill.
    3. concealing the findings of psychologists and neuroscientists on screen-time use from the public.
    4. pushing for greater privacy while working with advertisement-support platforms to mine data.
    Choice A
    promoting screen time in public schools while opting for a screen-free education for their own children.

The passage below is accompanied by a set of questions. Choose the best answer to each question.

I’ve been following the economic crisis for more than two years now. I began working on the subject as part of the background to a novel, and soon realized that I had stumbled across the most interesting story I’ve ever found. While I was beginning to work on it, the British bank Northern Rock blew up, and it became clear that, as I wrote at the time, “If our laws are not extended to control the new kinds of super-powerful, super-complex, and potentially super-risky investment vehicles, they will one day cause a financial disaster of global-systemic proportions.” . . . I was both right and too late, because all the groundwork for the crisis had already been done—though the sluggishness of the world’s governments, in not preparing for the great unraveling of autumn 2008, was then and still is stupefying. But this is the first reason why I wrote this book: because what’s happened is extraordinarily interesting. It is an absolutely amazing story, full of human interest and drama, one whose byways of mathematics, economics, and psychology are both central to the story of the last decades and mysteriously unknown to the general public. We have heard a lot about “the two cultures” of science and the arts—we heard a particularly large amount about it in 2009, because it was the fiftieth anniversary of the speech during which C. P. Snow first used the phrase. But I’m not sure the idea of a huge gap between science and the arts is as true as it was half a century ago—it’s certainly true, for instance, that a general reader who wants to pick up an education in the fundamentals of science will find it easier than ever before. It seems to me that there is a much bigger gap between the world of finance and that of the general public and that there is a need to narrow that gap, if the financial industry is not to be a kind of priesthood, administering to its own mysteries and feared and resented by the rest of us. Many bright, literate people have no idea about all sorts of economic basics, of a type that financial insiders take as elementary facts of how the world works. I am an outsider to finance and economics, and my hope is that I can talk across that gulf.

My need to understand is the same as yours, whoever you are. That’s one of the strangest ironies of this story: after decades in which the ideology of the Western world was personally and economically individualistic, we’ve suddenly been hit by a crisis which shows in the starkest terms that whether we like it or not—and there are large parts of it that you would have to be crazy to like—we’re all in this together. The aftermath of the crisis is going to dominate the economics and politics of our societies for at least a decade to come and perhaps longer.

  1. CAT 2020 Question Paper Slot 3

    Which one of the following, if false, could be seen as supporting the author’s claims?

    1. The global economic crisis lasted for more than two years. 
    2. The huge gap between science and the arts has steadily narrowed over time.
    3. The economic crisis was not a failure of collective action to rectify economic problems.
    4. Most people are yet to gain any real understanding of the workings of the financial world.
    Choice C
    The economic crisis was not a failure of collective action to rectify economic problems.

  2. CAT 2020 Question Paper Slot 3

    Which one of the following, if true, would be an accurate inference from the first sentence of the passage?

    1. The author’s preoccupation with the economic crisis is not less than two years old.
    2. The economic crisis outlasted the author’s preoccupation with it.
    3. The author is preoccupied with the economic crisis because he is being followed.
    4. The author has witnessed many economic crises by travelling a lot for two years.
    Choice A
    The author’s preoccupation with the economic crisis is not less than two years old.

  3. CAT 2020 Question Paper Slot 3

    Which one of the following best captures the main argument of the last paragraph of the passage?

    1. Whoever you are, you would be crazy to think that there is no crisis.
    2. In the decades to come, other ideologies will emerge in the aftermath of the crisis.
    3. The ideology of individualism must be set aside in order to deal with the crisis.
    4. The aftermath of the crisis will strengthen the central ideology of individualism in the Western world.
    Choice C
    The ideology of individualism must be set aside in order to deal with the crisis.

  4. CAT 2020 Question Paper Slot 3

    All of the following, if true, could be seen as supporting the arguments in the passage, EXCEPT:

    1. The story of the economic crisis is also one about international relations, global financial security, and mass psychology.
    2. Economic crises could be averted by changing prevailing ideologies and beliefs.
    3. The failure of economic systems does not necessarily mean the failure of their ideologies.
    4. The difficulty with understanding financial matters is that they have become so arcane.
    Choice C
    The failure of economic systems does not necessarily mean the failure of their ideologies.

  5. CAT 2020 Question Paper Slot 3

    According to the passage, the author is likely to be supportive of which one of the following programmes?

    1. An educational curriculum that promotes economic research.
    2. An educational curriculum that promotes developing financial literacy in the masses.
    3. The complete nationalisation of all financial institutions.
    4. Economic policies that are more sensitively calibrated to the fluctuations of the market.
    Choice B
    An educational curriculum that promotes developing financial literacy in the masses.

CAT VARC : CAT 2019 Question Paper Slot 1

Tale of Aladdin

In the past, credit for telling the tale of Aladdin has often gone to Antoine Galland . . . the first European translator of . . . Arabian Nights [which] started as a series of translations of an incomplete manuscript of a medieval Arabic story collection. . . But, though those tales were of medieval origin, Aladdin may be a more recent invention. Scholars have not found a manuscript of the story that predates the version published in 1712 by Galland, who wrote in his diary that he first heard the tale from a Syrian storyteller from Aleppo named Hanna Diyab . . .

Despite the fantastical elements of the story, scholars now think the main character may actually be based on a real person’s real experiences. . . . Though Galland never credited Diyab in his published translations of the Arabian Nights stories, Diyab wrote something of his own: a travelogue penned in the mid-18th century. In it, he recalls telling Galland the story of Aladdin [and] describes his own hard-knocks upbringing and the way he marveled at the extravagance of Versailles. The descriptions he uses were very similar to the descriptions of the lavish palace that ended up in Galland’s version of the Aladdin story. [Therefore, author Paulo Lemos] Horta believes that “Aladdin might be the young Arab Maronite from Aleppo, marveling at the jewels and riches of Versailles.” . . .

For 300 years, scholars thought that the rags-to-riches story of Aladdin might have been inspired by the plots of French fairy tales that came out around the same time, or that the story was invented in that 18th century period as a byproduct of French Orientalism, a fascination with stereotypical exotic Middle Eastern luxuries that was prevalent then. The idea that Diyab might have based it on his own life — the experiences of a Middle Eastern man encountering the French, not vice-versa — flips the script. [According to Horta,] “Diyab was ideally placed to embody the overlapping world of East and West, blending the storytelling traditions of his homeland with his youthful observations of the wonder of 18th-century France.” . . .

To the scholars who study the tale, its narrative drama isn’t the only reason storytellers keep finding reason to return to Aladdin. It reflects not only “a history of the French and the Middle East, but also [a story about] Middle Easterners coming to Paris and that speaks to our world today,” as Horta puts it. “The day Diyab told the story of Aladdin to Galland, there were riots due to food shortages during the winter and spring of 1708 to 1709, and Diyab was sensitive to those people in a way that Galland is not. When you read this diary, you see this solidarity among the Arabs who were in Paris at the time. . . . There is little in the writings of Galland that would suggest that he was capable of developing a character like Aladdin with sympathy, but Diyab’s memoir reveals a narrator adept at capturing the distinctive psychology of a young protagonist, as well as recognizing the kinds of injustices and opportunities that can transform the path of any youthful adventurer.”

  1. CAT 2019 Question Paper - Slot 1

    All of the following serve as evidence for the character of Aladdin being based on Hanna Diyab EXCEPT:

    1. Diyab’s cosmopolitanism and cross-cultural experience.
    2. Diyab’s humble origins and class struggles, as recounted in his travelogue.
    3. Diyab’s description of the wealth of Versailles in his travelogue.
    4. Diyab’s narration of the original story to Galland.
    Choice D
    Diyab’s narration of the original story to Galland.

  2. CAT 2019 Question Paper - Slot 1

    The author of the passage is most likely to agree with which of the following explanations for the origins of the story of Aladdin?

    1. Galland derived the story of Aladdin from Diyab’s travelogue in which he recounts his fascination with the wealth of Versailles.
    2. The story of Aladdin has its origins in an undiscovered, incomplete manuscript of a medieval Arabic collection of stories.
    3. Basing it on his own life experiences, Diyab transmitted the story of Aladdin to Galland who included it in Arabian Nights.
    4. Galland received the story of Aladdin from Diyab who, in turn, found it in an incomplete medieval manuscript.
    Choice C
    Basing it on his own life experiences, Diyab transmitted the story of Aladdin to Galland who included it in Arabian Nights.

  3. CAT 2019 Question Paper - Slot 1

    Which of the following, if true, would invalidate the inversion that the phrase “flips the script” refers to?

    1. Diyab’s travelogue described the affluence of the French city of Bordeaux, instead of Versailles.
    2. The French fairy tales of the eighteenth century did not have rags-to-riches plot lines like that of the tale of Aladdin.
    3. The description of opulence in Hanna Diyab’s and Antoine Galland’s narratives bore no resemblance to each other.
    4. Galland acknowledged in the published translations of Arabian Nights that he heard the story of Aladdin from Diyab.
    Choice C
    The description of opulence in Hanna Diyab’s and Antoine Galland’s narratives bore no resemblance to each other.

  4. CAT 2019 Question Paper - Slot 1

    Which of the following is the primary reason for why storytellers are still fascinated by the story of Aladdin?

    1. The traveller's experience that inspired the tale of Aladdin resonates even today.
    2. The tale of Aladdin documents the history of Europe and Middle East.
    3. The archetype of the rags-to-riches story of Aladdin makes it popular even today.
    4. The story of Aladdin is evidence of the eighteenth century French Orientalist.
    Choice A
    The traveller's experience that inspired the tale of Aladdin resonates even today.

  5. CAT 2019 Question Paper - Slot 1

    Which of the following does not contribute to the passage’s claim about the authorship of Aladdin?

    1. The narrative sensibility of Diyab’s travelogue.
    2. The depiction of the affluence of Versailles in Diyab’s travelogue.
    3. Galland’s acknowledgment of Diyab in his diary.
    4. The story-line of many French fairy tales of the 18th century.
    Choice D
    The story-line of many French fairy tales of the 18th century.

Choice Fatigue

Contemporary internet shopping conjures a perfect storm of choice anxiety. Research has consistently held that people who are presented with a few options make better, easier decisions than those presented with many. . . . Helping consumers figure out what to buy amid an endless sea of choice online has become a cottage industry unto itself. Many brands and retailers now wield marketing buzzwords such as curation, differentiation, and discovery as they attempt to sell an assortment of stuff targeted to their ideal customer. Companies find such shoppers through the data gold mine of digital advertising, which can catalog people by gender, income level, personal interests, and more. Since Americans have lost the ability to sort through the sheer volume of the consumer choices available to them, a ghost now has to be in the retail machine, whether it’s an algorithm, an influencer, or some snazzy ad tech to help a product follow you around the internet. Indeed, choice fatigue is one reason so many people gravitate toward lifestyle influencers on Instagram—the relentlessly chic young moms and perpetually vacationing 20-somethings—who present an aspirational worldview, and then recommend the products and services that help achieve it. . . .

For a relatively new class of consumer-products start-ups, there’s another method entirely. Instead of making sense of a sea of existing stuff, these companies claim to disrupt stuff as Americans know it. Casper (mattresses), Glossier (makeup), Away (suitcases), and many others have sprouted up to offer consumers freedom from choice: The companies have a few aesthetically pleasing and supposedly highly functional options, usually at mid-range prices. They’re selling nice things, but maybe more importantly, they’re selling a confidence in those things, and an ability to opt out of the stuff rat race. . . .

One-thousand-dollar mattresses and $300 suitcases might solve choice anxiety for a certain tier of consumer, but the companies that sell them, along with those that attempt to massage the larger stuff economy into something navigable, are still just working within a consumer market that’s broken in systemic ways. The presence of so much stuff in America might be more valuable if it were more evenly distributed, but stuff’s creators tend to focus their energy on those who already have plenty. As options have expanded for people with disposable income, the opportunity to buy even basic things such as fresh food or quality diapers has contracted for much of America’s lower classes.

For start-ups that promise accessible simplicity, their very structure still might eventually push them toward overwhelming variety. Most of these companies are based on hundreds of millions of dollars of venture capital, the investors of which tend to expect a steep growth rate that can’t be achieved by selling one great mattress or one great sneaker. Casper has expanded into bedroom furniture and bed linens. Glossier, after years of marketing itself as no-makeup makeup that requires little skill to apply, recently launched a full line of glittering color cosmetics. There may be no way to opt out of stuff by buying into the right thing.

  1. CAT 2019 Question Paper - Slot 1

    Which of the following hypothetical statements would add the least depth to the author’s prediction of the fate of start-ups offering few product options?

    1. An exponential surge in their sales enables start-ups to meet their desired profit goals without expanding their product catalogue
    2. With the motive of promoting certain rival companies, the government decides to double the tax-rates for these start-ups.
    3. With Casper and Glossier venturing into new product ranges, their regular customers start losing trust in the companies and their products.
    4. Start-ups with few product options are no exception to the American consumer market that is deeply divided along class lines.
    Choice A
    An exponential surge in their sales enables start-ups to meet their desired profit goals without expanding their product catalogue

  2. CAT 2019 Question Paper - Slot 1

    Which one of the following best sums up the overall purpose of the examples of Casper and Glossier in the passage?

    1. They are increasing the purchasing power of poor Americans.
    2. They are exceptions to a dominant trend in consumer markets.
    3. They are facilitating a uniform distribution of commodities in the market.
    4. They might transform into what they were exceptions to.
    Choice D
    They might transform into what they were exceptions to.

  3. CAT 2019 Question Paper - Slot 1

    A new food brand plans to launch a series of products in the American market. Which of the following product plans is most likely to be supported by the author of the passage?

    1. A range of 10 products priced between $5 and $10.
    2. A range of 25 products priced between $10 and $25.
    3. A range of 25 products priced between $5 and $10.
    4. A range of 10 products priced between $10 and $25.
    Choice A
    A range of 10 products priced between $5 and $10.

  4. CAT 2019 Question Paper - Slot 1

    All of the following, IF TRUE, would weaken the author’s claims EXCEPT:

    1. product options increased market competition, bringing down the prices of commodities, which, in turn, increased purchasing power of the poor.
    2. the annual sales growth of companies with fewer product options were higher than that of companies which curated their products for target consumers.
    3. the annual sale of companies that hired lifestyle influencers on Instagram for marketing their products were 40% less than those that did not.
    4. the empowerment felt by purchasers in buying a commodity were directly proportional to the number of options they could choose from.
    Choice B
    the annual sales growth of companies with fewer product options were higher than that of companies which curated their products for target consumers.

  5. CAT 2019 Question Paper - Slot 1

    Based on the passage, all of the following can be inferred about consumer behaviour EXCEPT that:

    1. too many options have made it difficult for consumers to trust products.
    2. consumers are susceptible to marketing images that they see on social media.
    3. having too many product options can be overwhelming for consumers.
    4. consumers tend to prefer products by start-ups over those by established companies.
    Choice D
    consumers tend to prefer products by start-ups over those by established companies.

Emperor Penguins

Scientists recently discovered that Emperor Penguins—one of Antarctica’s most celebrated species—employ a particularly unusual technique for surviving the daily chill. As detailed in an article published today in the journal Biology Letters, the birds minimize heat loss by keeping the outer surface of their plumage below the temperature of the surrounding air. At the same time, the penguins’ thick plumage insulates their body and keeps it toasty. . . .

The researchers analyzed thermographic images . . . taken over roughly a month during June 2008. During that period, the average air temperature was 0.32 degrees Fahrenheit. At the same time, the majority of the plumage covering the penguins’ bodies was even colder: the surface of their warmest body part, their feet, was an average 1.76 degrees Fahrenheit, but the plumage on their heads, chests and backs were -1.84, -7.24 and -9.76 degrees Fahrenheit respectively. Overall, nearly the entire outer surface of the penguins’ bodies was below freezing at all times, except for their eyes and beaks. The scientists also used a computer simulation to determine how much heat was lost or gained from each part of the body—and discovered that by keeping their outer surface below air temperature, the birds might paradoxically be able to draw very slight amounts of heat from the air around them. The key to their trick is the difference between two different types of heat transfer: radiation and convection.

The penguins do lose internal body heat to the surrounding air through thermal radiation, just as our bodies do on a cold day. Because their bodies (but not surface plumage) are warmer than the surrounding air, heat gradually radiates outward over time, moving from a warmer material to a colder one. To maintain body temperature while losing heat, penguins, like all warm-blooded animals, rely on the metabolism of food. The penguins, though, have an additional strategy. Since their outer plumage is even colder than the air, the simulation showed that they might gain back a little of this heat through thermal convection—the transfer of heat via the movement of a fluid (in this case, the air). As the cold Antarctic air cycles around their bodies, slightly warmer air comes into contact with the plumage and donates minute amounts of heat back to the penguins, then cycles away at a slightly colder temperature.

Most of this heat, the researchers note, probably doesn’t make it all the way through the plumage and back to the penguins’ bodies, but it could make a slight difference. At the very least, the method by which a penguin’s plumage wicks heat from the bitterly cold air that surrounds it helps to cancel out some of the heat that’s radiating from its interior. And given the Emperors’ unusually demanding breeding cycle, every bit of warmth counts. . . . Since [penguins trek as far as 75 miles to the coast to breed and male penguins] don’t eat anything during [the incubation period of 64 days], conserving calories by giving up as little heat as possible is absolutely crucial.

  1. CAT 2019 Question Paper - Slot 1

    In the last sentence of paragraph 3, “slightly warmer air” and “at a slightly colder temperature” refer to ______ AND ______ respectively:

    1. the air inside penguins’ bodies kept warm because of metabolism of food AND the fall in temperature of the body air after it transfers some heat to the plumage.
    2. the cold Antarctic air which becomes warmer because of the heat radiated out from penguins’ bodies AND the fall in temperature of the surrounding air after thermal convection.
    3. the air trapped in the plumage which is warmer than the Antarctic air AND the fall in temperature of the trapped plumage air after it radiates out some heat.
    4. the cold Antarctic air whose temperature is higher than that of the plumage AND the fall in temperature of the Antarctic air after it has transmitted some heat to the plumage.
    Choice D
    the cold Antarctic air whose temperature is higher than that of the plumage AND the fall in temperature of the Antarctic air after it has transmitted some heat to the plumage.

  2. CAT 2019 Question Paper - Slot 1

    Which of the following best explains the purpose of the word “paradoxically” as used by the author?

    1. Keeping their body colder helps penguins keep their plumage warmer.
    2. Keeping a part of their body colder helps penguins keep their bodies warmer.
    3. Heat gain through radiation happens despite the heat loss through convection.
    4. Heat loss through radiation happens despite the heat gain through convection.
    Choice B
    Keeping a part of their body colder helps penguins keep their bodies warmer.

  3. CAT 2019 Question Paper - Slot 1

    All of the following, if true, would negate the findings of the study reported in the passage EXCEPT:

    1. the penguins’ plumage were made of a material that did not allow any heat transfer through convection or radiation.
    2. the average air temperature recorded during the month of June 2008 in the area of study were –10 degrees Fahrenheit.
    3. the temperature of the plumage on the penguins’ heads, chests and backs were found to be 1.84, 7.24 and 9.76 degrees Fahrenheit respectively.
    4. the average temperature of the feet of penguins in the month of June 2008 were found to be 2.76 degrees Fahrenheit.
    Choice D
    the average temperature of the feet of penguins in the month of June 2008 were found to be 2.76 degrees Fahrenheit.

  4. CAT 2019 Question Paper - Slot 1

    Which of the following can be responsible for Emperor Penguins losing body heat?

    1. Food metabolism.
    2. Reproduction process.
    3. Plumage.
    4. Thermal convection.
    Choice B
    Reproduction process.

Folk Music

"Free of the taint of manufacture" – that phrase, in particular, is heavily loaded with the ideology of what the Victorian socialist William Morris called the "anti-scrape", or an anti- capitalist conservationism (not conservatism) that solaced itself with the vision of a pre- industrial golden age. In Britain, folk may often appear a cosy, fossilised form, but when you look more closely, the idea of folk – who has the right to sing it, dance it, invoke it, collect it, belong to it or appropriate it for political or cultural ends – has always been contested territory. . . .

In our own time, though, the word "folk" . . . has achieved the rare distinction of occupying fashionable and unfashionable status simultaneously. Just as the effusive floral prints of the radical William Morris now cover genteel sofas, so the revolutionary intentions of many folk historians and revivalists have led to music that is commonly regarded as parochial and conservative. And yet – as newspaper columns periodically rejoice – folk is hip again, influencing artists, clothing and furniture designers, celebrated at music festivals, awards ceremonies and on TV, reissued on countless record labels. Folk is a sonic "shabby chic", containing elements of the uncanny and eerie, as well as an antique veneer, a whiff of Britain's heathen dark ages. The very obscurity and anonymity of folk music's origins open up space for rampant imaginative fancies. . . .

[Cecil Sharp, who wrote about this subject, believed that] folk songs existed in constant transformation, a living example of an art form in a perpetual state of renewal. "One man sings a song, and then others sing it after him, changing what they do not like" is the most concise summary of his conclusions on its origins. He compared each rendition of a ballad to an acorn falling from an oak tree; every subsequent iteration sows the song anew. But there is tension in newness. In the late 1960s, purists were suspicious of folk songs recast in rock idioms. Electrification, however, comes in many forms. For the early-20th-century composers such as Vaughan Williams and Holst, there were thunderbolts of inspiration from oriental mysticism, angular modernism and the body blow of the first world war, as well as input from the rediscovered folk tradition itself.

For the second wave of folk revivalists, such as Ewan MacColl and AL Lloyd, starting in the 40s, the vital spark was communism's dream of a post-revolutionary New Jerusalem. For their younger successors in the 60s, who thronged the folk clubs set up by the old guard, the lyrical freedom of Dylan and the unchained melodies of psychedelia created the conditions for folk- rock's own golden age, a brief Indian summer that lasted from about 1969 to 1971. . . . Four decades on, even that progressive period has become just one more era ripe for fashionable emulation and pastiche. The idea of a folk tradition being exclusively confined to oral transmission has become a much looser, less severely guarded concept. Recorded music and television, for today's metropolitan generation, are where the equivalent of folk memories are seeded. . . .

  1. CAT 2019 Question Paper - Slot 1

    The author says that folk “may often appear a cosy, fossilised form” because:

    1. of its nostalgic association with a pre-industrial past.
    2. it has been arrogated for various political and cultural purposes.
    3. folk is a sonic “shabby chic” with an antique veneer.
    4. the notion of folk has led to several debates and disagreements.
    Choice A
    of its nostalgic association with a pre-industrial past.

  2. CAT 2019 Question Paper - Slot 1

    All of the following are causes for plurality and diversity within the British folk tradition EXCEPT:

    1. the fluidity of folk forms owing to their history of oral mode of transmission.
    2. paradoxically, folk forms are both popular and unpopular.
    3. that British folk forms can be traced to the remote past of the country.
    4. that British folk continues to have traces of pagan influence from the dark ages.
    Choice B
    paradoxically, folk forms are both popular and unpopular.

  3. CAT 2019 Question Paper - Slot 1

    At a conference on folk forms, the author of the passage is least likely to agree with which one of the following views?

    1. The power of folk resides in its contradictory ability to influence and be influenced by the present while remaining rooted in the past.
    2. Folk forms, despite their archaic origins, remain intellectually relevant in contemporary times.
    3. Folk forms, in their ability to constantly adapt to the changing world, exhibit an unusual poise and homogeneity with each change.
    4. The plurality and democratising impulse of folk forms emanate from the improvisation that its practitioners bring to it.
    Choice C
    Folk forms, in their ability to constantly adapt to the changing world, exhibit an unusual poise and homogeneity with each change.

  4. CAT 2019 Question Paper - Slot 1

    The primary purpose of the reference to William Morris and his floral prints is to show:

    1. the pervasive influence of folk on contemporary art, culture, and fashion.
    2. that what is once regarded as radical in folk, can later be seen as conformist.
    3. that what was once derided as genteel is now considered revolutionary.
    4. that despite its archaic origins, folk continues to remain a popular tradition.
    Choice B
    that what is once regarded as radical in folk, can later be seen as conformist.

  5. CAT 2019 Question Paper - Slot 1

    Which of the following statements about folk revivalism of the 1940s and 1960s cannot be inferred from the passage?

    1. Freedom and rebellion were popular themes during the second wave of folk revivalism.
    2. Electrification of music would not have happened without the influence of rock music.
    3. Even though it led to folk-rock’s golden age, it wasn’t entirely free from critique.
    4. It reinforced Cecil Sharp’s observation about folk’s constant transformation.
    Choice B
    Electrification of music would not have happened without the influence of rock music.

Topophilia

As defined by the geographer Yi-Fu Tuan, topophilia is the affective bond between people and place. His 1974 book set forth a wide-ranging exploration of how the emotive ties with the material environment vary greatly from person to person and in intensity, subtlety, and mode of expression. Factors influencing one’s depth of response to the environment include cultural background, gender, race, and historical circumstance, and Tuan also argued that there is a biological and sensory element. Topophilia might not be the strongest of human emotions— indeed, many people feel utterly indifferent toward the environments that shape their lives— but when activated it has the power to elevate a place to become the carrier of emotionally charged events or to be perceived as a symbol.

Aesthetic appreciation is one way in which people respond to the environment. A brilliantly colored rainbow after gloomy afternoon showers, a busy city street alive with human interaction—one might experience the beauty of such landscapes that had seemed quite ordinary only moments before or that are being newly discovered. This is quite the opposite of a second topophilic bond, namely that of the acquired taste for certain landscapes and places that one knows well. When a place is home, or when a space has become the locus of memories or the means of gaining a livelihood, it frequently evokes a deeper set of attachments than those predicated purely on the visual. A third response to the environment also depends on the human senses but may be tactile and olfactory, namely a delight in the feel and smell of air, water, and the earth.

Topophilia—and its very close conceptual twin, sense of place—is an experience that, however elusive, has inspired recent architects and planners. Most notably, new urbanism seeks to counter the perceived placelessness of modern suburbs and the decline of central cities through neo-traditional design motifs. Although motivated by good intentions, such attempts to create places rich in meaning are perhaps bound to disappoint. As Tuan noted, purely aesthetic responses often are suddenly revealed, but their intensity rarely is long- lasting. Topophilia is difficult to design for and impossible to quantify, and its most articulate interpreters have been self-reflective philosophers such as Henry David Thoreau, evoking a marvelously intricate sense of place at Walden Pond, and Tuan, describing his deep affinity for the desert.

Topophilia connotes a positive relationship, but it often is useful to explore the darker affiliations between people and place. Patriotism, literally meaning the love of one’s terra patria or homeland, has long been cultivated by governing elites for a range of nationalist projects, including war preparation and ethnic cleansing. Residents of upscale residential developments have disclosed how important it is to maintain their community’s distinct identity, often by casting themselves in a superior social position and by reinforcing class and racial differences. And just as a beloved landscape is suddenly revealed, so too may landscapes of fear cast a dark shadow over a place that makes one feel a sense of dread or anxiety—or topophobia.

  1. CAT 2019 Question Paper - Slot 1

    The word “topophobia” in the passage is used:

    1. to represent a feeling of dread towards particular spaces and places.
    2. as a metaphor expressing the failure of the homeland to accommodate non-citizens.
    3. to signify the fear of studying the complex discipline of topography.
    4. to signify feelings of fear or anxiety towards topophilic people.
    Choice A
    to represent a feeling of dread towards particular spaces and places.

  2. CAT 2019 Question Paper - Slot 1

    In the last paragraph, the author uses the example of “Residents of upscale residential developments” to illustrate the:

    1. introduction of nationalist projects by such elites to produce a sense of dread or topophobia.
    2. manner in which environments are designed to minimise the social exclusion of their clientele.
    3. sensitive response to race and class problems in upscale residential developments.
    4. social exclusivism practised by such residents in order to enforce a sense of racial or class superiority.
    Choice D
    social exclusivism practised by such residents in order to enforce a sense of racial or class superiority.

  3. CAT 2019 Question Paper - Slot 1

    Which one of the following best captures the meaning of the statement, “Topophilia is difficult to design for and impossible to quantify . . .”?

    1. Philosopher-architects are uniquely suited to develop topophilic design.
    2. People’s responses to their environment are usually subjective and so cannot be rendered in design.
    3. Architects have to objectively quantify spaces and hence cannot be topophilic.
    4. The deep anomie of modern urbanisation led to new urbanism’s intricate sense of place.
    Choice B
    People’s responses to their environment are usually subjective and so cannot be rendered in design.

  4. CAT 2019 Question Paper - Slot 1

    Which one of the following comes closest in meaning to the author’s understanding of topophilia?

    1. The French are not overly patriotic, but they will refuse to use English as far as possible, even when they know it well.
    2. The tendency of many cultures to represent their land as “motherland” or “fatherland” may be seen as an expression of their topophilia.
    3. Scientists have found that most creatures, including humans, are either born with or cultivate a strong sense of topography.
    4. Nomadic societies are known to have the least affinity for the lands through which they traverse because they tend to be topophobic.
    Choice B
    The tendency of many cultures to represent their land as “motherland” or “fatherland” may be seen as an expression of their topophilia.

  5. CAT 2019 Question Paper - Slot 1

    Which of the following statements, if true, could be seen as not contradicting the arguments in the passage?

    1. Generally speaking, in a given culture, the ties of the people to their environment vary little in significance or intensity.
    2. Patriotism, usually seen as a positive feeling, is presented by the author as a darker form of topophilia.
    3. New Urbanism succeeded in those designs where architects collaborated with their clients.
    4. The most important, even fundamental, response to our environment is our tactile and olfactory response.
    Choice B
    Patriotism, usually seen as a positive feeling, is presented by the author as a darker form of topophilia.

CAT VARC : CAT 2019 Question Paper Slot 2

Bureaucracy

Around the world, capital cities are disgorging bureaucrats. In the post-colonial fervour of the 20th century, coastal capitals picked by trade-focused empires were spurned for “regionally neutral” new ones . . . . But decamping wholesale is costly and unpopular; governments these days prefer piecemeal dispersal. The trend reflects how the world has changed. In past eras, when information travelled at a snail’s pace, civil servants had to cluster together. But now desk-workers can ping emails and video-chat around the world. Travel for face-to-face meetings may be unavoidable, but transport links, too, have improved. . . .

Proponents of moving civil servants around promise countless benefits. It disperses the risk that a terrorist attack or natural disaster will cripple an entire government. Wonks in the sticks will be inspired by new ideas that walled-off capitals cannot conjure up. Autonomous regulators perform best far from the pressure and lobbying of the big city. Some even hail a cure for ascendant cynicism and populism. The unloved bureaucrats of faraway capitals will become as popular as firefighters once they mix with regular folk.

Beyond these sunny visions, dispersing central-government functions usually has three specific aims: to improve the lives of both civil servants and those living in clogged capitals; to save money; and to redress regional imbalances. The trouble is that these goals are not always realised.

The first aim—improving living conditions—has a long pedigree. After the second world war Britain moved thousands of civil servants to “agreeable English country towns” as London was rebuilt. But swapping the capital for somewhere smaller is not always agreeable. Attrition rates can exceed 80%. . . . The second reason to pack bureaucrats off is to save money. Office space costs far more in capitals. . . . Agencies that are moved elsewhere can often recruit better workers on lower salaries than in capitals, where well-paying multinationals mop up talent.

The third reason to shift is to rebalance regional inequality. . . . Norway treats federal jobs as a resource every region deserves to enjoy, like profits from oil. Where government jobs go, private ones follow. . . . Sometimes the aim is to fulfil the potential of a country’s second-tier cities. Unlike poor, remote places, bigger cities can make the most of relocated government agencies, linking them to local universities and businesses and supplying a better-educated workforce. The decision in 1946 to set up America’s Centres for Disease Control in Atlanta rather than Washington, D.C., has transformed the city into a hub for health-sector research and business.

The dilemma is obvious. Pick small, poor towns, and areas of high unemployment get new jobs, but it is hard to attract the most qualified workers; opt for larger cities with infrastructure and better-qualified residents, and the country’s most deprived areas see little benefit. . . .

Others contend that decentralisation begets corruption by making government agencies less accountable. . . . A study in America found that state-government corruption is worse when the state capital is isolated—journalists, who tend to live in the bigger cities, become less watchful of those in power.

  1. CAT 2019 Question Paper - Slot 2

    According to the passage, colonial powers located their capitals:

    1. to showcase their power and prestige.
    2. where they had the densest populations.
    3. based on political expediency.
    4. to promote their trading interests.
    Choice D
    to promote their trading interests.

  2. CAT 2019 Question Paper - Slot 2

    The “dilemma” mentioned in the passage refers to:

    1. keeping government agencies in the largest city with good infrastructure or moving them to a remote area with few amenities.
    2. concentrating on decongesting large cities or focusing on boosting employment in relatively larger cities.
    3. encouraging private enterprises to relocate to smaller towns or not incentivising them in order to keep government costs in those towns low.
    4. relocating government agencies to boost growth in remote areas with poor amenities or to relatively larger cities with good amenities.
    Choice D
    relocating government agencies to boost growth in remote areas with poor amenities or to relatively larger cities with good amenities.

  3. CAT 2019 Question Paper - Slot 2

    People who support decentralising central government functions are LEAST likely to cite which of the following reasons for their view?

    1. More independence could be enjoyed by regulatory bodies located away from political centres.
    2. Policy makers may benefit from fresh thinking in a new environment.
    3. It reduces expenses as infrastructure costs and salaries are lower in smaller cities.
    4. It could weaken the nexus between bureaucrats and media in the capital.
    Choice D
    It could weaken the nexus between bureaucrats and media in the capital.

  4. CAT 2019 Question Paper - Slot 2

    The “long pedigree” of the aim to shift civil servants to improve their living standards implies that this move:

    1. is not a new idea and has been tried in the past.
    2. has become common practice in several countries worldwide.
    3. is supported by politicians and the ruling elites.
    4. takes a long time to achieve its intended outcomes.
    Choice A
    is not a new idea and has been tried in the past.

  5. CAT 2019 Question Paper - Slot 2

    According to the author, relocating government agencies has not always been a success for all of the following reasons EXCEPT:

    1. a rise in pollution levels and congestion in the new locations.
    2. the difficulty of attracting talented, well-skilled people in more remote areas.
    3. increased avenues of corruption away from the capital city.
    4. high staff losses, as people may not be prepared to move to smaller towns.
    Choice A
    a rise in pollution levels and congestion in the new locations.

Preservation

War, natural disasters and climate change are destroying some of the world's most precious cultural sites. Google is trying to help preserve these archaeological wonders by allowing users access to 3D images of these treasures through its site.
But the project is raising questions about Google's motivations and about who should own the digital copyrights. Some critics call it a form of "digital colonialism."
When it comes to archaeological treasures, the losses have been mounting. ISIS blew up parts of the ancient city of Palmyra in Syria and an earthquake hit Bagan, an ancient city in Myanmar, damaging dozens of temples, in 2016. In the past, all archaeologists and historians had for restoration and research were photos, drawings, remnants and intuition.
But that's changing. Before the earthquake at Bagan, many of the temples on the site were scanned. . . . [These] scans . . . are on Google's Arts & Culture site. The digital renditions allow viewers to virtually wander the halls of the temple, look up-close at paintings and turn the building over, to look up at its chambers. . . . [Google Arts & Culture] works with museums and other nonprofits . . . to put high-quality images online.
The images of the temples in Bagan are part of a collaboration with CyArk, a nonprofit that creates the 3D scanning of historic sites. . . . Google . . . says [it] doesn't make money off this website, but it fits in with Google's mission to make the world's information available and useful.
Critics say the collaboration could be an attempt by a large corporation to wrap itself in the sheen of culture. Ethan Watrall, an archaeologist, professor at Michigan State University and a member of the Society for American Archaeology, says he's not comfortable with the arrangement between CyArk and Google. . . . Watrall says this project is just a way for Google to promote Google. "They want to make this material accessible so people will browse it and be filled with wonder by it," he says. "But at its core, it's all about advertisements and driving traffic." Watrall says these images belong on the site of a museum or educational institution, where there is serious scholarship and a very different mission. . . .
[There's] another issue for some archaeologists and art historians. CyArk owns the copyrights of the scans — not the countries where these sites are located. That means the countries need CyArk's permission to use these images for commercial purposes.
Erin Thompson, a professor of art crime at John Jay College of Criminal Justice in New York City, says it's the latest example of a Western nation appropriating a foreign culture, a centuries-long battle. . . . CyArk says it copyrights the scans so no one can use them in an inappropriate way. The company says it works closely with authorities during the process, even training local people to help. But critics like Thompson are not persuaded. . . . She would prefer the scans to be owned by the countries and people where these sites are located.

  1. CAT 2019 Question Paper - Slot 2

    Based on his views mentioned in the passage, one could best characterise Dr. Watrall as being:

    1. opposed to the use of digital technology in archaeological and cultural sites in developing countries.
    2. dismissive of laypeople’s access to specialist images of archaeological and cultural sites.
    3. uneasy about the marketing of archaeological images for commercial use by firms such as Google and CyArk.
    4. critical about the links between a non-profit and a commercial tech platform for distributing archaeological images.
    Choice D
    critical about the links between a non-profit and a commercial tech platform for distributing archaeological images.

  2. CAT 2019 Question Paper - Slot 2

    By “digital colonialism”, critics of the CyArk–Google project are referring to the fact that:

    1. CyArk and Google have been scanning images without copyright permission from host countries.
    2. the scanning process can damage delicate frescos and statues at the sites.
    3. countries where the scanned sites are located do not own the scan copyrights.
    4. CyArk and Google have not shared the details of digitisation with the host countries.
    Choice C
    countries where the scanned sites are located do not own the scan copyrights.

  3. CAT 2019 Question Paper - Slot 2

    Which of the following, if true, would most strongly invalidate Dr. Watrall’s objections?

    1. Google takes down advertisements on its website hosting CyArk’s scanned images.
    2. There is a ban on CyArk scanning archeological sites located in other countries.
    3. CyArk does not own the copyright on scanned images of archaeological sites.
    4. CyArk uploads its scanned images of archaeological sites onto museum websites only.
    Choice D
    CyArk uploads its scanned images of archaeological sites onto museum websites only.

  4. CAT 2019 Question Paper - Slot 2

    In Dr. Thompson’s view, CyArk owning the copyright of its digital scans of archaeological sites is akin to:

    1. tourists uploading photos of monuments onto social media.
    2. the seizing of ancient Egyptian artefacts by a Western museum.
    3. the illegal downloading of content from the internet.
    4. digital platforms capturing users’ data for market research.
    Choice B
    the seizing of ancient Egyptian artefacts by a Western museum.

  5. CAT 2019 Question Paper - Slot 2

    Of the following arguments, which one is LEAST likely to be used by the companies that digitally scan cultural sites?

    1. It enables people who cannot physically visit these sites to experience them.
    2. It helps preserve precious images in case the sites are damaged or destroyed.
    3. It allows a large corporation to project itself as a protector of culture.
    4. It provides images free of cost to all users.
    Choice C
    It allows a large corporation to project itself as a protector of culture.

Urban Settlements

The magic of squatter cities is that they are improved steadily and gradually by their residents. To a planner’s eye, these cities look chaotic. I trained as a biologist and to my eye, they look organic. Squatter cities are also unexpectedly green. They have maximum density—1 million people per square mile in some areas of Mumbai—and have minimum energy and material use. People get around by foot, bicycle, rickshaw, or the universal shared taxi.

Not everything is efficient in the slums, though. In the Brazilian favelas where electricity is stolen and therefore free, people leave their lights on all day. But in most slums recycling is literally a way of life. The Dharavi slum in Mumbai has 400 recycling units and 30,000 ragpickers. Six thousand tons of rubbish are sorted every day. In 2007, the Economist reported that in Vietnam and Mozambique, “Waves of gleaners sift the sweepings of Hanoi’s streets, just as Mozambiquan children pick over the rubbish of Maputo’s main tip. Every city in Asia and Latin America has an industry based on gathering up old cardboard boxes.” . . .

In his 1985 article, Calthorpe made a statement that still jars with most people: “The city is the most environmentally benign form of human settlement. Each city dweller consumes less land, less energy, less water, and produces less pollution than his counterpart in settlements of lower densities.” “Green Manhattan” was the inflammatory title of a 2004 New Yorker article by David Owen. “By the most significant measures,” he wrote, “New York is the greenest community in the United States, and one of the greenest cities in the world . . . The key to New York’s relative environmental benignity is its extreme compactness. . . . Placing one and a half million people on a twenty-three-square-mile island sharply reduces their opportunities to be wasteful.” He went on to note that this very compactness forces people to live in the world’s most energy-efficient apartment buildings. . . .

Urban density allows half of humanity to live on 2.8 per cent of the land. . . . Consider just the infrastructure efficiencies. According to a 2004 UN report: “The concentration of population and enterprises in urban areas greatly reduces the unit cost of piped water, sewers, drains, roads, electricity, garbage collection, transport, health care, and schools.” . . .

[T]he nationally subsidised city of Manaus in northern Brazil “answers the question” of how to stop deforestation: give people decent jobs. Then they can afford houses, and gain security. One hundred thousand people who would otherwise be deforesting the jungle around Manaus are now prospering in town making such things as mobile phones and televisions. . . .

Of course, fast-growing cities are far from an unmitigated good. They concentrate crime, pollution, disease and injustice as much as business, innovation, education and entertainment. . . . But if they are overall a net good for those who move there, it is because cities offer more than just jobs. They are transformative: in the slums, as well as the office towers and leafy suburbs, the progress is from hick to metropolitan to cosmopolitan . . .

  1. CAT 2019 Question Paper - Slot 2

    Which one of the following statements would undermine the author’s stand regarding the greenness of cities?

    1. The compactness of big cities in the West increases the incidence of violent crime.
    2. Sorting through rubbish contributes to the rapid spread of diseases in the slums.
    3. The high density of cities leads to an increase in carbon dioxide and global warming.
    4. Over the last decade the cost of utilities has been increasing for city dwellers.
    Choice C
    The high density of cities leads to an increase in carbon dioxide and global warming.

  2. CAT 2019 Question Paper - Slot 2

    According to the passage, squatter cities are environment-friendly for all of the following reasons EXCEPT:

    1. their transportation is energy efficient.
    2. they recycle material.
    3. they sort out garbage.
    4. their streets are kept clean.
    Choice D
    their streets are kept clean.

  3. CAT 2019 Question Paper - Slot 2

    We can infer that Calthorpe’s statement “still jars” with most people because most people:

    1. regard cities as places of disease and crime.
    2. do not consider cities to be eco-friendly places.
    3. do not regard cities as good places to live in.
    4. consider cities to be very crowded and polluted.
    Choice B
    do not consider cities to be eco-friendly places.

  4. CAT 2019 Question Paper - Slot 2

    In the context of the passage, the author refers to Manaus in order to:

    1. explain how urban areas help the environment.
    2. describe the infrastructure efficiencies of living in a city.
    3. explain where cities source their labour for factories.
    4. promote cities as employment hubs for people.
    Choice A
    explain how urban areas help the environment.

  5. CAT 2019 Question Paper - Slot 2

    From the passage it can be inferred that cities are good places to live in for all of the following reasons EXCEPT that they:

    1. offer employment opportunities.
    2. help prevent destruction of the environment.
    3. contribute to the cultural transformation of residents.
    4. have suburban areas as well as office areas.
    Choice D
    have suburban areas as well as office areas.

Linguistics

For two years, I tracked down dozens of . . . Chinese in Upper Egypt [who were] selling lingerie. In a deeply conservative region, where Egyptian families rarely allow women to work or own businesses, the Chinese flourished because of their status as outsiders. They didn’t gossip, and they kept their opinions to themselves. In a New Yorker article entitled “Learning to Speak Lingerie,” I described the Chinese use of Arabic as another non-threatening characteristic. I wrote, “Unlike Mandarin, Arabic is inflected for gender, and Chinese dealers, who learn the language strictly by ear, often pick up speech patterns from female customers. I’ve come to think of it as the lingerie dialect, and there’s something disarming about these Chinese men speaking in the feminine voice.” . . .

When I wrote about the Chinese in the New Yorker, most readers seemed to appreciate the unusual perspective. But as I often find with topics that involve the Middle East, some people had trouble getting past the black-and-white quality of a byline. “This piece is so orientalist I don’t know what to do,” Aisha Gani, a reporter who worked at The Guardian, tweeted. Another colleague at the British paper, Iman Amrani, agreed: “I wouldn’t have minded an article on the subject written by an Egyptian woman—probably would have had better insight.” . . .

As an MOL (man of language), I also take issue with this kind of essentialism. Empathy and understanding are not inherited traits, and they are not strictly tied to gender and race. An individual who wrestles with a difficult language can learn to be more sympathetic to outsiders and open to different experiences of the world. This learning process—the embarrassments, the frustrations, the gradual sense of understanding and connection—is invariably transformative. In Upper Egypt, the Chinese experience of struggling to learn Arabic and local culture had made them much more thoughtful. In the same way, I was interested in their lives not because of some kind of voyeurism, but because I had also experienced Egypt and Arabic as an outsider. And both the Chinese and the Egyptians welcomed me because I spoke their languages. My identity as a white male was far less important than my ability to communicate.

And that easily lobbed word—“Orientalist”—hardly captures the complexity of our interactions. What exactly is the dynamic when a man from Missouri observes a Zhejiang native selling lingerie to an Upper Egyptian woman? . . . If all of us now stand beside the same river, speaking in ways we all understand, who’s looking east and who’s looking west? Which way is Oriental?

For all of our current interest in identity politics, there’s no corresponding sense of identity linguistics. You are what you speak—the words that run throughout your mind are at least as fundamental to your selfhood as is your ethnicity or your gender. And sometimes it’s healthy to consider human characteristics that are not inborn, rigid, and outwardly defined. After all, you can always learn another language and change who you are.

  1. CAT 2019 Question Paper - Slot 2

    Which of the following can be inferred from the author’s claim, “Which way is Oriental?”

    1. Learning another language can mitigate cultural hierarchies and barriers.
    2. Globalisation has mitigated cultural hierarchies and barriers.
    3. Goodwill alone mitigates cultural hierarchies and barriers.
    4. Orientalism is a discourse of the past, from colonial times, rarely visible today.
    Choice A
    Learning another language can mitigate cultural hierarchies and barriers.

  2. CAT 2019 Question Paper - Slot 2

    A French ethnographer decides to study the culture of a Nigerian tribe. Which of the following is most likely to be the view of the author of the passage?

    1. The author would encourage the ethnographer, but ask him/her to first learn the language of the Nigerian tribe s/he wishes to study.
    2. The author would encourage the ethnographer, but ask him/her to be mindful of his/her racial and gender identity in the process.
    3. The author would discourage the ethnographer from conducting the study as Nigerian ethnographers can better understand the tribe.
    4. The author would encourage the ethnographer and recommend him/her to hire a good translator for the purpose of holding interviews.
    Choice A
    The author would encourage the ethnographer, but ask him/her to first learn the language of the Nigerian tribe s/he wishes to study.

  3. CAT 2019 Question Paper - Slot 2

    The author’s critics would argue that:

    1. Empathy can overcome identity politics.
    2. Language is insufficient to bridge cultural barriers.
    3. Linguistic politics can be erased.
    4. Orientalism cannot be practiced by Egyptians.
    Choice B
    Language is insufficient to bridge cultural barriers.

  4. CAT 2019 Question Paper - Slot 2

    According to the passage, which of the following is not responsible for language’s ability to change us?

    1. The ups and downs involved in the course of learning a language.
    2. Language’s intrinsic connection to our notions of self and identity.
    3. Language’s ability to mediate the impact of identity markers one is born with.
    4. The twists and turns in the evolution of language over time.
    Choice D
    The twists and turns in the evolution of language over time.

British Colonial Policy

British colonial policy . . . went through two policy phases, or at least there were two strategies between which its policies actually oscillated, sometimes to its great advantage. At first, the new colonial apparatus exercised caution, and occupied India by a mix of military power and subtle diplomacy, the high ground in the middle of the circle of circles. This, however, pushed them into contradictions. For, whatever their sense of the strangeness of the country and the thinness of colonial presence, the British colonial state represented the great conquering discourse of Enlightenment rationalism, entering India precisely at the moment of its greatest unchecked arrogance. As inheritors and representatives of this discourse, which carried everything before it, this colonial state could hardly adopt for long such a self-denying attitude. It had restructured everything in Europe—the productive system, the political regimes, the moral and cognitive orders—and would do the same in India, particularly as some empirically inclined theorists of that generation considered the colonies a massive laboratory of utilitarian or other theoretical experiments. Consequently, the colonial state could not settle simply for eminence at the cost of its marginality; it began to take initiatives to introduce the logic of modernity into Indian society. But this modernity did not enter a passive society. Sometimes, its initiatives were resisted by pre-existing structural forms. At times, there was a more direct form of collective resistance. Therefore the map of continuity and discontinuity that this state left behind at the time of independence was rather complex and has to be traced with care.

Most significantly, of course, initiatives for . . . modernity came to assume an external character. The acceptance of modernity came to be connected, ineradicably, with subjection. This again points to two different problems, one theoretical, the other political. Theoretically, because modernity was externally introduced, it is explanatorily unhelpful to apply the logical format of the ‘transition process’ to this pattern of change. Such a logical format would be wrong on two counts. First, however subtly, it would imply that what was proposed to be built was something like European capitalism. (And, in any case, historians have forcefully argued that what it was to replace was not like feudalism, with or without modificatory adjectives.) But, more fundamentally, the logical structure of endogenous change does not apply here. Here transformation agendas attack as an external force. This externality is not something that can be casually mentioned and forgotten. It is inscribed on every move, every object, every proposal, every legislative act, each line of causality. It comes to be marked on the epoch itself. This repetitive emphasis on externality should not be seen as a nationalist initiative that is so well rehearsed in Indian social science. . . .

Quite apart from the externality of the entire historical proposal of modernity, some of its contents were remarkable. . . . Economic reforms, or rather alterations . . . did not foreshadow the construction of a classical capitalist economy, with its necessary emphasis on extractive and transport sectors. What happened was the creation of a degenerate version of capitalism—what early dependency theorists called the ‘development of underdevelopment’.

  1. CAT 2019 Question Paper - Slot 2

    All of the following statements about British colonialism can be inferred from the first paragraph, EXCEPT that it:

    1. was at least partly an outcome of Enlightenment rationalism.
    2. faced resistance from existing structural forms of Indian modernity.
    3. was at least partly shaped by the project of European modernity.
    4. allowed the treatment of colonies as experimental sites.
    Choice B
    faced resistance from existing structural forms of Indian modernity.

  2. CAT 2019 Question Paper - Slot 2

    All of the following statements, if true, could be seen as supporting the arguments in the passage, EXCEPT:

    1. the introduction of capitalism in India was not through the transformation of feudalism, as happened in Europe.
    2. modernity was imposed upon India by the British and, therefore, led to underdevelopment.
    3. throughout the history of colonial conquest, natives have often been experimented on by the colonisers.
    4. the change in British colonial policy was induced by resistance to modernity in Indian society.
    Choice D
    the change in British colonial policy was induced by resistance to modernity in Indian society.

  3. CAT 2019 Question Paper - Slot 2

    “Consequently, the colonial state could not settle simply for eminence at the cost of its marginality; it began to take initiatives to introduce the logic of modernity into Indian society.” Which of the following best captures the sense of this statement?

    1. The colonial state’s eminence was unsettled by its marginal position; therefore, it developed Indian society by modernising it.
    2. The colonial enterprise was a costly one; so to justify the cost it began to take initiatives to introduce the logic of modernity into Indian society.
    3. The colonial state felt marginalised from Indian society because of its own modernity; therefore, it sought to address that marginalisation by bringing its modernity to change Indian society.
    4. The cost of the colonial state’s eminence was not settled; therefore, it took the initiative of introducing modernity into Indian society.
    Choice C
    The colonial state felt marginalised from Indian society because of its own modernity; therefore, it sought to address that marginalisation by bringing its modernity to change Indian society.

  4. CAT 2019 Question Paper - Slot 2

    Which one of the following 5-word sequences best captures the flow of the arguments in the passage?

    1. Military power—arrogance—laboratory—modernity—capitalism.
    2. Colonial policy—Enlightenment—external modernity—subjection—underdevelopment.
    3. Colonial policy—arrogant rationality—resistance—independence—development.
    4. Military power—colonialism—restructuring—feudalism—capitalism.
    Choice B
    Colonial policy—Enlightenment—external modernity—subjection—underdevelopment.

  5. CAT 2019 Question Paper - Slot 2

    Which of the following observations is a valid conclusion to draw from the author’s statement that “the logical structure of endogenous change does not apply here. Here transformation agendas attack as an external force”?

    1. The endogenous logic of colonialism can only bring change if it attacks and transforms external forces.
    2. Indian society is not endogamous; it is more accurately characterised as aggressively exogamous.
    3. Colonised societies cannot be changed through logic; they need to be transformed with external force.
    4. The transformation of Indian society did not happen organically, but was forced by colonial agendas.
    Choice D
    The transformation of Indian society did not happen organically, but was forced by colonial agendas.

CAT VARC : CAT 2018 Question Paper Slot 1

Plastic Pollution

The only thing worse than being lied to is not knowing you’re being lied to. It’s true that plastic pollution is a huge problem, of planetary proportions. And it’s true we could all do more to reduce our plastic footprint. The lie is that blame for the plastic problem is wasteful consumers and that changing our individual habits will fix it.

Recycling plastic is to saving the Earth what hammering a nail is to halting a falling skyscraper. You struggle to find a place to do it and feel pleased when you succeed. But your effort is wholly inadequate and distracts from the real problem of why the building is collapsing in the first place. The real problem is that single-use plastic—the very idea of producing plastic items like grocery bags, which we use for an average of 12 minutes but can persist in the environment for half a millennium—is an incredibly reckless abuse of technology. Encouraging individuals to recycle more will never solve the problem of a massive production of single-use plastic that should have been avoided in the first place.

As an ecologist and evolutionary biologist, I have had a disturbing window into the accumulating literature on the hazards of plastic pollution. Scientists have long recognized that plastics biodegrade slowly, if at all, and pose multiple threats to wildlife through entanglement and consumption. More recent reports highlight dangers posed by absorption of toxic chemicals in the water and by plastic odors that mimic some species’ natural food. Plastics also accumulate up the food chain, and studies now show that we are likely ingesting it ourselves in seafood. . . .

Beginning in the 1950s, big beverage companies like Coca-Cola and Anheuser-Busch, along with Phillip Morris and others, formed a non-profit called Keep America Beautiful. Its mission is/was to educate and encourage environmental stewardship in the public. . . . At face value, these efforts seem benevolent, but they obscure the real problem, which is the role that corporate polluters play in the plastic problem. This clever misdirection has led journalist and author Heather Rogers to describe Keep America Beautiful as the first corporate greenwashing front, as it has helped shift the public focus to consumer recycling behavior and actively thwarted legislation that would increase extended producer responsibility for waste management. . . . [T]he greatest success of Keep America Beautiful has been to shift the onus of environmental responsibility onto the public while simultaneously becoming a trusted name in the environmental movement. . . .

So what can we do to make responsible use of plastic a reality? First: reject the lie. Litterbugs are not responsible for the global ecological disaster of plastic. Humans can only function to the best of their abilities, given time, mental bandwidth and systemic constraints. Our huge problem with plastic is the result of a permissive legal framework that has allowed the uncontrolled rise of plastic pollution, despite clear evidence of the harm it causes to local communities and the world’s oceans. Recycling is also too hard in most parts of the U.S. and lacks the proper incentives to make it work well.

  1. CAT 2018 Question Paper - Slot 1

    Which of the following interventions would the author most strongly support:

    1. recycling all plastic debris in the seabed.
    2. having all consumers change their plastic consumption habits.
    3. completely banning all single-use plastic bags.
    4. passing regulations targeted at producers that generate plastic products.
    Choice D
    passing regulations targeted at producers that generate plastic products.

  2. CAT 2018 Question Paper - Slot 1

    The author lists all of the following as negative effects of the use of plastics EXCEPT the:

    1. slow pace of degradation or non-degradation of plastics in the environment.
    2. air pollution caused during the process of recycling plastics.
    3. poisonous chemicals released into the water and food we consume.
    4. adverse impacts on the digestive systems of animals exposed to plastic.
    Choice B
    air pollution caused during the process of recycling plastics.

  3. CAT 2018 Question Paper - Slot 1

    In the first paragraph, the author uses “lie” to refer to the:

    1. blame assigned to consumersfor indiscriminate use of plastics.
    2. understatement of the enormity of the plastics pollution problem.
    3. fact that people do not know they have been lied to.
    4. understatement of the effects of recycling plastics.
    Choice A
    blame assigned to consumersfor indiscriminate use of plastics.

  4. CAT 2018 Question Paper - Slot 1

    In the second paragraph, the phrase “what hammering a nail is to halting a falling skyscraper” means:

    1. relying on emerging technologies to mitigate the ill-effects of plastic pollution.
    2. encouraging the responsible production of plastics by firms.
    3. focusing on consumer behaviour to tackle the problem of plastics pollution.
    4. focusing on single-use plastic bags to reduce the plastics footprint.
    Choice C
    focusing on consumer behaviour to tackle the problem of plastics pollution.

  5. CAT 2018 Question Paper - Slot 1

    It can be inferred that the author considers the Keep America Beautiful organisation:

    1. a "greenwash" because it was a benevolent attempt to improve public recycling habits.
    2. a sham as it diverted attention away from the role of corporates in plastics pollution.
    3. an important step in sensitising producers to the need to tackle plastics pollution.
    4. an innovative example of a collaborative corporate social responsibility initiative.
    Choice B
    a sham as it diverted attention away from the role of corporates in plastics pollution.

Human-Elephant Conflict

“Everybody pretty much agrees that the relationship between elephants and people has dramatically changed,” [says psychologist Gay] Bradshaw. “Where for centuries humans and elephants lived in relatively peaceful coexistence, there is now hostility and violence. Now, I use the term ‘violence’ because of the intentionality associated with it, both in the aggression of humans and, at times, the recently observed behavior of elephants.”

Typically, elephant researchers have cited, as a cause of aggression, the high levels of testosterone in newly matured male elephants or the competition for land and resources between elephants and humans. But, Bradshaw and several colleagues argue that today’s elephant populations are suffering from a form of chronic stress, a kind of species-wide trauma. Decades of poaching and culling and habitat loss, they claim, have so disrupted the intricate web of familial and societal relations by which young elephants have traditionally been raised in the wild, and by which established elephant herds are governed, that what we are now witnessing is nothing less than a precipitous collapse of elephant culture.

Elephants, when left to their own devices, are profoundly social creatures. young elephants are raised within an extended, multi-tiered network of doting female caregivers that includes the birth mother, grandmothers, aunts and friends. These relations are maintained over a life span as long as 70 years. Studies of established herds have shown that young elephants stay within 15 feet of their mothers for nearly all of their first eight years of life, after which young females are socialized into the matriarchal network, while young males go off for a time into an all-male social group before coming back into the fold as mature adults.

This fabric of elephant society, Bradshaw and her colleagues [demonstrate], ha[s] effectively been frayed by years of habitat loss and poaching, along with systematic culling by government agencies to control elephant numbers and translocations of herds to different habitats. As a result of such social upheaval, calves are now being born to and raised by ever younger and inexperienced mothers. Young orphaned elephants, meanwhile, that have witnessed the death of a parent at the hands of poachers are coming of age in the absence of the support system that defines traditional elephant life. “The loss of elephant elders,” [says] Bradshaw "and the traumatic experience of witnessing the massacres of their family, impairs normal brain and behavior development in young elephants.”

What Bradshaw and her colleagues describe would seem to be an extreme form of anthropocentric conjecture if the evidence that they’ve compiled from various elephant researchers weren’t so compelling. The elephants of decimated herds, especially orphans who’ve watched the death of their parents and elders from poaching and culling, exhibit behavior typically associated with post-traumatic stress disorder and other trauma-related disorders in humans: abnormal startle response, unpredictable asocial behavior, inattentive mothering and hyper-aggression.

[According to Bradshaw], “Elephants are suffering and behaving in the same ways that we recognize in ourselves as a result of violence. Except perhaps for a few specific features, brain organization and early development of elephants and humans are extremely similar.”

  1. CAT 2018 Question Paper - Slot 1

    Which of the following statements best expresses the overall argument of this passage?

    1. Elephants, like the humans they are in conflict with, are profoundly social creatures.
    2. The relationship between elephants and humans has changed from one of coexistence to one of hostility
    3. Recent elephant behaviour could be understood as a form of species-wide trauma-related response.
    4. The brain organisation and early development of Aelephants and humans are extremely similar
    Choice C
    Recent elephant behaviour could be understood as a form of species-wide trauma-related response.

  2. CAT 2018 Question Paper - Slot 1

    In the first paragraph, Bradshaw uses the term "violence" to describe the recent change in the human-elephant relationship because, according to him:

    1. Both humans and elephants have killed members of each other's species.
    2. There is a purposefulness in human and elephant aggression towards each other.
    3. Human-elephant interactions have changed their character over time.
    4. Elephant herds and their habitat have been systematically destroyed by humans.
    Choice B
    There is a purposefulness in human and elephant aggression towards each other.

  3. CAT 2018 Question Paper - Slot 1

    The passage makes all of the following claims EXCEPT

    1. Elephant mothers are evolving newer ways of rearing their calves to adapt to emerging threats.
    2. The elephant response to deeply disturbing experiences is similar to that of humans.
    3. Elephants establish e[tended and enduring familial relationships as do humans.
    4. Human actions such as poaching and culling have created stressful conditions for elephant communities.
    Choice A
    Elephant mothers are evolving newer ways of rearing their calves to adapt to emerging threats.

  4. CAT 2018 Question Paper - Slot 1

    Which of the following measures is Bradshaw most likely to support to address the problem of elephant aggression?

    1. The development of treatment programmes for elephants drawing on insights gained from treating post-traumatic stress disorder in humans.
    2. Funding of more studies to better understand the impact of testosterone on male elephant aggression.
    3. Studying the impact of isolating elephant calves on their early brain development, behaviour and aggression.
    4. Increased funding for research into the similarity of humans and other animals drawing on insights gained from human-elephant similarities.
    Choice A
    The development of treatment programmes for elephants drawing on insights gained from treating post-traumatic stress disorder in humans.

  5. CAT 2018 Question Paper - Slot 1

    In paragraph 4, the phrase, “The fabric of elephant society . . . has(s) effectively been frayed by . . .” is:

    1. an exaggeration aimed at bolstering Bradshaw's claims.
    2. an accurate description of the condition of elephant herds today.
    3. an ode to the fragility of elephant society today.
    4. a metaphor for the effect of human activity on elephant communities.
    Choice D
    a metaphor for the effect of human activity on elephant communities.

India and the World War

The Indian government has announced an international competition to design a National War Memorial in New Delhi, to honour all of the Indian soldiers who served in the various wars and counter-insurgency campaigns from 1947 onwards. The terms of the competition also specified that the new structure would be built adjacent to the India Gate – a memorial to the Indian soldiers who died in the First World War. Between the old imperialist memorial and the proposed nationalist one, India’s contribution to the Second World War is airbrushed out of existence.

The Indian government’s conception of the war memorial was not merely absentminded. Rather, it accurately reflected the fact that both academic history and popular memory have yet to come to terms with India’s Second World War, which continues to be seen as little more than mood music in the drama of India’s advance towards independence and partition in 1947. Further, the political trajectory of the postwar subcontinent has militated against popular remembrance of the war. With partition and the onset of the India-Pakistan rivalry, both of the new nations needed fresh stories for self-legitimisation rather than focusing on shared wartime experiences.

However, the Second World War played a crucial role in both the independence and partition of India. The Indian army recruited, trained and deployed some 2.5 million men, almost 90,000 of which were killed and many more injured. Even at the time, it was recognised as the largest volunteer force in the war.

India’s material and financial contribution to the war was equally significant. India emerged as a major military-industrial and logistical base for Allied operations in south-east Asia and the Middle East. This led the United States to take considerable interest in the country’s future, and ensured that this was no longer the preserve of the British government. Other wartime developments pointed in the direction of India’s independence. In a stunning reversal of its long-standing financial relationship with Britain, India finished the war as one of the largest creditors to the imperial power.

Such extraordinary mobilization for war was achieved at great human cost, with the Bengal famine the most extreme manifestation of widespread wartime deprivation. The costs on India’s home front must be counted in millions of lives.

Indians signed up to serve on the war and home fronts for a variety of reasons. Many were convinced that their contribution would open the doors to India’s freedom. The political and social churn triggered by the war was evident in the massive waves of popular protest and unrest that washed over rural and urban India in the aftermath of the conflict. This turmoil was crucial in persuading the Attlee government to rid itself of the incubus of ruling India. Seventy years on, it is time that India engaged with the complex legacies of the Second World War. Bringing the war into the ambit of the new national memorial would be a fitting – if not overdue – recognition that this was India’s War.

  1. CAT 2018 Question Paper - Slot 1

    In the first paragraph, the author laments the fact that

    1. the new war memorial will be built right next to India Gate.
    2. there is no recognition of the Indian soldiers who served in the Second World War.
    3. funds will be wasted on another war memorial when we already have the India Gate memorial.
    4. India lost thousands of human lives during the Second World War
    Choice B
    there is no recognition of the Indian soldiers who served in the Second World War.

  2. CAT 2018 Question Paper - Slot 1

    The author lists all of the following as outcomes of the Second World War EXCEPT:

    1. US recognition of India's strategic location and role in the war.
    2. Large-scale deaths in Bengal as a result of deprivation and famine.
    3. Independence of the subcontinent and its partition into two countries.
    4. The large financial debt India owed to Britain after the war.
    Choice D
    The large financial debt India owed to Britain after the war.

  3. CAT 2018 Question Paper - Slot 1

    The phrase “mood music” is used in the second paragraph to indicate that the Second World War is viewed as:

    1. Setting the stage for the emergence of the India–Pakistan rivalry in the subcontinent.
    2. a backdrop to the subsequent independence and partition of the region.
    3. a part of the narrative on the ill-effects of colonial rule on India.
    4. a tragic period in terms of loss of lives and national wealth.
    Choice B
    a backdrop to the subsequent independence and partition of the region.

  4. CAT 2018 Question Paper - Slot 1

    The author suggests that a major reason why India has not so far acknowledged its role in the Second World War is that it:

    1. wants to forget the human and financial toll of the War on the country
    2. has been focused on building an independent, non-colonial political identity.
    3. views the War as a predominantly Allied effort, with India playing only a supporting role.
    4. blames the War for leading to the momentous partition of the country.
    Choice B
    has been focused on building an independent, non-colonial political identity.

  5. CAT 2018 Question Paper - Slot 1

    The author claims that omitting mention of Indians who served in the Second World War from the new National War Memorial is:

    1. a reflection of misplaced priorities of the post-independence Indian governments.
    2. a reflection of the academic and popular view of India’s role in the War.
    3. appropriate as their names can always be included in the India Gate memorial.
    4. is something which can be rectified in future by constructing a separate memorial.
    Choice B
    a reflection of the academic and popular view of India’s role in the War

Economy and Happiness

Economists have spent most of the 20th century ignoring psychology, positive or otherwise. But today there is a great deal of emphasis on how happiness can shape global economies, or — on a smaller scale — successful business practice. This is driven, in part, by a trend in "measuring" positive emotions, mostly so they can be optimized. Neuroscientists, for example, claim to be able to locate specific emotions, such as happiness or disappointment, in particular areas of the brain. Wearable technologies, such as Spire, offer data-driven advice on how to reduce stress.

We are no longer just dealing with "happiness" in a philosophical or romantic sense — it has become something that can be monitored and measured, including by our behavior, use of social media and bodily indicators such as pulse rate and facial expressions. There is nothing automatically sinister about this trend. But it is disquieting that the businesses and experts driving the quantification of happiness claim to have our best interests at heart, often concealing their own agendas in the process. In the workplace, happy workers are viewed as a "win-win." Work becomes more pleasant, and employees, more productive. But this is now being pursued through the use of performance-evaluating wearable technology, such as Humanyze or Virgin Pulse, both of which monitor physical signs of stress and activity toward the goal of increasing productivity.

Cities such as Dubai, which has pledged to become the "happiest city in the world," dream up ever-more elaborate and intrusive ways of collecting data on well-being — to the point where there is now talk of using CCTV cameras to monitor facial expressions in public spaces. New ways of detecting emotions are hitting the market all the time: One company, Beyond Verbal, aims to calculate moods conveyed in a phone conversation, potentially without the knowledge of at least one of the participants. And Facebook [has] demonstrated that it could influence our emotions through tweaking our news feeds — opening the door to ever-more targeted manipulation in advertising and influence.

As the science grows more sophisticated and technologies become more intimate with our thoughts and bodies, a clear trend is emerging. Where happiness indicators were once used as a basis to reform society, challenging the obsession with money that G.D.P. measurement entrenches, they are increasingly used as a basis to transform or discipline individuals.

Happiness becomes a personal project, that each of us must now work on, like going to the gym. Since the 1970s, depression has come to be viewed as a cognitive or neurological defect in the individual, and never a consequence of circumstances. All of this simply escalates the sense of responsibility each of us feels for our own feelings, and with it, the sense of failure when things go badly. A society that deliberately removed certain sources of misery, such as precarious and exploitative employment, may well be a happier one. But we won't get there by making this single, often fleeting emotion, the over-arching goal.

  1. CAT 2018 Question Paper - Slot 1

    According to the author, wearable technologies and social media are contributing most to:

    1. happiness as a “personal project”.
    2. depression as a thing of the past.
    3. disciplining individualsto be happy.
    4. making individuals aware of stress in their lives.
    Choice C
    disciplining individualsto be happy.

  2. CAT 2018 Question Paper - Slot 1

    The author’s view would be undermined by which of the following research findings?

    1. There is a definitive move towards the adoption of wearable technology that taps into emotions.
    2. Stakeholders globally are moving away from collecting data on the well-being of individuals.
    3. A proliferation of gyms that are collecting data on customer well-being.
    4. Individuals worldwide are utilising technologies to monitor and increase their wellbeing.
    Choice B
    Stakeholders globally are moving away from collecting data on the well-being of individuals.

  3. CAT 2018 Question Paper - Slot 1

    According to the author, Dubai:

    1. is on its way to becoming one of the world’s happiest cities.
    2. collaborates with Facebook to selectively influence its inhabitants’ moods.
    3. develops sophisticated technologies to monitor its inhabitants’ states of mind.
    4. incentivises companies that prioritise worker welfare.
    Choice C
    develops sophisticated technologies to monitor its inhabitants’ states of mind.

  4. CAT 2018 Question Paper - Slot 1

    In the author's opinion, the shift in thinking in the 1970s:

    1. was a welcome change from the earlier view that depression could be cured by changing circumstances.
    2. introduced greater stress into people’s lives as they were expected to be responsible for their own happiness.
    3. put people in touch with their own feelings rather than depending on psychologists.
    4. reflected the emergence of neuroscience as the authority on human emotions.
    Choice B
    introduced greater stress into people’s lives as they were expected to be responsible for their own happiness.

  5. CAT 2018 Question Paper - Slot 1

    From the passage we can infer that the author would like economists to:

    1. incorporate psychological findings into their research cautiously
    2. correlate measurements of happiness with economic indicators
    3. measure the effectiveness of Facebook and social media advertising
    4. work closely with neuroscientists to understand human behaviour.
    Choice A
    incorporate psychological findings into their research cautiously

Acquired Characterisitcs

When researchers at Emory University in Atlanta trained mice to fear the smell of almonds (by pairing it with electric shocks), they found, to their consternation, that both the children and grandchildren of these mice were spontaneously afraid of the same smell. That is not supposed to happen. Generations of schoolchildren have been taught that the inheritance of acquired characteristics is impossible. A mouse should not be born with something its parents have learned during their lifetimes, any more than a mouse that loses its tail in an accident should give birth to tailless mice.

Modern evolutionary biology dates back to a synthesis that emerged around the 1940s60s, which married Charles Darwin’s mechanism of natural selection with Gregor Mendel’s discoveries of how genes are inherited. The traditional, and still dominant, view is that adaptations – from the human brain to the peacock’s tail – are fully and satisfactorily explained by natural selection (and subsequent inheritance). Yet [new evidence] from genomics, epigenetics and developmental biology [indicates] that evolution is more complex than we once assumed.

In his book On Human Nature (1978), the evolutionary biologist Edward O Wilson claimed that human culture is held on a genetic leash. The metaphor [needs revision]. Imagine a dogwalker (the genes) struggling to retain control of a brawny mastiff (human culture). The pair’s trajectory (the pathway of evolution) reflects the outcome of the struggle. Now imagine the same dog-walker struggling with multiple dogs, on leashes of varied lengths, with each dog tugging in different directions. All these tugs represent the influence of developmental factors, including epigenetics, antibodies and hormones passed on by parents, as well as the ecological legacies and culture they bequeath.

The received wisdom is that parental experiences can’t affect the characters of their offspring. Except they do. The way that genes are expressed to produce an organism’s phenotype– the actual characteristics it ends up with – is affected by chemicals that attach to them. Everything from diet to air pollution to parental behaviour can influence the addition or removal of these chemical marks, which switches genes on or off. Usually these so-called ‘epigenetic’ attachments are removed during the production of sperm and eggs cells, but it turns out that some escape the resetting process and are passed on to the next generation, along with the genes. This is known as ‘epigenetic inheritance’, and more and more studies are confirming that it really happens. Let’s return to the almond-fearing mice. The inheritance of an epigenetic mark transmitted in the sperm is what led the mice’s offspring to acquire an inherited fear.

Epigenetics is only part of the story. Through culture and society, [humans and other animals] inherit knowledge and skills acquired by [their] parents. All this complexity points to an evolutionary process in which genomes (over hundreds to thousands of generations), epigenetic modifications and inherited cultural factors (over several, perhaps tens or hundreds of generations), and parental effects (over single-generation timespans) collectively informb how organisms adapt. These extra-genetic kinds of inheritance give organisms the flexibility to make rapid adjustments to environmental challenges, dragging genetic change in their wake – much like a rowdy pack of dogs.

  1. CAT 2018 Question Paper - Slot 1

    The passage uses the metaphor of a dog walker to argue that evolutionary adaptation is most comprehensively understood as being determined by:

    1. genetic, epigenetic, developmental factors, and ecological legacies.
    2. socio-cultural, genetic, epigenetic, and genomic legacies.
    3. ecological, hormonal, extra genetic and genetic legacies.
    4. extra genetic, genetic, epigenetic and genomic legacies.
    Choice A
    genetic, epigenetic, developmental factors, and ecological legacies.

  2. CAT 2018 Question Paper - Slot 1

    Which of the following options best describes the author's argument?

    1. Darwin’s theory of natural selection cannot fully explain evolution.
    2. Mendel’s theory of inheritance is unfairly underestimated in explaining evolution.
    3. Darwin’s and Mendel’s theories together best explain evolution.
    4. Wilson’s theory of evolution is scientifically superior to either Darwin’s or Mendel’s.
    Choice A
    Darwin’s theory of natural selection cannot fully explain evolution.

  3. CAT 2018 Question Paper - Slot 1

    The Emory University experiment with mice points to the inheritance of:

    1. acquired characteristics
    2. psychological markers
    3. personality traits
    4. acquired parental fears
    Choice A
    acquired characteristics

  4. CAT 2018 Question Paper - Slot 1

    Which of the following, if found to be true, would negate the main message of the passage?

    1. A study indicating the primacy of ecological impact on human adaptation.
    2. A study highlighting the criticality of epigenetic inheritance to evolution.
    3. A study affirming the sole influence of natural selection and inheritance on evolution.
    4. A study affirming the influence of socio-cultural markers on evolutionary processes.
    Choice C
    A study affirming the sole influence of natural selection and inheritance on evolution.

CAT VARC : CAT 2018 Question Paper Slot 2

E-Governance

Will a day come when India’s poor can access government services as easily as drawing cash from an ATM? No country in the world has made accessing education or health or policing or dispute resolution as easy as an ATM, because the nature of these activities requires individuals to use their discretion in a positive way. Technology can certainly facilitate this in a variety of ways if it is seen as one part of an overall approach, but the evidence so far in education, for instance, is that just adding computers alone doesn’t make education any better.

The dangerous illusion of technology is that it can create stronger, top down accountability of service providers in implementation-intensive services within existing public sector organisations. One notion is that electronic management information systems (EMIS) keep better track of inputs and those aspects of personnel that are ‘EMIS visible’ can lead to better services. A recent study examined attempts to increase attendance of Auxiliary Nurse Midwife (ANMs) at clinics in Rajasthan, which involved high-tech time clocks to monitor attendance. The study’s title says it all: Band-Aids on a Corpse. E-governance can be just as bad as any other governance when the real issue is people and their motivation.

For services to improve, the people providing the services have to want to do a better job with the skills they have. A study of medical care in Delhi found that even though providers, in the public sector had much better skills than private sector providers their provision of care in actual practice was much worse.

In implementation-intensive services the key to success is face-to-face interactions between a teacher, a nurse, a policeman, an extension agent and a citizen. This relationship is about power. Amartya Sen’s report on education in West Bengal had a supremely telling anecdote in which the villagers forced the teacher to attend school, but then, when the parents went off to work, the teacher did not teach, but forced the children to massage his feet. As long as the system empowers providers over citizens, technology is irrelevant.

The answer to successfully providing basic services is to create systems that provide both autonomy and accountability. In basic education for instance, the answer to poor teaching is not controlling teachers more. The key is to hire teachers who want to teach and let them teach, expressing their professionalism and vocation as a teacher through autonomy in the classroom. This autonomy has to be matched with accountability for results—not just narrowly measured through test scores, but broadly for the quality of the education they provide.

A recent study in Uttar Pradesh showed that if, somehow, all civil service teachers could be replaced with contract teachers, the state could save a billion dollars a year in revenue and double student learning. Just the additional autonomy and accountability of contracts through local groups—even without complementary system changes in information and empowerment—led to that much improvement. The first step to being part of the solution is to create performance information accessible to those outside of the government.

  1. CAT 2018 Question Paper - Slot 2

    In the context of the passage, we can infer that the title “Band Aids on a Corpse” (in paragraph 2) suggests that:

    1. the nurses who attended the clinics were too poorly trained to provide appropriate medical care.
    2. the electronic monitoring system was a superficial solution to a serious problem.
    3. the nurses attended the clinics, but the clinics were ill-equipped.
    4. the clinics were better funded, but performance monitoring did not result in any improvement.
    Choice B
    the electronic monitoring system was a superficial solution to a serious problem.

  2. CAT 2018 Question Paper - Slot 2

    According to the author, service delivery in Indian education can be improved in all of the following ways EXCEPT through:

    1. use of technology.
    2. recruitment of motivated teachers.
    3. access to information on the quality of teaching.
    4. elimination of government involvement.
    Choice D
    elimination of government involvement.

  3. CAT 2018 Question Paper - Slot 2

    Which of the following, IF TRUE, would undermine the passage’s main argument?

    1. If absolute instead of moderate technological surveillance is exercised over the performance of service providers.
    2. Empowerment of service providers leads to increased complacency and rigged performance results.
    3. If it were proven that increase in autonomy of service providers leads to an exponential increase in their work ethic and sense of responsibility.
    4. If it were proven that service providers in the private sector have better skills than those in the public sector.
    Choice B
    Empowerment of service providers leads to increased complacency and rigged performance results.

  4. CAT 2018 Question Paper - Slot 2

    The author questions the use of monitoring systems in services that involve face-to-face interaction between service providers and clients because such systems:

    1. do not improve services that need committed service providers.
    2. are not as effective in the public sector as they are in the private sector.
    3. improve the skills but do not increase the motivation of service providers
    4. are ineffective because they are managed by the government.
    Choice A
    do not improve services that need committed service providers.

  5. CAT 2018 Question Paper - Slot 2

    The main purpose of the passage is to:

    1. critique the government’s involvement in educational activities and other implementation-intensive services.
    2. argue that some types of services can be improved by providing independence and requiring accountability.
    3. analyse the shortcomings of government-appointed nurses and their management through technology.
    4. find a solution to the problem of poor service delivery in education by examining different strategies.
    Choice B
    argue that some types of services can be improved by providing independence and requiring accountability.

White-lipped Grove Snails

Grove snails as a whole are distributed all over Europe, but a specific variety of the snail, with a distinctive white-lipped shell, is found exclusively in Ireland and in the Pyrenees mountains that lie on the border between France and Spain. The researchers sampled a total of 423 snail specimens from 36 sites distributed across Europe, with an emphasis on gathering large numbers of the white-lipped variety. When they sequenced genes from the mitochondrial DNA of each of these snails and used algorithms to analyze the genetic diversity between them, they found that a distinct lineage (the snails with the white-lipped shells) was indeed endemic to the two very specific and distant places in question.

Explaining this is tricky. Previously, some had speculated that the strange distributions of creatures such as the white-lipped grove snails could be explained by convergent evolution—in which two populations evolve the same trait by coincidence—but the underlying genetic similarities between the two groups rules that out. Alternately, some scientists had suggested that the white-lipped variety had simply spread over the whole continent, then been wiped out everywhere besides Ireland and the Pyrenees, but the researchers say their sampling and subsequent DNA analysis eliminate that possibility too.

“If the snails naturally colonized Ireland, you would expect to find some of the same genetic type in other areas of Europe, especially Britain. We just don’t find them,” Davidson, the lead author, said in a press statement.

Moreover, if they’d gradually spread across the continent, there would be some genetic variation within the white-lipped type, because evolution would introduce variety over the thousands of years it would have taken them to spread from the Pyrenees to Ireland. That variation doesn’t exist, at least in the genes sampled. This means that rather than the organism gradually expanding its range, large populations instead were somehow moved en mass to the other location within the space of a few dozen generations, ensuring a lack of genetic variety.

“There is a very clear pattern, which is difficult to explain except by involving humans,” Davidson said. Humans, after all, colonized Ireland roughly 9,000 years ago, and the oldest fossil evidence of grove snails in Ireland dates to roughly the same era. Additionally, there is archaeological evidence of early sea trade between the ancient peoples of Spain and Ireland via the Atlantic and even evidence that humans routinely ate these types of snails before the advent of agriculture, as their burnt shells have been found in Stone Age trash heaps.

The simplest explanation, then? Boats. These snails may have inadvertently traveled on the floor of the small, coast-hugging skiffs these early humans used for travel, or they may have been intentionally carried to Ireland by the seafarers as a food source. “The highways of the past were rivers and the ocean–as the river that flanks the Pyrenees was an ancient trade route to the Atlantic, what we’re actually seeing might be the long lasting legacy of snails that hitched a ride as humans travelled from the South of France to Ireland 8,000 years ago,” Davidson said.

  1. CAT 2018 Question Paper - Slot 2

    All of the following evidence supports the passage’s explanation of sea travel/trade EXCEPT:

    1. the coincidental existence of similar traits in the white-lipped grove snails of Ireland and the Pyrenees because of convergent evolution.
    2. the oldest fossil evidence of white-lipped grove snails in Ireland dates back to roughly 9,000 years ago, the time when humans colonised Ireland.
    3. absence of genetic variation within the white-lipped grove snails of Ireland and the Pyrenees, whose genes were sampled.
    4. archaeological evidence of early sea trade between the ancient peoples of Spain and Ireland via the Atlantic Ocean.
    Choice A
    the coincidental existence of similar traits in the white-lipped grove snails of Ireland and the Pyrenees because of convergent evolution.

  2. CAT 2018 Question Paper - Slot 2

    The passage outlines several hypotheses and evidence related to white-lipped grove snails to arrive at the most convincing explanation for:

    1. why the white-lipped variety of grove snails were wiped out everywhere except in Ireland and the Pyrenees.
    2. how the white-lipped variety of grove snails independently evolved in Ireland and the Pyrenees.
    3. why the white-lipped variety of grove snails are found only in Ireland and the Pyrenees.
    4. how the white-lipped variety of grove snails independently evolved in Ireland and the Pyrenees.
    Choice C
    why the white-lipped variety of grove snails are found only in Ireland and the Pyrenees.

  3. CAT 2018 Question Paper - Slot 2

    Which one of the following makes the author eliminate convergent evolution as a probable explanation for why white-lipped grove snails are found in Ireland and the Pyrenees?

    1. The absence of genetic variation between white-lipped grove snails of Ireland and the Pyrenees.
    2. The absence of genetic similarities between white-lipped grove snails of Ireland and snails from other parts of Europe, especially Britain.
    3. The coincidental evolution of similar traits (white-lipped shell) in the grove snails of Ireland and the Pyrenees.
    4. The distinct lineage of white-lipped grove snails found specifically in Ireland and the Pyrenees.
    Choice A
    The absence of genetic variation between white-lipped grove snails of Ireland and the Pyrenees.

  4. CAT 2018 Question Paper - Slot 2

    In paragraph 4, the evidence that “humans routinely ate these types of snails before the advent of agriculture” can be used to conclude that:

    1. white-lipped grove snails may have inadvertently traveled from the Pyrenees to Ireland on the floor of the small, coast-hugging skiffs that early seafarers used for travel.
    2. the seafarers who traveled from the Pyrenees to Ireland might have carried white-lipped grove snails with them as edibles.
    3. rivers and oceans in the Stone Age facilitated trade in white-lipped grove snails.
    4. 9,000 years ago, during the Stone Age, humans traveled from the South of France to Ireland via the Atlantic Ocean.
    Choice B
    the seafarers who traveled from the Pyrenees to Ireland might have carried white-lipped grove snails with them as edibles.

Meritocracy

The complexity of modern problems often precludes any one person from fully understanding them. Factors contributing to rising obesity levels, for example, include transportation systems and infrastructure, media, convenience foods, changing social norms, human biology and psychological factors. The multidimensional or layered character of complex problems also undermines the principle of meritocracy: the idea that the ‘best person’ should be hired. There is no best person. When putting together an oncological research team, a biotech company such as Gilead or Genentech would not construct a multiple-choice test and hire the top scorers, or hire people whose resumes score highest according to some performance criteria. Instead, they would seek diversity. They would build a team of people who bring diverse knowledge bases, tools and analytic skills.

Believers in a meritocracy might grant that teams ought to be diverse but then argue that meritocratic principles should apply within each category. Thus the team should consist of the ‘best’ mathematicians, the ‘best’ oncologists, and the ‘best’ biostatisticians from within the pool. That position suffers from a similar flaw.

Even with a knowledge domain, no test or criteria applied to individuals will produce the best team. Each of these domains possesses such depth and breadth, that no test can exist. Consider the field of neuroscience. Upwards of 50,000 papers were published last year covering various techniques, domains of enquiry and levels of analysis, ranging from molecules and synapses up through networks of neurons. Given that complexity, any attempt to rank a collection of neuroscientists from best to worst, as if they were competitors in the 50-metre butterfly, must fail. What could be true is that given a specific task and the composition of a particular team, one scientist would be more likely to contribute than another. Optimal hiring depends on context. Optimal teams will be diverse.

Evidence for this claim can be seen in the way that papers and patents that combine diverse ideas tend to rank as high-impact. It can also be found in the structure of the so-called random decision forest, a state-of-the-art machine-learning algorithm.

Random forests consist of ensembles of decision trees. If classifying pictures, each tree makes a vote: is that a picture of a fox or a dog? A weighted majority rules. Random forests can serve many ends. They can identify bank fraud and diseases, recommend ceiling fans and predict online dating behaviour. When building a forest, you do not select the best trees as they tend to make similar classifications. You want diversity. Programmers achieve that diversity by training each tree on different data, a technique known as bagging. They also boost the forest ‘cognitively’ by training trees on the hardest cases – those that the current forest gets wrong. This ensures even more diversity and accurate forests."

Yet the fallacy of meritocracy persists. Corporations, non-profits, governments, universities and even preschools test, score and hire the ‘best’. This all but guarantees not creating the best team. Ranking people by common criteria produces homogeneity. That’s not likely to lead to breakthroughs.

  1. CAT 2018 Question Paper - Slot 2

    The author critiques meritocracy for all the following reasons EXCEPT that:

    1. modern problems are multifaceted and require varied skill-sets to be solved.
    2. diversity and context-specificity are important for making major advances in any field.
    3. criteria designed to assess merit are insufficient to test expertise in any field of knowledge.
    4. an ideal team comprises of best individuals from diverse fields of knowledge.
    Choice D
    an ideal team comprises of best individuals from diverse fields of knowledge.

  2. CAT 2018 Question Paper - Slot 2

    Which of the following conditions would weaken the efficacy of a random decision forest?

    1. If a large number of decision trees in the ensemble were trained on data derived from easy cases.
    2. If a large number of decision trees in the ensemble were trained on data derived from easy and hard cases.
    3. If the types of ensembles of decision trees in the forest were doubled.
    4. If the types of decision trees in each ensemble of the forest were doubled.
    Choice A
    If a large number of decision trees in the ensemble were trained on data derived from easy cases.

  3. CAT 2018 Question Paper - Slot 2

    Which of the following conditions, if true, would invalidate the passage’s main argument?

    1. If assessment tests were made more extensive and rigorous.
    2. If top-scorers possessed multidisciplinary knowledge that enabled them to look at a problem from several perspectives.
    3. If it were proven that teams characterised by diversity end up being conflicted about problems and take a long time to arrive at a solution.
    4. If a new machine-learning algorithm were developed that proved to be more effective than the random decision forest.
    Choice B
    If top-scorers possessed multidisciplinary knowledge that enabled them to look at a problem from several perspectives.

  4. CAT 2018 Question Paper - Slot 2

    On the basis of the passage, which of the following teams is likely to be most effective in solving the problem of rising obesity levels?

    1. A team comprised of nutritionists, psychologists, urban planners and media personnel, who have each scored a distinction in their respective subject tests.
    2. A team comprised of nutritionists, psychologists, urban planners and media personnel, who have each performed well in their respective subject tests.
    3. A specialised team of nutritionists from various countries, who are also trained in the machine-learning algorithm of random decision forest.
    4. A specialised team of top nutritionists from various countries, who also possess some knowledge of psychology.
    Choice B
    A team comprised of nutritionists, psychologists, urban planners and media personnel, who have each performed well in their respective subject tests.

  5. CAT 2018 Question Paper - Slot 2

    Which of the following best describes the purpose of the example of neuroscience?

    1. In the modern age, every field of knowledge is so vast that a meaningful assessment of merit is impossible.
    2. Unlike other fields of knowledge, neuroscience is an exceptionally complex field, making a meaningful assessment of neuroscientists impossible.
    3. In narrow fields of knowledge, a meaningful assessment of expertise has always been possible.
    4. Neuroscience is an advanced field of science because of its connections with other branches of science like oncology and biostatistics.
    Choice A
    In the modern age, every field of knowledge is so vast that a meaningful assessment of merit is impossible.

Metric fixation

More and more companies, government agencies, educational institutions and philanthropic organisations are today in the grip of a new phenomenon: ‘metric fixation’. The key components of metric fixation are the belief that it is possible – and desirable – to replace professional judgment (acquired through personal experience and talent) with numerical indicators of comparative performance based upon standardised data (metrics); and that the best way to motivate people within these organisations is by attaching rewards and penalties to their measured performance.

The rewards can be monetary, in the form of pay for performance, say, or reputational, in the form of college rankings, hospital ratings, surgical report cards and so on. But the most dramatic negative effect of metric fixation is its propensity to incentivise gaming: that is, encouraging professionals to maximise the metrics in ways that are at odds with the larger purpose of the organisation. If the rate of major crimes in a district becomes the metric according to which police officers are promoted, then some officers will respond by simply not recording crimes or downgrading them from major offences to misdemeanours. Or take the case of surgeons. When the metrics of success and failure are made public – affecting their reputation and income – some surgeons will improve their metric scores by refusing to operate on patients with more complex problems, whose surgical outcomes are more likely to be negative. Who suffers? The patients who don’t get operated upon.

When reward is tied to measured performance, metric fixation invites just this sort of gaming. But metric fixation also leads to a variety of more subtle unintended negative consequences. These include goal displacement, which comes in many varieties: when performance is judged by a few measures, and the stakes are high (keeping one’s job, getting a pay rise or raising the stock price at the time that stock options are vested), people focus on satisfying those measures – often at the expense of other, more important organisational goals that are not measured. The best-known example is ‘teaching to the test’, a widespread phenomenon that has distorted primary and secondary education in the United States since the adoption of the No Child Left Behind Act of 2001.

Short-termism is another negative. Measured performance encourages what the US sociologist Robert K Merton in 1936 called ‘the imperious immediacy of interests where the actor’s paramount concern with the foreseen immediate consequences excludes consideration of further or other consequences’. In short, advancing short-term goals at the expense of long-range considerations. This problem is endemic to publicly traded corporations that sacrifice long-term research and development, and the development of their staff, to the perceived imperatives of the quarterly report.

  1. CAT 2018 Question Paper - Slot 2

    Of the following, which would have added the least depth to the author’s argument?

    1. An analysis of the reasons why metrics fixation is becoming popular despite its drawbacks.
    2. A comparative case study of metrics- and non-metrics-based evaluation, and its impact on the main goals of an organisation.
    3. More real-life illustrations of the consequences of employees and professionals gaming metrics-based performance measurement systems.
    4. Assessment of the pros and cons of a professional judgment-based evaluation system.
    Choice C
    More real-life illustrations of the consequences of employees and professionals gaming metrics-based performance measurement systems.

  2. CAT 2018 Question Paper - Slot 2

    Which of the following is NOT a consequence of the 'metric fixation' phenomenon mentioned in the passage?

    1. Short-term orientation induced by frequent measurement of performance.
    2. Finding a way to show better results without actually improving performance.
    3. Improving cooperation among employees leading to increased organisational effectiveness in the long run.
    4. Deviating from organisationally important objectives to measurable yet less important objectives.
    Choice C
    Improving cooperation among employees leading to increased organisational effectiveness in the long run.

  3. CAT 2018 Question Paper - Slot 2

    What main point does the author want to convey through the examples of the police officer and the surgeon?

    1. Some professionals are likely to be significantly influenced by the design of performance measurement systems.
    2. Metrics-linked rewards may encourage unethical behaviour among some professionals.
    3. The actions of police officers and surgeons have a significantly impact on society.
    4. Critical public roles should not be evaluated on metrics-based performance measures.
    Choice B
    Metrics-linked rewards may encourage unethical behaviour among some professionals.

  4. CAT 2018 Question Paper - Slot 2

    All of the following can be a possible feature of the No Child Left Behind Act of 2001, EXCEPT:

    1. school funding and sanctions are tied to yearly improvement shown on tests.
    2. standardised test scores can be critical in determining a student’s educational future.
    3. the focus is more on test-taking skills than on higher order thinking and problem-solving.
    4. assessment is dependent on the teacher's subjective evaluation of students' class participation.
    Choice D
    assessment is dependent on the teacher's subjective evaluation of students' class participation.

  5. CAT 2018 Question Paper - Slot 2

    What is the main idea that the author is trying to highlight in the passage?

    1. Long-term organisational goals should not be ignored for short-term measures of organisational success.
    2. All kinds of organisations are now relying on metrics to measure performance and to give rewards and punishments.
    3. Evaluating performance by using measurable performance metrics may misguide organisational goal achievement.
    4. Performance measurement needs to be precise and cost-effective to be useful for evaluating organisational performance.
    Choice C
    Evaluating performance by using measurable performance metrics may misguide organisational goal achievement.

Rings of Satrun

NOT everything looks lovelier the longer and closer its inspection. But Saturn does. It is gorgeous through Earthly telescopes. However, the 13 years of close observation provided by Cassini, an American spacecraft, showed the planet, its moons and its remarkable rings off better and better, revealing finer structures, striking novelties and greater drama.

By and large the big things in the solar system—planets and moons—are thought of as having been around since the beginning. The suggestion that rings and moons are new is, though, made even more interesting by the fact that one of those moons, Enceladus, is widely considered the most promising site in the solar system on which to look for alien life. If Enceladus is both young and bears life, that life must have come into being quickly. This is also believed to have been the case on Earth. Were it true on Enceladus, that would encourage the idea that life evolves easily when conditions are right.

One reason for thinking Saturn’s rings are young is that they are bright. The solar system is suffused with comet dust, and comet dust is dark. Leaving Saturn’s ring system (which Cassini has shown to be more than 90% water ice) out in such a mist is like leaving laundry hanging on a line downwind from a smokestack: it will get dirty. The lighter the rings are, the faster this will happen, for the less mass they contain, the less celestial pollution they can absorb before they start to discolour Jeff Cuzzi, a scientist at America’s space agency, NASA, who helped run Cassini, told the Lunar and Planetary Science Conference in Houston that combining the mass estimates with Cassini’s measurements of the density of comet-dust near Saturn suggests the rings are no older than the first dinosaurs, nor younger than the last of them—that is, they are somewhere between 200m and 70m years old.

That timing fits well with a theory put forward in 2016, by Matija Cuk of the SETI Institute, in California and his colleagues. They suggest that at around the same time as the rings came into being an old set of moons orbiting Saturn destroyed themselves, and from their remains emerged not only the rings but also the planet’s current suite of inner moons—Rhea, Dione, Tethys, Enceladus and Mimas.

Dr Cuk and his colleagues used computer simulations of Saturn’s moons’ orbits as a sort of time machine. Looking at the rate at which tidal friction is causing these orbits to lengthen they extrapolated backwards to find out what those orbits would have looked like in the past. They discovered that about 100m years ago the orbits of two of them, Tethys and Dione, would have interacted in a way that left the planes in which they orbit markedly tilted. But their orbits are untilted. The obvious, if unsettling, conclusion was that this interaction never happened—and thus that at the time when it should have happened, Dione and Tethys were simply not there. They must have come into being later.

  1. CAT 2018 Question Paper - Slot 2

    The phrase “leaving laundry hanging on a line downwind from a smokestack” is used to explain how the ringed planet’s:

    1. atmosphere absorbs comet dust.
    2. moons create a gap between the rings.
    3. rings discolour and darken over time.
    4. rings lose mass over time.
    Choice C
    rings discolour and darken over time.

  2. CAT 2018 Question Paper - Slot 2

    Data provided by Cassini challenged the assumption that:

    1. all big things in the solar system have been around since the beginning.
    2. new celestial bodies can form from the destruction of old celestial bodies
    3. Saturn’s ring system is composed mostly of water ice.
    4. there was life on earth when Saturn’s rings were being formed.
    Choice A
    all big things in the solar system have been around since the beginning.

  3. CAT 2018 Question Paper - Slot 2

    Based on information provided in the passage, we can infer that, in addition to water ice, Saturn’s rings might also have small amounts of:

    1. methane and rock particles.
    2. rock particles and comet dust.
    3. helium and methane.
    4. helium and comet dust.
    Choice B
    rock particles and comet dust.

  4. CAT 2018 Question Paper - Slot 2

    The main objective of the passage is to:

    1. provide evidence that Saturn’s rings and moons are recent creations.
    2. highlight the beauty, finer structures and celestial drama of Saturn’s rings and moons.
    3. demonstrate how the orbital patterns of Saturn’s rings and moons change over time.
    4. establish that Saturn’s rings and inner moons have been around since the beginning of time.
    Choice A
    provide evidence that Saturn’s rings and moons are recent creations.

  5. CAT 2018 Question Paper - Slot 2

    Based on information provided in the passage, we can conclude all of the following EXCEPT:

    1. Saturn’s lighter rings discolour faster than rings with greater mass.
    2. Thethys and Dione are less than 100 million years old.
    3. none of Saturn’s moons ever had suitable conditions for life to evolve.
    4. Saturn’s rings were created from the remains of older moons.
    Choice C
    none of Saturn’s moons ever had suitable conditions for life to evolve.

CAT VARC : CAT 2017 Question Paper Slot 1

Passage 1: Map Making

Understanding where you are in the world is a basic survival skill, which is why we, like most species come hard-wired with specialized brain areas to create cognitive maps of our surroundings. Where humans are unique, though, with the possible exception of honeybees, is that we try to communicate this understanding the world with others. We have along history of doing this by drawing maps – the earliest version yet discovered were scrawled on cave walls 14,000 years ago. Human cultures have been drawing them on stone tablets, papyrus, paper and now computer screens ever since.

Given such a long history of human map-making, it perhaps surprising that is only within the last few hundred years that north has been consistently considered to be at the top. In fact, for much of human history, north almost never appeared at the top, according to Jerry Brotton, a map historian... “North was rarely put at the top for the simple fact that north is where darkness comes from,” he says. “West is also very unlikely o be put at the top because west is where the sun disappears.”

Confusingly, early Chinese maps seem to buck this trend. But, Brotton, says, even though they did have compasses at the time, that isn’t the reason that they placed north at the top. Early Chinese compasses were actually oriented to point south, which was considered to be more desirable than deepest darkest north. But in Chinese maps, the emperor, who lived in the north of the country was always put at the top of the map, with everyone else, his loyal subjects, looking up towards him. “In Chinese culture the Emperor looks south because it’s where the winds come from, it’s a good direction. North is not very good but you are in a position of the subjection to the emperor, so you look up to him,” says Brotton.

Given that each culture has a very different idea of who, or what, they should look upto it’s perhaps not surprising that there is very little consistency in which way early maps pointed. In ancient Egyptian times the top of the world was east, the position of sunrise. Early Islamic maps favoured south at the top because most of the early Muslim cultures were north of Mecca, so they imagined looking up (south) towards it Christian maps from the same era (called Mappa Mundi) put east at the top, towards the Garden of Eden and with Jerusalem in the centre.

So when did everyone get together and decide that north was the top? It’s tempting to put it down to European explorers like Christopher Columbus and Ferdinand Megellan who were navigating by the North Star. But Brotton argues that these early explorers didn’t think of the world like that at all. “When Columbus describes the world it is in accordance with east being at the top,” he says “Columbus says he is going towards paradise, so his mentality is from a medieval mappa mundi.” We’ve got to remember, adds Brotton, that at the time, “no one knows what they are doing and where they are going.”

  1. CAT 2017 Question Paper - Slot 1

    Which one of the following best describes what the passage is trying to do?

    1. It questions on explanation about how maps are designed.
    2. It corrects a misconception about the way maps are designed.
    3. It critiques a methodology used to create maps.
    4. It explores some myths about maps.
    Choice B
    It corrects a misconception about the way maps are designed.

  2. CAT 2017 Question Paper - Slot 1

    Early maps did NOT put north at the top for all the following reasons EXCEPT

    1. North was the source of darkness
    2. South was favoured by some emperors
    3. East and south were more important for religious reasons for some civilisations
    4. East was considered by some civilisations to be a more positive direction
    Choice B
    South was favoured by some emperors

  3. CAT 2017 Question Paper - Slot 1

    According to the passage, early Chinese maps placed north at the top because

    1. the Chinese invented the compass and were aware of magnetic north.
    2. they wanted to show respect to the emperor.
    3. the Chinese emperor appreciated the winds from the south.
    4. north was considered the most desirable direction.
    Choice B
    they wanted to show respect to the emperor.

  4. CAT 2017 Question Paper - Slot 1

    It can be inferred from the passage that European explorers like Columbus and Megellan

    1. set the precedent for north-up maps.
    2. navigated by the compass.
    3. used an eastward orientation for religious reasons.
    4. navigated with the help of early maps.
    Choice C
    used an eastward orientation for religious reasons.

  5. CAT 2017 Question Paper - Slot 1

    Which one of the following about the northern orientation of modern maps is asserted in the passage

    1. The biggest contributory factor was the understanding of magnetic north
    2. The biggest contributory factor was the role of European explorers
    3. The biggest contributory factor was the influence of Christian maps
    4. The biggest contributory factor is not stated in the passage
    Choice D
    The biggest contributory factor is not stated in the passage

  6. CAT 2017 Question Paper - Slot 1

    The role of natural phenomena in influencing map-making conventions is seen most clearly in

    1. early Egyptian maps
    2. early Islamic maps
    3. early Chinese maps
    4. early Christian maps
    Choice A
    early Egyptian maps

Passage 2: Impact of Printed text and iPhone

I used a smartphone GPS to find my way through the cobblestoned maze of Geneva's Old Town, in search of a handmade machine that changed the world more than any other invention. Near a 13th-century cathedral in this Swiss city on the shores of a lovely lake, I found what I was looking for: a Gutenberg printing press. "This was the Internet of its day — at least as influential as the iPhone," said Gabriel de Montmollin, the director of the Museum of the Reformation, toying with the replica of Johann Gutenberg's great invention. [Before the invention of the printing press] it used to take four monks...up to a year to produce a single book. With the advance in movable type in 15th-century Europe, one press could crank out 3,000 pages a day.

Before long, average people could travel to places that used to be unknown to them — with maps! Medical information passed more freely and quickly, diminishing the sway of quacks...The printing press offered the prospect that tyrants would never be able to kill a book or suppress an idea. Gutenberg's brainchild broke the monopoly that clerics had on scripture. And later, stirred by pamphlets from a version of that same press, the American colonies rose up against a king and gave birth to a nation. So, a question in the summer of this 10th anniversary of the iPhone: has the device that is perhaps the most revolutionary of all time given us a single magnificent idea? Nearly every advancement of the written word through new technology has also advanced humankind. Sure, you can say the iPhone changed everything. By putting the world's recorded knowledge in the palm of a hand, it revolutionized work, dining, travel and socializing. It made us more narcissistic — here's more of me doing cool stuff! — and it unleashed an army of awful trolls. We no longer have the patience to sit through a baseball game without that reach to the pocket. And one more casualty of Apple selling more than a billion phones in a decade's time: daydreaming has become a lost art.

For all of that, I'm still waiting to see if the iPhone can do what the printing press did for religion and democracy...the Geneva museum makes a strong case that the printing press opened more minds than anything else...it's hard to imagine the French or American revolutions without those enlightened voices in print...

Not long after Steve Jobs introduced his iPhone, he said the bound book was probably headed for history's attic. Not so fast. After a period of rapid growth in e-books, something closer to the medium for Chaucer's volumes has made a great comeback.

The hope of the iPhone, and the Internet in general, was that it would free people in closed societies. But the failure of the Arab Spring, and the continued suppression of ideas in North Korea, China and Iran, has not borne that out... The iPhone is still young. It has certainly been "one of the most important, world-changing and successful products in history, “ as Apple CEO. Tim Cook said. But I'm not sure if the world changed for the better with the iPhone — as it did with the printing press — or merely, changed.

  1. CAT 2017 Question Paper - Slot 1

    The printing press has been likened to the Internet for which one of the following reasons?

    1. It enabled rapid access to new information and the sharing of new ideas
    2. It represented new and revolutionary technology compared to the past
    3. It encouraged reading among people by giving them access to thousands of books
    4. It gave people access to pamphlets and literature in several languages
    Choice A
    It enabled rapid access to new information and the sharing of new ideas

  2. CAT 2017 Question Paper - Slot 1

    According to the passage, the invention of the printing press did all of the following EXCEPT

    1. Promoted the spread of enlightened political views across countries.
    2. Gave people direct access to authentic medical information and religious texts.
    3. Shortened the time taken to produce books and pamphlets.
    4. Enabled people to perform various tasks simultaneously.
    Choice D
    Enabled people to perform various tasks simultaneously.

  3. CAT 2017 Question Paper - Slot 1

    Steve Jobs predicted which one'of the following with the introduction of the iPhone?

    1. People would switch from reading on the Internet to reading on their iPhones.
    2. People would lose interest in historical and traditional classics.
    3. Reading printed books would become a thing of the past.
    4. The production of e-books would eventually fall.
    Choice C
    Reading printed books would become a thing of the past.

  4. CAT 2017 Question Paper - Slot 1

    "I'm still waiting to see if the iPhone can do what the printing press did for religion and democracy." The author uses which one of the following to indicate his uncertainty?

    1. The rise of religious groups in many parts of the world.
    2. The expansion in trolling and narcissism among users of the Internet.
    3. The continued suppression of free speech in closed societies.
    4. The decline in reading habits among those who use the device.
    Choice C
    The continued suppression of free speech in closed societies.

  5. CAT 2017 Question Paper - Slot 1

    The author attributes the French and American revolutions to the invention of the printing press because

    1. maps enabled large numbers of Europeans to travel and settle in the American continent.
    2. the rapid spread of information exposed people to new ideas on freedom and democracy.
    3. it encouraged religious freedom among the people by destroying the monopoly of religious leaders on the scriptures.
    4. it made available revolutionary strategies and opinions to the people.
    Choice B
    the rapid spread of information exposed people to new ideas on freedom and democracy.

  6. CAT 2017 Question Paper - Slot 1

    The main conclusion of the passage is that the new technology has

    1. some advantages, but these are outweighed by its disadvantages.
    2. so far not proved as successful as the printing press in opening people's minds.
    3. been disappointing because it has changed society too rapidly.
    4. been more wasteful than the printing press because people spend more time daydreaming or surfing.
    Choice B
    so far not proved as successful as the printing press in opening people's minds.

Passage 3: American Malls

This year alone, more than 8,600 stores could close, according to industry estimates, many of them the brand -name anchor outlets that real estate developers once stumbled over themselves to court. Already there have been 5,300 retail closings this year... Sears Holdings—which owns Kmart—said in March that there's "substantial doubt" it can stay in business altogether, and will close 300 stores this year. So far this year, nine national retail chains have filed for bankruptcy.

Local jobs are a major casualty of what analysts are calling, with only a hint of hyperbole, the retail apocalypse. Since 2002, department stores have lost 448,000 jobs, a 25% decline, while the number of store closures this year is on pace to surpass the worst depths of the Great Recession. The growth of online retailers, meanwhile, has failed to offset those losses, with the ecommerce sector adding just 178,000 jobs over the past 15 years. Some of those jobs can be found in the massive distribution centers Amazon has opened across the country, often not too far from malls the company helped shutter.

But those are workplaces, not gathering places. The mall is both. And in the 61 years since the first enclosed one opened in suburban Minneapolis, the shopping mall has been where a huge swath of middle-class America went for far more than shopping. It was the home of first jobs and blind dates, the place for family photos and ear piercings, where goths and grandmothers could somehow walk through the same doors and find something they all liked. Sure, the food was lousy for you and the oceans of parking lots encouraged car-heavy development, something now scorned by contemporary planners. But for better or worse, the mall has been America's public square for the last 60 years.

So what happens when it disappears?

Think of your mall. Or think of the one you went to as a kid. Think of the perfume clouds in the department stores. The fountains splashing below the skylights. The cinnamon wafting from the food court. As far back as ancient Greece, societies have congregated around a central marketplace. In medieval Europe, they were outside cathedrals. For half of the 20th century and almost 20 years into the new one, much of America has found their agora on the terrazzo between Orange Julius and Sbarro, Waldenbooks and the Gap, Sunglass Hut and Hot Topic.

That mall was an ecosystem unto itself, a combination of community and commercialism peddling everything you needed and everything you didn't: Magic Eye posters, wind catchers. Air Jordans....

A growing number of Americans, however, don't see the need to go to any Macy's at all. Our digital lives are frictionless and ruthlessly efficient, with retail and romance available at a click. Malls were designed for leisure, abundance, ambling. You parked and planned to spend some time. Today, much of that time has been given over to busier lives and second jobs and apps that let you swipe right instead of haunt the food court. ' Malls, says Harvard business professor Leonard Schlesinger, "were built for patterns of social interaction that increasingly don't exist."

  1. CAT 2017 Question Paper - Slot 1

    The central idea of this passage is that:

    1. the closure of mails has affected the economic and social life of middle-class America.
    2. the advantages of malls outweigh their disadvantages.
    3. malls used to perform a social function that has been lost.
    4. malls are closing down because people have found alternate ways to shop.
    Choice C
    malls used to perform a social function that has been lost.

  2. CAT 2017 Question Paper - Slot 1

    Why does the author say in paragraph 2, 'the massive distribution centers Amazon has opened across the country, often not too far from malls the company helped shutter'?

    1. To highlight the irony of the situation.
    2. To indicate that mails and distribution centres are located in the same area.
    3. To show that Amazon is helping certain brands go online.
    4. To indicate that the shopping habits of the American middle class have changed.
    Choice A
    To highlight the irony of the situation.

  3. CAT 2017 Question Paper - Slot 1

    In paragraph 1, the phrase "real estate developers once stumbled over themselves to court" suggests that they

    1. took brand-name anchor outlets to court.
    2. no longer pursue brand-name hanger outlets.
    3. malls are closing down because people have found alternate ways to shop.
    4. collaborated with one another to get brand-name anchor outlets.
    Choice B
    no longer pursue brand-name hanger outlets.

  4. CAT 2017 Question Paper - Slot 1

    The author calls the mall an ecosystem unto itself because

    1. people of all ages and from all walks of life went there.
    2. people could shop as well as eat in one place.
    3. it was a commercial space as well as a gathering place.
    4. it sold things that were needed as well as those that were not.
    Choice C
    it was a commercial space as well as a gathering place.

  5. CAT 2017 Question Paper - Slot 1

    Why does the author say that the mall has been America's public square?

    1. Malls did not bar anybody from entering the space.
    2. Malls were a great place to shop for a huge section of the middle class.
    3. Malls were a hangout place where families grew close to each other.
    4. Malls were a great place for everyone to gather and interact.
    Choice D
    Malls were a great place to shop for a huge section of the middle class.

  6. CAT 2017 Question Paper - Slot 1

    The author describes 'Perfume clouds in the department stores' in order to

    1. evoke memories by painting a picture of malls
    2. describe the smells and sights of mails
    3. emphasise that all brands were available under one roof.
    4. show that malls smelt good because of the various stores and food court.
    Choice A
    evoke memories by painting a. picture of mails

Passage 4: Evolutionary diversity

Scientists have long recognised the incredible diversity within a species. But they thought it reflected evolutionary changes that unfolded imperceptibly, over millions of years. That divergence between populations within a species was enforced, according to Ernst Mayr, the great evolutionary biologist of the 1940s, when a population was separated from the rest of the species by a mountain range or a desert, preventing breeding across the divide over geologic scales of time. Without the separation, gene flow was relentless. But as the separation persisted, the isolated population grew apart and speciation occurred.

In the mid-1960s, the biologist Paul Ehrlich - author of The Population Bomb (1968) - and his Stanford University colleague Peter Raven challenged Mayr's ideas about speciation. They had studied checkerspot butterflies living in the Jasper Ridge Biological Preserve in California, and it soon became clear that they were not examining a single population. Through years of capturing, marking and then recapturing the butterflies, they were able to prove that within the population, spread over just 50 acres of suitable checkerspot habitat, there were three groups that rarely interacted despite their very close proximity.

Among other ideas, Ehrlich and Raven argued in a now classic paper from 1969 that gene flow was not as predictable and ubiquitous as Mayr and his cohort maintained, and thus evolutionary divergence between neighbouring groups in a population was probably common. They also asserted that isolation and gene flow were less important to evolutionary divergence than natural selection (when factors such as mate choice, weather, disease or predation cause better-adapted individuals to survive and pass on their successful genetic traits). For example, Ehrlich and Raven suggested that, without the force of natural selection, an isolated population would remain unchanged and that, in other scenarios, natural selection could be strong enough to overpower gene flow...

  1. CAT 2017 Question Paper - Slot 1

    Which of the following best sums up Ehrlich and Raven's argument in their classic 1969 paper?

    1. Ernst Mayr was wrong in identifying physical separation as the cause of species diversity
    2. Checkerspot butterflies in the 50-acre Jasper Ridge Preserve formed three groups that rarely interacted with each other
    3. While a factor, isolation was not as important to speciation as natural selection
    4. Gene flow is less common and more erratic than Mayr and his colleagues claimed.
    Choice C
    While a factor, isolation was not as important to speciation as natural selection

  2. CAT 2017 Question Paper - Slot 1

    All of the following statements are true according to the passage EXCEPT

    1. Gene flow contributes to evolutionary divergence.
    2. The Population Bomb questioned dominant ideas about species diversity.
    3. Evolutionary changes unfold imperceptibly over time.
    4. Checkerspot butterflies are known to exhibit speciation while living in close proximity.
    Choice B
    The Population Bomb questioned dominant ideas about species diversity.

  3. CAT 2017 Question Paper - Slot 1

    The author discusses Mayr, Ehrlich and Raven to demonstrate that

    1. evolution is a sensitive and controversial topic.
    2. Ehrlich and Raven's ideas about evolutionary divergence are widely accepted by scientists.
    3. the causes of speciation are debated by scientists.
    4. checkerspot butterflies offer the best example of Ehrlich and Raven's ideas about speciation.
    Choice C
    the causes of speciation are debated by scientists.

Passage 5: Olympics

Do sports mega events like the summer Olympic Games benefit the host city economically? It depends, but the prospects are less than rosy. The trick is converting...several billion dollars in operating costs during the 17-day fiesta of the Games into a basis for long-term economic returns. These days, the summer Olympic Games themselves generate total revenue of $4 billion to $5 billion, but the lion's share of this goes to the International Olympics Committee, the National Olympics Committees and the International Sports Federations. Any economic benefit would have to flow from the value of the Games as an advertisement for the city, the new transportation and communications infrastructure that was created for the Games, or the ongoing use of the new facilities.

Evidence suggests that the advertising effect is far from certain. The infrastructure benefit depends on the initial condition of the city and the effectiveness of the planning. The facilities benefit is dubious at best for buildings such as velodromes or natatoriums and problematic for 100,000-seat Olympic stadiums. The latter require a conversion plan for future use, the former are usually doomed to near vacancy. Hosting the summer Games generally requires 30-plus sports venues and dozens of training centers. Today, the Bird's Nest in Beijing sits virtually empty, while the Olympic Stadium in Sydney costs some $30 million a year to operate.

Part of the problem is that Olympics planning takes place in a frenzied and time-pressured atmosphere of intense competition with the other prospective host cities — not optimal conditions for contemplating the future shape of an urban landscape. Another part of the problem is that urban land is generally scarce and growing scarcer. The new facilities often stand for decades or longer. Even if they have future use, are they the best use of precious urban real estate?

Further, cities must consider the human cost. Residential areas often are razed and citizens relocated (without adequate preparation or compensation). Life is made more hectic and congested. There are, after all, other productive uses that can be made of vanishing fiscal resources.

  1. CAT 2017 Question Paper - Slot 1

    The central point in the first paragraph is that the economic benefits of the Olympic Games

    1. are shared equally among the three organising committees.
    2. accrue mostly through revenue from advertisements and ticket sales.
    3. accrue to host cities, if at all, only in the long term.
    4. are usually eroded by expenditure incurred by the host city.
    Choice C
    accrue to host cities, if at all, only in the long term.

  2. CAT 2017 Question Paper - Slot 1

    Sports facilities built for the Olympics are not fully utilised after the Games are over because

    1. their scale and the costs of operating them are large.
    2. their location away from the city centre usually limits easy access.
    3. the authorities do not adapt them to local conditions.
    4. they become outdated having being built with little planning and under time pressure.
    Choice A
    their scale and the costs of operating them are large.

  3. CAT 2017 Question Paper - Slot 1

    The author feels that the Games place a burden on the host city for all of the following reasons EXCEPT that

    1. they divert scarce urban land from more productive uses.
    2. they involve the demolition of residential structures to accommodate sports facilities and infrastructure.
    3. the finances used to fund the Games could be better used for other purposes.
    4. the influx of visitors during the Games places a huge strain on the urban infrastructure.
    Choice D
    the influx of visitors during the Games places a huge strain on the urban infrastructure.

CAT VARC : CAT 2017 Question Paper Slot 2

Passage 1: Creativity

Creativity is at once our most precious resource and our most inexhaustible one. As anyone who has ever spent any time with children knows, every single human being is born creative; every human being is innately endowed with the ability to combine and recombine data, perceptions, materials and ideas, and devise new ways of thinking and doing.What fosters creativity? More than anything else: the presence of other creative people. The big myth is that creativity is the province of great individual gen.iuses. In. fact creativity is a social process. Our biggest creative breakthroughs come when people learn from, compete with, and collaborate with other people.

Cities are the true fonts of creativity... With their diverse populations, dense social networks, and public spaces where people can meet spontaneously and serendipitously, they spark and catalyze new ideas. With their infrastructure for finance, organization and trade, they allow those ideas to be swiftly actualized.

As for what staunches creativity, that's easy, if ironic. It's the very institutions that we build to manage, exploit and perpetuate the fruits of creativity — our big bureaucracies, and sad to say, too many of our schools. Creativity is disruptive; schools and organizations are regimented, standardized and stultifying.

The education expert Sir Ken Robinson points to a 1968 study reporting on a group of 1,600 children who were tested over time for their ability to think in out-of-the-box ways. When the children were between 3 and 5 years old, 98 percent achieved positive scores. When they were 8 to 10, only 32 percent passed the same test, and only 10 percent at 13 to 15. When 280,000 25-year-olds took the test, just 2 percent passed. By the time we are adults, our creativity has been wrung out of us.

I once asked the great urbanist Jane Jacobs what makes some places more creative than others. She said, essentially, that the question was an easy one. All cities, she said, were filled with creative people; that's our default state as people. But some cities had more than their shares of leaders, people and institutions that blocked out that creativity. She called them "squelchers."

Creativity (or the lack of it) follows the same general contours of the great socio-economic divide — our rising inequality — that plagues us. According to my own estimates, roughly a third of us across the United States, and perhaps as much as half of us in our most creative cities — are able to do work which engages our creative faculties to some extent, whether as artists, musicians, writers, techies, innovators, entrepreneurs, doctors, lawyers, journalists or educators — those of us who work with our minds. That leaves a group that I term "the other 66 percent," who toil in low-wage rote and rotten jobs — if they have jobs at all — in which their creativity is subjugated, ignored or wasted.

Creativity itself is not in danger. It's flourishing is all around us — in science and technology, arts and culture, in our rapidly revitalizing cities. But we still have a long way to go if we want to build a truly creative society that supports and rewards the creativity of each and every one of us.

  1. CAT 2017 Question Paper - Slot 2

    In the author's view, cities promote human creativity for all the following reasons EXCEPT that they

    1. contain spaces that enable people to meet and share new ideas.
    2. expose people to different and novel ideas, because they are home to varied groups of people.
    3. provide the financial and institutional networks that enable ideas to become reality.
    4. provide access to cultural activities that promote new and creative ways of thinking.
    Choice D
    provide access to cultural activities that promote new and creative ways of thinking.

  2. CAT 2017 Question Paper - Slot 2

    The author uses 'ironic' in the third paragraph to point out that

    1. people need social contact rather than isolation to nurture their creativity.
    2. institutions created to promote creativity eventually stifle it.
    3. the larger the creative population in a city, the more likely it is to be stifled.
    4. large bureaucracies and institutions are the inevitable outcome of successful cities.
    Choice B
    institutions created to promote creativity eventually stifle it.

  3. CAT 2017 Question Paper - Slot 2

    The central idea of this passage is that

    1. social interaction is necessary to nurture creativity.
    2. creativity and ideas are gradually declining in all societies.
    3. the creativity divide is widening in societies in line with socio-economic trends.
    4. more people should work in jobs that engage their creative faculties.
    Choice A
    social interaction is necessary to nurture creativity.

  4. CAT 2017 Question Paper - Slot 2

    Jane Jacobs believed that cities that are more creative

    1. have to struggle to retain their creativity.
    2. have to 'squelch' unproductive people and promote creative ones.
    3. have leaders and institutions that do not block creativity.
    4. typically do not start off as creative hubs.
    Choice C
    have leaders and institutions that do not block creativity.

  5. CAT 2017 Question Paper - Slot 2

    The 1968 study is used here to show that

    1. as they get older, children usually learn to be more creative.
    2. schooling today does not encourage creative thinking in children.
    3. the more children learn, the less creative they become.
    4. technology today prevents children from being creative.
    Choice B
    schooling today does not encourage creative thinking in children.

  6. CAT 2017 Question Paper - Slot 2

    The author's conclusions about the most 'creative cities' in the US (paragraph 6) are based on his assumption that

    1. people who work with their hands are not doing creative work
    2. more than half the population works in non-creative jobs.
    3. only artists, musicians, writers, and so on should be valued in a society.
    4. most cities ignore or waste the creativity of low-wage workers.
    Choice A
    people who work with their hands are not doing creative work

CAT VARC : CAT 2017 Question Paper Slot 2

Passage 2: Subnivium

During the frigid season... it's often necessary to nestle under a blanket to try to stay warm. The temperature difference between the blanket and the air outside is so palpable that we often have trouble leaving our warm refuge. Many plants and animals similarly hunker down, relying on snow cover for safety from winter's harsh conditions. The small area between the snowpack and the ground, called the subnivium... might be the most important ecosystem that you have never heard of.

The subnivium is so well-insulated and stable that its temperature holds steady at around 32 degree Fahrenheit (0 degree Celsius). Although that might still sound cold, a constant temperature of 32 degree Fahrenheit can often be 30 to 40 degrees warmer than the air temperature during the peak of winter. Because of this large temperature difference, a wide variety of species...depend on the subnivium for winter protection.

For many organisms living in temperate and Arctic regions, the difference between being under the snow or outside it is a matter of life and death. Consequently, disruptions to the subnivium brought about by climate change will affect everything from population dynamics to nutrient cycling through the ecosystem.

The formation and stability of the subnivium requires more than a few flurries. Winter ecologists have suggested that eight inches of snow is necessary to develop a stable layer of insulation. Depth is not the only factor, however. More accurately, the stability of the subnivium depends on the interaction between snow depth and snow density. Imagine being under a stack of blankets that are all flattened and pressed together. When compressed, the blankets essentially form one compacted layer. In contrast, when they are lightly placed on top of one another, their insulative capacity increases because the air pockets between them trap heat. Greater depths of low-density snow are therefore better at insulating the ground.

Both depth and density of snow are sensitive to temperature. Scientists are now beginning to explore how climate change will affect the subnivium, as well as the species that depend on it. At first glance, warmer winters seem beneficial for species that have difficulty surviving subzero temperatures; however, as with most ecological phenomena, the consequences are not so straightforward. Research has shown that the snow season (the period when snow is more likely than rain) has become shorter since l970. When rain falls on snow, it increases the density of the snow and reduces its insulative capacity. Therefore, even though winters are expected to become warmer overall from future climate change, the subnivium will tend to become colder and more variable with less protection from the above-ground temperatures.

The effects of a colder subnivium are complex... For example, shrubs such as crowberry and alpine azalea that grow along the forest floor tend to block the wind and so retain higher depths of snow around them. This captured snow helps to keep soils insulated and in turn increases plant decomposition and nutrient release. In field experiments, researchers removed a portion. of the snow cover to investigate the importance of the subnivium's insulation. They found that soil frost in the snow-free area resulted in damage to plant roots and sometimes even the death of the plant.

  1. CAT 2017 Question Paper - Slot 2

    The purpose of this passage is to

    1. introduce readers to a relatively unknown ecosystem: the subnivium.
    2. explain how the subnivium works to provide shelter and food to several species.
    3. outline the effects of climate change on the subnivium.
    4. draw an analogy between the effect of blankets on humans and of snow cover on species living in the subnivium.
    Choice C
    outline the effects of climate change on the subnivium.

  2. CAT 2017 Question Paper - Slot 2

    All of the following statements are true EXCEPT

    1. Snow depth and Snow density both influence the stability of the subnivium.
    2. Climate change has some positive effects on the subnivium.
    3. The subnivium maintains a steady temperature that can be 30 to 40 degrees warmer than the winter air temperature.
    4. Researchers have established the adverse effects of dwindling snow cover on the subnivium.
    Choice B
    Climate change has some positive effects on the subnivium.

  3. CAT 2017 Question Paper - Slot 2

    Based on this extract, the author would support which one of the following actions?

    1. The use of snow machines in winter to ensure snow cover of at least eight inches.
    2. Government action to curb climate change.
    3. Adding nutrients to the soil in winter.
    4. Planting more shrubs in areas of short snow season.
    Choice B
    Government action to curb climate change.

  4. CAT 2017 Question Paper - Slot 2

    In paragraph 6, the author provides the examples of crowberry and alpine azalea to demonstrate that

    1. Despite frigid temperatures, several species survive in temperate and Arctic regions.
    2. Due to frigid temperatures in the temperate and Arctic regions, plant species that survive tend to be shrubs rather than trees.
    3. The crowberry and alpine azalea are abundant in temperate and Arctic regions.
    4. The stability of the subnivium depends on several interrelated factors, including shrubs on the forest floor.
    Choice D
    The stability of the subnivium depends on several interrelated factors, including shrubs on the forest floor.

  5. CAT 2017 Question Paper - Slot 2

    Which one of the following statements can be inferred from the passage?

    1. In an ecosystem, altering any one element has a ripple effect on all others.
    2. Climate change affects temperate and Artie regions more than equatorial or arid ones.
    3. A compact layer of wool is warmer than a similarly compact layer of goose down.
    4. The loss of the subnivium, while tragic, will affect only temperate and Artic regions.
    Choice A
    In an ecosystem, altering any one element has a ripple effect on all others.

  6. CAT 2017 Question Paper - Slot 2

    In paragraph 1, the author uses blankets as a device to

    1. evoke the bitter cold of winter in the minds of readers.
    2. explain how blankets work to keep us warm.
    3. draw an analogy between blankets and the snow pack.
    4. alert readers to the fatal effects of excessive exposure to the cold
    Choice C
    draw an analogy between blankets and the snow pack.

CAT VARC : CAT 2017 Question Paper Slot 2

Passage 3: Electric Vehicles

The end of the age of the internal combustion engine is in sight. There are small signs everywhere: the shift to hybrid vehicles is already under way among manufacturers. Volvo has announced it will make no purely petrol-engined cars after 2019...and Tesla has just started selling its first electric car aimed squarely at the middle classes: the Tesla 3 sells for $35,000 in the US, and 400,000 people have put down a small, refundable deposit towards one. Several thousand have already taken delivery, and the company hopes to sell half a million more next year. This is a remarkable figure for a machine with a fairly short range and a very limited number of specialised charging stations.

Some of it reflects the remarkable abilities of Elon Musk, the company's founder, as a salesman, engineer, and a man able to get the most out his factory workers and the governments he deals with...Mr Musk is selling a dream that the world wants to believe in. This last may be the most important factor in the story. The private car is...a device of immense practical help and economic significance, but at the same time a theatre for myths of unattainable selffulfilment. The one thing you will never see in a car advertisement is traffic, even though that is the element in which drivers spend their lives. Every single driver in a traffic jam is trying to escape from it, yet it is the inevitable consequence of mass car ownership.

The sleek and swift electric car is at one level merely the most contemporary fantasy of autonomy and power. But it might also disrupt our exterior landscapes nearly as much as the fossil fuel-engined car did in the last century. Electrical cars would of course pollute far less than fossil fuel-driven ones; instead of oil reserves, the rarest materials for batteries would make undeserving despots and their dynasties fantastically rich. Petrol stations would disappear. The air in cities would once more be breathable and their streets as quiet as those of Venice. This isn't an unmixed good. Cars that were as silent as bicycles would still be as dangerous as they are now to anyone they hit without audible warning.

The dream goes further than that. The electric cars of the future will be so thoroughly equipped with sensors and reaction mechanisms that they will never hit anyone. Just as brakes don't let you skid today, the steering wheel of tomorrow will swerve you away from danger before you have even noticed it...

This is where the fantasy of autonomy comes full circle. The logical outcome of cars which need no driver is that they will become cars which need no owner either. Instead, they will work as taxis do, summoned at will but only for the journeys we actually need. This the future towards which Uber...is working. The ultimate development of the private car will be to reinvent public transport. Traffic jams will be abolished only when the private car becomes a public utility. What then will happen to our fantasies of independence? We' ll all have to take to electrically powered bicycles.

  1. CAT 2017 Question Paper - Slot 2

    Which of the following statements best reflects the author's argument?

    1. Hybrid and electric vehicles signal the end of the age of internal combustion engines.
    2. Elon Musk is a remarkably gifted salesman.
    3. The private car represents an unattainable myth of independence.
    4. The future Uber car will be environmentally friendlier than even the Tesla.
    Choice C
    The private car represents an unattainable myth of independence.

  2. CAT 2017 Question Paper - Slot 2

    The author points out all of the following about electric cars EXCEPT

    1. Their reliance on rare materials for batteries will support despotic rule.
    2. They will reduce air and noise pollution.
    3. They will not decrease the number of traffic jams.
    4. They will ultimately undermine rather than further driver autonomy.
    Choice D
    They will ultimately undermine rather than further driver autonomy

  3. CAT 2017 Question Paper - Slot 2

    According to the author, the main reason for Tesla's remarkable sales is that

    1. in the long run, the Tesla is more cost effective than fossil fuel-driven cars.
    2. the US government has announced a tax subsidy for Tesla buyers.
    3. the company is rapidly upscaling the number of specialised charging stations for customer convenience.
    4. people believe in the autonomy represented by private cars.
    Choice D
    people believe in the autonomy represented by private cars.

  4. CAT 2017 Question Paper - Slot 2

    The author comes to the conclusion that

    1. car drivers will no longer own cars but will have to use public transport.
    2. cars will be controlled by technology that is more efficient than car drivers.
    3. car drivers dream of autonomy but the future may be public transport.
    4. electrically powered bicycles are the only way to achieve autonomy in transportation.
    Choice C
    car drivers dream of autonomy but the future may be public transport.

  5. CAT 2017 Question Paper - Slot 2

    In paragraphs 5 and 6, the author provides the example of Uber to argue that

    1. in the future, electric cars will be equipped with mechanisms that prevent collisions.
    2. in the future, traffic jams will not exist.
    3. in the future, the private car will be transformed into a form of public transport.
    4. in the future, Uber rides will outstrip Tesla sales.
    Choice C
    in the future, the private car will be transformed into a form of public transport.

  6. CAT 2017 Question Paper - Slot 2

    In paragraph 6, the author mentions electrically powered bicycles to argue that

    1. if Elon Musk were a true visionary, he would invest funds in developing electric bicycles.
    2. our fantasies of autonomy might unexpectedly require us to consider electric bicycles.
    3. in terms of environmental friendliness and safety, electric bicycles rather than electric cars are the future.
    4. electric buses are the best form of public transport.
    Choice B
    our fantasies of autonomy might unexpectedly require us to consider electric bicycles.

Passage 4: Typewriters

Typewriters are the epitome of a technology that has been comprehensively rendered obsolete by the digital age. The ink comes off the ribbon, they weigh a ton, and second thoughts are a disaster. But they are also personal, portable and, above all, private. Type a document and lock it away and more or less the only way anyone else can get it is if you give it to them. That is why the Russians have decided to go back to typewriters in some government offices, and why in the US, some departments have never abandoned them. Yet it is not just their resistance to algorithms and secret surveillance that keeps typewriter production lines — well one, at least — in business (the last British one closed a year ago). Nor is it only the nostalgic appeal of the metal body and the stout well-defined keys that make them popular on eBay. A typewriter demands something particular: attentiveness. By the time the paper is loaded, the ribbon tightened, the carriage returned, the spacing and the margins set, there's a big premium on hitting the right key. That means sorting out ideas, pulling together a kind of order and organising details before actually striking off. There can be no thinking on screen with a typewriter. Nor are there any easy distractions. No online shopping. No urgent emails. No Twitter. No need even for electricity — perfect for writing in a remote hideaway. The thinking process is accompanied by the encouraging clack of keys, and the ratchet of the carriage return. Ping!

  1. CAT 2017 Question Paper - Slot 2

    Which one of the following best describes what the passage is trying to do?

    1. It describes why people continue to use typewriters even in the digital age.
    2. It argues that typewriters will continue to be used even though they are an obsolete technology.
    3. It highlights the personal benefits of using typewriters.
    4. It shows that computers offer fewer options than typewriters.
    Choice A
    It describes why people continue to use typewriters even in the digital age.

  2. CAT 2017 Question Paper - Slot 2

    According to the passage, some governments still use typewriters because:

    1. they do not want to abandon old technologies that may be useful in the future.
    2. they want to ensure that typewriter production lines remain in business.
    3. they like the nostalgic appeal of typewriter.
    4. they can control who reads the document.
    Choice D
    they can control who reads the document.

  3. CAT 2017 Question Paper - Slot 2

    The writer praises typewriters for all the following reasons EXCEPT

    1. Unlike computers, they can only be used for typing.
    2. You cannot revise what you have typed on a typewriter.
    3. Typewriters are noisier than computers.
    4. Typewriters are messier to use than computers.
    Choice D
    Typewriters are messier to use than computers.

Passage 5: Viking age

Despite their fierce reputation, Vikings may not have always been the plunderers and pillagers popular culture imagines them to be. In fact, they got their start trading in northern European markets, researchers suggest.

Combs carved from animal antlers, as well as comb manufacturing waste and raw antler material has turned up at three archaeological sites in Denmark, including a medieval marketplace in the city of Ribe. A team of researchers from Denmark and the U.K. hoped to identify the species of animal to which the antlers once belonged by analyzing collagen proteins in the samples and comparing them across the animal kingdom, Laura Geggel reports for LiveScience. Somewhat surprisingly, molecular analysis of the artifacts revealed that some combs and other material had been carved from reindeer antlers.... Given that reindeer (Rangifer tarandus) don't live in Denmark, the researchers posit that it arrived on Viking ships from Norway. Antler craftsmanship, in the form of decorative combs, was part of Viking culture. Such combs served as symbols of good health, Geggel writes. The fact that the animals shed their antlers also made them easy to collect from the large herds that inhabited Norway.

Since the artifacts were found in marketplace areas at each site it's more likely that the Norsemen came to trade rather than pillage. Most of the artifacts also date to the 780s, but some are as old as 725. That predates the beginning of Viking raids on Great Britain by about 70 years. (Traditionally, the so-called "Viking Age" began with these raids in 793 and ended with the Norman conquest of Great Britain in l066.) Archaeologists had suspected that the Vikings had experience with long maritime voyages [that] might have preceded their raiding days. Beyond Norway, these combs would have been a popular industry in Scandinavia as wela: It' s possible that the antler combs represent a larger trade network, where the Norsemen supplied raw material to craftsmen in Denmark and elsewhere.

  1. CAT 2017 Question Paper - Slot 2

    The primary purpose of the passage is:

    1. to explain the presence of reindeer antler combs in Denmark.
    2. to contradict the widely-accepted beginning date for the Viking Age in Britain, and propose an alternate one.
    3. to challenge the popular perception of Vikings as raiders by using evidence that suggests their early trade relations with Europe.
    4. to argue that besides being violent pillagers„Vikings were also skilled craftsmen and efficient traders.
    Choice C
    to challenge the popular perception of Vikings as raiders by using evidence that suggests their early trade relations with Europe.

  2. CAT 2017 Question Paper - Slot 2

    The evidence - "Most of the artifacts also date to the 780s, but some are as old as 725" — has been used in the passage to argue that:

    1. the beginning date of the Viking Age should be changed from 793 to 725.
    2. the Viking raids started as early as 725.
    3. some of the antler artifacts found in Denmark and Great Britain could have come from Scandinavia.
    4. the Vikings' trade relations with Europe pre-dates the Viking raids.
    Choice D
    the Vikings' trade relations with Europe pre-dates the Viking raids.

  3. CAT 2017 Question Paper - Slot 2

    All of the following hold true for Vikings EXCEPT

    1. Vikings brought reindeer from Norway to Denmark for trade purposes.
    2. Before becoming the raiders of northern Europe, Vikings had trade relations with European nations.
    3. Antler combs, regarded by the Vikings as a symbol of good health, were part of the Viking culture.
    4. Vikings, once upon a time, had trade relations with Denmark and Scandinavia.
    Choice A
    Vikings brought reindeer from Norway to Denmark for trade purposes.


CAT Questions | CAT Quantitative Aptitude

CAT Questions | CAT DILR

CAT Questions | Verbal Ability for CAT


Where is 2IIM located?

2IIM Online CAT Coaching
A Fermat Education Initiative,
58/16, Indira Gandhi Street,
Kaveri Rangan Nagar, Saligramam, Chennai 600 093

How to reach 2IIM?

Phone: (91) 44 4505 8484
Mobile: (91) 99626 48484
WhatsApp: WhatsApp Now
Email: prep@2iim.com